CH33 , 37,43 Med Surg ctc adn

Lakukan tugas rumah & ujian kamu dengan baik sekarang menggunakan Quizwiz!

Thoracic Aneurysm

Balloon-like dilatation of the thoracic aorta dilation of the aortic root can lead to regurg also can cause thrombosis due to turbulent flow Cause commonly is tertiary syph wich causes treebark like appearnce of the aorta and fibrosis of the vaso vasorum leading to weakness Balloon-like dilatation of the thoracic aorta dilation of the aortic root can lead to regurg also can cause thrombosis due to turbulent flow Cause commonly is tertiary syph wich causes treebark like appearnce of the aorta and fibrosis of the vaso vasorum leading to weakness

Aortic Aneurysms Nursing Assessment

Because atherosclerosis is a systemic disease, look for signs of coexisting cardiac, pulmonary, cerebral, and lower extremity vascular problems. Monitor the patient for signs of aneurysm rupture. These include diaphoresis; pallor; weakness; tachycardia; hypotension; abdominal, back, groin, or periumbilical pain; changes in level of consciousness; or a pulsating abdominal mass. Establish baseline data to compare with later assessments. Pay special attention to the character and quality of the patient's peripheral pulses and renal and neurologic status. Before surgery, mark pedal pulse sites (dorsalis pedis, posterior tibial) with a single-use marker and document any skin lesions on the lower extremities.

what process are disrupted with hepatitis

Bile production, coagulation, blood glucose, and protein metabolism can be affected. Detoxification and processing of drugs, hormones, and metabolites (e.g., ammonia from protein catabolism) may be disrupted.

Increase Lipoprotein Removal Bile-Acid Sequestrants cholestyramine (Prevalite) colesevelam (Welchol) colestipol (Colestid)

Binds with bile acids in intestine, forming insoluble complex and excreted in feces Binding results in removal of LDL and cholesterol ↓ LDL S/E:Unpleasant quality to taste, GI problems (e.g., indigestion, constipation, bloating) Nursing considerations: Effective and safe for long-term use. Side effects lessen with time. Interferes with absorption of many drugs (e.g., digoxin, thiazide diuretics, warfarin, some antibiotics [e.g., penicillins])

During thrombolytic therapy, the nurse monitors the patient for adverse effects. What is the most frequent undesirable effect of thrombolytic therapy?

Bleeding, both internal and superficial, as well as intracranial, is the most frequent undesirable effect of thrombolytic therapy

HMG-CoA Reductase Inhibitors (Statins) atorvastatin (Lipitor) fluvastatin (Lescol XL) lovastatin pitavastatin (Livalo) pravastatin (Pravachol) rosuvastatin (Crestor) simvastatin (Zocor)

Block synthesis of cholesterol and increase LDL receptors in liver ↓ LDL ↓ Triglycerides ↑ HDL (small amount) Rash, GI problems, high liver enzymes, myopathy, rhabdomyolysis Well tolerated with few side effects. Monitor liver enzymes and creatine kinase (if muscle weakness or pain occurs).

Non-Cardioselective Blockers nadolol (Corgard) pindolol propranolol (Inderal)

Block β1- and β2-adrenergic receptors Reduce BP by blocking β1- and β2-adrenergic effects except may cause bronchospasm, especially in patients with a history of asthma Consideration: Monitor pulse and BP regularly Use with caution in patients with diabetes because may depress the tachycardia associated with hypoglycemia and adversely affect glucose metabolism Drug of choice for patients with a history of an MI or HF Less effective BP reduction in black patients Esmolol is for IV use only Lose cardioselectivity at higher doses

Etiology and patho Vein ulcers/CVI

Both long-standing primary varicose veins and PTS can progress to CVI. Ambulatory venous hypertension causes serous fluid and RBCs to leak from the capillaries and venules into the tissue. This causes edema and chronic inflammatory changes. Enzymes in the tissue eventually break down RBCs, causing the release of hemosiderin, which causes a brownish skin discoloration. Over time, fibrous tissue replaces the skin and subcutaneous tissue around the ankle. This results in thick, hardened, contracted skin. Although the causes of CVI are known, the exact pathophysiology of venous leg ulcers is unknown.

Arterial bruits with both systolic and diastolic components suggest

Bruits auscultated over one of the renal arteries that have both systolic and diastolic components strongly suggest renal artery stenosis (partial occlusion) as a cause of the patient's hypertension.

After a patient has had a transjugular intrahepatic portosystemic shunt (TIPS) placement,which finding indicates that the procedure has been effective?a.Lower indirect bilirubin level b.Increase in serum albumin level c.Decrease in episodes of variceal bleeding d.Improvement in alertness and orientation

C

The condition of the pt who has cirrhosis of the liver has deteriorated. Which diagnostic study would help determine if the pt has developed liver cancer? a. serum a-fetoprotein level b. ventilation/perfusion scan c. hepatic structure ultasound d. abdominal girth measurement

C

Arterial AAA Interventions

1) Frequently assess distal pulses and capillary refill 2) Ensure urine output >30cc/hr 3) be alert for complications 4) Report all changes to physician

Pathogenesis of atherosclerosis.

1. Damage to endothelium allows lipids to leak into intima 2. Lipids are oxidized and then consumed by macrophages via scavenger receptors, resulting in foam cells (leads to fatty streak) 3. Inflammation and healing lead to deposition of extracellular matrix and proliferation of smooth muscle (leads to fibromuscular cap)

Damaged endothelium exposes collagen

1. Platelets bind to collagen. 2. Von Willebrand factor holds them there. 3. Platelets recruit more platelets and form a platelet plug by secreting: - ADP (sticky platelets) - Serotonin (vasoconstriction) - Thromboxane A (sticky platelets and vasoconstriction)

Which patient statement to the nurse is most consistent with the diagnosis of venous insufficiency? a. "I can't get my shoes on at the end of the day." b. "I can't ever seem to get my feet warm enough." c. "I have burning leg pains after I walk two blocks." d. "I wake up during the night because my legs hurt."

ANS: A Because the edema associated with venous insufficiency increases when the patient has been standing, shoes will feel tighter at the end of the day. The other patient statements are characteristic of peripheral artery disease

The nurse is caring for a patient with critical limb ischemia who has just arrived on the nursing unit after having percutaneous transluminal balloon angioplasty. Which action should the nurse perform first? a. Obtain vital signs. b. Teach wound care. c. Assess pedal pulses.d. Check the wound site.

ANS: A Bleeding is a possible complication after catheterization of the femoral artery, so the nurse's first action should be to assess for changes in vital signs that might indicate hemorrhage. The other actions are also appropriate but can be done after determining that bleeding is not occurring

Which laboratory test result will the nurse monitor to evaluate the effects of therapy for a patient who has acute pancreatitis? a. Lipase b. Calcium c. Bilirubin d. Potassium

ANS: A Lipase is elevated in acute pancreatitis. Although changes in the other values may occur, they would not be useful in evaluating whether the prescribed therapies have been effective

Which action will the nurse include in the plan of care for a patient who has been diagnosed with chronic hepatitis B? a. Advise limiting alcohol intake to 1 drink daily. b. Schedule for liver cancer screening every 6 months. c. Initiate administration of the hepatitis C vaccine series. d. Monitor anti-hepatitis B surface antigen (anti-HBs) levels.

ANS: B Patients with chronic hepatitis are at higher risk for development of liver cancer and should be screened for liver cancer every 6 to 12 months. Patients with chronic hepatitis are advised to completely avoid alcohol. There is no hepatitis C vaccine. Because anti-HBs is present whenever there has been a past hepatitis B infection or vaccination, there is no need to regularly monitor for this antibody.

Which action will the nurse include in the plan of care for a patient who has been diagnosed with chronic hepatitis B? a. Advise limiting alcohol intake to 1 drink daily. b. Schedule for liver cancer screening every 6 months. c. Initiate administration of the hepatitis C vaccine series. d. Monitor anti-hepatitis B surface antigen (anti-HBs) levels.

ANS: B Patients with chronic hepatitis are at higher risk for development of liver cancer and should be screened for liver cancer every 6 to 12 months. Patients with chronic hepatitis are advised to completely avoid alcohol. There is no hepatitis C vaccine. Because anti-HBs is present whenever there has been a past hepatitis B infection or vaccination, there is no need to regularly monitor for this antibody. DIF: Cognitive Level: Apply (application) REF: 984 TOP: Nursing Process: Planning MSC: NCLEX: Physiological Integrity

The nurse evaluates that administration of hepatitis B vaccine to a healthy patient has been effective when the patient's blood specimen reveals: a. HBsAg. b. anti-HBs. c. anti-HBc IgG. d. anti-HBc IgM

ANS: B The presence of surface antibody to HBV (anti-HBs) is a marker of a positive response to the vaccine. The other laboratory values indicate current infection with HBV

The nurse evaluates that administration of hepatitis B vaccine to a healthy patient has beeneffective when the patient's blood specimen reveals a. HBsAg. b. anti-HBs c. anti-HBc IgG. d. anti-HBc IgM.

ANS: B The presence of surface antibody to HBV (anti-HBs) is a marker of a positive response to thevaccine. The other laboratory values indicate current infection with HBV.DIF: Cognitive Level: Apply (application) REF: 980TOP: Nursing Process: Evaluation MSC: NCLEX: Health Promotion and Maintenance

To detect possible complications in a patient with severe cirrhosis who has bleeding esophageal varices, it is most important for the nurse to monitor a. bilirubin levels. b. ammonia levels. c. potassium levels. d. prothrombin time.

ANS: B The protein in the blood in the gastrointestinal tract will be absorbed and may result in an increase in the ammonia level because the liver cannot metabolize protein very well. The prothrombin time, bilirubin, and potassium levels should also be monitored, but they will not be affected by the bleeding episode. DIF: Cognitive Level: Analyze (analysis) REF: 990 TOP: Nursing Process: Assessment MSC: NCLEX: Physiological Integrity

A patient who has cirrhosis and esophageal varices is being treated with propranolol (Inderal). Which finding is the best indicator to the nurse that the medication has been effective? a.The patient reports no chest pain. b.Blood pressure is 130/80 mm Hg. c.Stools test negative for occult blood. d.The apical pulse rate is 68 beats/min.

ANS: C Because the purpose of -blocker therapy for patients with esophageal varices is to decrease the risk for bleeding from esophageal varices, the best indicator of the effectiveness for propranolol is the lack of blood in the stools. Although propranolol is used to treat hypertension, angina, and tachycardia, the purpose for use in this patient is to decrease the risk for bleeding from esophageal varices

A patient with cirrhosis and esophageal varices has a new prescription for propranolol (Inderal). Which finding is the best indicator to the nurse that the medication has been effective? a. The patient reports no chest pain. b. Blood pressure is 140/90 mm Hg. c. Stools test negative for occult blood. d. The apical pulse rate is 68 beats/minute.

ANS: C Because the purpose of -blocker therapy for patients with esophageal varices is to decrease the risk for bleeding from esophageal varices, the best indicator of the effectiveness for propranolol is the lack of blood in the stools. Although propranolol is used to treat hypertension, angina, and tachycardia, the purpose for use in this patient is to decrease the risk for bleeding from esophageal varices. DIF: Cognitive Level: Analyze (analysis) REF: 993 TOP: Nursing Process: Evaluation MSC: NCLEX: Physiological Integrity

A 46-yr-old service-counter worker undergoes sclerotherapy for treatment of superficial varicose veins at an outpatient center. Which instructions should the nurse provide to the patient before discharge? a. Sitting at the work counter, rather than standing, is recommended. b. Exercise, such as walking or jogging, can cause recurrence of varicosities. c. Elastic compression stockings should be applied before getting out of bed. d. Taking an aspirin daily will help prevent clots from forming around venous valves.

ANS: C Elastic compression stockings are applied with the legs elevated to reduce pressure in the lower legs. Walking is recommended to prevent recurrent varicosities. Sitting and standing are both risk factors for varicose veins and venous insufficiency. An aspirin a day is not adequate to prevent venous thrombosis and would not be recommended for a patient who had just had sclerotherapy

Which assessment finding for a patient who has been admitted with a right calf venous thromboembolism (VTE) requires immediate action by the nurse? a. Erythema of right lower leg b. Complaint of right calf pain c. New onset shortness of breath d. Temperature of 100.4°F (38°C)

ANS: C New onset dyspnea suggests a pulmonary embolus, which will require rapid actions such as O2 administration and notification of the health care provider. The other findings are typical of VTE.

Which action by a new nurse who is giving fondaparinux (Arixtra) to a patient with a lower leg venous thromboembolism (VTE) indicates that more education about the drug is needed? a. The nurse avoids rubbing the injection site after giving the drug. b. The nurse injects the drug into the abdominal subcutaneous tissue. c. The nurse ejects the air bubble from the syringe before giving the drug. d. The nurse does not check partial thromboplastin time (PTT) before giving the drug.

ANS: C The air bubble is not ejected before giving fondaparinux to avoid loss of drug. The other actions by the nurse are appropriate for subcutaneous administration of a low molecular weight heparin (LMWH). LMWHs typically do not require ongoing PTT monitoring and dose adjustment.

Which action should the nurse in the emergency department take first for a new patient who is vomiting blood? a. Insert a large-gauge IV catheter. b. Draw blood for coagulation studies. c. Check blood pressure and heart rate. d. Place the patient in the supine position

ANS: C The nurse's first action should be to determine the patient's hemodynamic status by assessing vital signs. Drawing blood for coagulation studies and inserting an IV catheter are also appropriate. However, the vital signs may indicate the need for more urgent actions. Because aspiration is a concern for this patient, the nurse will need to assess the patient's vital signs and neurologic status before placing the patient in a supine position.

To prepare a patient with ascites for paracentesis, the nurse a. places the patient on NPO status. b. assists the patient to lie flat in bed. c. asks the patient to empty the bladder. d. positions the patient on the right side.

ANS: C The patient should empty the bladder to decrease the risk of bladder perforation during the procedure. The patient would be positioned in Fowler's position and would not be able to lie flat without compromising breathing. Because no sedation is required for paracentesis, the patient does not need to be NPO. DIF: Cognitive Level: Apply (application) REF: 994 TOP: Nursing Process: Implementation MSC: NCLEX: Physiological Integrity

When discussing risk factor modification for a 63-year-old patient who has a 5-cm abdominal aortic aneurysm, the nurse will focus discharge teaching on which patient risk factor? a. Male gender b. Turner syndrome c. Abdominal trauma history d. Uncontrolled hypertension

ANS: D All of the factors contribute to the patient's risk, but only hypertension can potentially be modified to decrease the patient's risk for further expansion of the aneurysm.

What risk factor will the nurse specifically ask about when a patient is being admitted with acute pancreatitis? a. Diabetes b. Alcohol use c. High-protein diet d. Cigarette smoking

ANS: D Alcohol use is one of the most common risk factors for pancreatitis in the United States. Cigarette smoking, diabetes, and high-protein diets are not risk factors.

A patient with chronic hepatitis B infection has several medications prescribed. Which medication order requires clarification with the health care provider before administration? a. Tenofovir (Viread) orally once daily b. Adefovir (Hepsera) orally once daily c. Entecarvir (Baraclude) orally once daily d. Pegylated -interferon (Pegasys) orally once daily

ANS: D Pegylated -interferon is administered subcutaneously, not orally. The medications are all appropriate for a patient with chronic hepatitis B infection

Direct Vasodilators fenoldopam (Corlopam)

Activates dopamine receptors, resulting in systemic and renal vasodilation IV use only for hypertensive crisis in hospitalized patients Use cautiously in patients with glaucoma Patient should remain flat for 1 hr after administration

Nonmodifiable risk factors for CAD

Age Gender Ethnicity Family history Genetic predisposition

Auscultation

An artery that is narrowed or has a bulging wall may create turbulent blood flow. This abnormal flow can cause a buzzing or humming termed a bruit, heard with the bell of the stethoscope over the vessel. Auscultate major arteries such as the carotid arteries, abdominal aorta, and femoral arteries as part of the initial cardiovascular assessment.

Peripheral Artery disease symptomps

Ankle-brachial index ≤0.90 Capillary refill >3 sec Dermatitis:Rarely occurs Edema:Absent unless leg constantly in dependent position Hair: Loss of hair on legs, feet, toes Nails:Thickened, brittle Pain:Intermittent claudication or rest pain in foot.Ulcer may or may not be painful Peripheral pulses:Decreased or absent Pruritus:Rarely occurs Skin color:Dependent rubor, elevation pallor Skin temperature: Cool, temperature gradient down the leg Skin texture: Thin, shiny, taut Ulcer • Location Tips of toes, foot, or lateral malleolus,Drainage Minimal,Tissue Black eschar or pale pink granulation

A nurse is creating an education plan for a patient with venous insufficiency. What measure should the nurse include in the plan? A) Avoiding tight-fitting socks. B) Limit activity whenever possible. C) Sleep with legs in a dependent position. D) Avoid the use of pressure stockings.

Ans: A Feedback: Measures taken to prevent complications include avoiding tight-fitting socks and panty girdles; maintaining activities, such as walking, sleeping with legs elevated, and using pressure stockings. Not included in the teaching plan for venous insufficiency would be reducing activity, sleeping with legs dependent, and avoiding pressure stockings. Each of these actions exacerbates venous insufficiency.

A patient with end-stage liver disease has developed hypervolemia. What nursing interventions would be most appropriate when addressing the patient's fluid volume excess? Select all that apply. A) Administering diuretics B) Administering calcium channel blockers C) Implementing fluid restrictions D) Implementing a 1500 kcal/day restriction E) Enhancing patient positioning

Ans: A, C, E Feedback: Administering diuretics, implementing fluid restrictions, and enhancing patient positioning can optimize the management of fluid volume excess. Calcium channel blockers and calorie restriction do not address this problem.

The nurse is providing care for a patient with a new diagnosis of hypertension. How can the nurse best promote the patient's adherence to the prescribed therapeutic regimen? A) Screen the patient for visual disturbances regularly. B) Have the patient participate in monitoring his or her own BP. C) Emphasize the dire health outcomes associated with inadequate BP control. D) Encourage the patient to lose weight and exercise regularly.

Ans: B Feedback: Adherence to the therapeutic regimen increases when patients actively participate in self-care, including self-monitoring of BP and diet. Dire warnings may motivate some patients, but for many patients this is not an appropriate or effective strategy. Screening for vision changes and promoting healthy lifestyle are appropriate nursing actions, but do not necessarily promote adherence to a therapeutic regimen.

A participant in a health fair has asked the nurse about the role of drugs in liver disease. What health promotion teaching has the most potential to prevent drug-induced hepatitis? A) Finish all prescribed courses of antibiotics, regardless of symptom resolution. B) Adhere to dosing recommendations of OTC analgesics. C) Ensure that expired medications are disposed of safely. D) Ensure that pharmacists regularly review drug regimens for potential interactions.

Ans: B Feedback: Although any medication can affect liver function, use of acetaminophen (found in many over-the-counter medications used to treat fever and pain) has been identified as the leading cause of acute liver failure. Finishing prescribed antibiotics and avoiding expired medications are unrelated to this disease. Drug interactions are rarely the cause of drug-induced hepatitis.

The nursing lab instructor is teaching student nurses how to take blood pressure. To ensure accurate measurement, the lab instructor would teach the students to avoid which of the following actions? A) Measuring the BP after the patient has been seated quietly for more than 5 minutes B) Taking the BP at least 10 minutes after nicotine or coffee ingestion C) Using a cuff with a bladder that encircles at least 80% of the limb D) Using a bare forearm supported at heart level on a firm surface

Ans: B Feedback: Blood pressures should be taken with the patient seated with arm bare, supported, and at heart level. The patient should not have smoked tobacco or taken caffeine in the 30 minutes preceding the measurement. The patient should rest quietly for 5 minutes before the reading is taken. The cuff bladder should encircle at least 80% of the limb being measured and have a width of at least 40% of limb circumference. Using a cuff that is too large results in a lower BP and a cuff that is too small will give a higher BP measurement.

The nurse is providing care for a patient who has just been diagnosed with peripheral arterial occlusive disease (PAD). What assessment finding is most consistent with this diagnosis? A) Numbness and tingling in the distal extremities B) Unequal peripheral pulses between extremities C) Visible clubbing of the fingers and toes D) Reddened extremities with muscle atrophy

Ans: B Feedback: PAD assessment may manifest as unequal pulses between extremities, with the affected leg cooler and paler than the unaffected leg. Intermittent claudication is far more common than sensations of numbness and tingling. Clubbing and muscle atrophy are not associated with PAD

A patient with a history of injection drug use has been diagnosed with hepatitis C. When collaborating with the care team to plan this patient's treatment, the nurse should anticipate what intervention? A) Administration of immune globulins B) A regimen of antiviral medications C) Rest and watchful waiting D) Administration of fresh-frozen plasma (FFP)

Ans: B Feedback: There is no benefit from rest, diet, or vitamin supplements in HCV treatment. Studies have demonstrated that a combination of two antiviral agents, Peg-interferon and ribavirin (Rebetol), is effective in producing improvement in patients with hepatitis C and in treating relapses. Immune globulins and FFP are not indicated.

A patient has been diagnosed as being prehypertensive. What should the nurse encourage this patient to do to aid in preventing a progression to a hypertensive state? A) Avoid excessive potassium intake. B) Exercise on a regular basis. C) Eat less protein and more vegetables. D) Limit morning activity.

Ans: B Feedback: To prevent or delay progression to hypertension and reduce risk, JNC 7 urged health care providers to encourage people with blood pressures in the prehypertension category to begin lifestyle modifications, such as nutritional changes and exercise. There is no need for patients to limit their activity in the morning or to avoid potassium and protein intake.

The prevention of VTE is an important part of the nursing care of high-risk patients. When providing patient teaching for these high-risk patients, the nurse should advise lifestyle changes, including which of the following? Select all that apply. A) High-protein diet B) Weight loss C) Regular exercise D) Smoking cessation E) Calcium and vitamin D supplementation

Ans: B, C, D Feedback: Patients at risk for VTE should be advised to make lifestyle changes, as appropriate, which may include weight loss, smoking cessation, and regular exercise. Increased protein intake and supplementation with vitamin D and calcium do not address the main risk factors for VTE.

A patient with a diagnosis of cirrhosis has developed variceal bleeding and will imminently undergo variceal banding. What psychosocial nursing diagnosis should the nurse most likely prioritize during this phase of the patient's treatment? A) Decisional Conflict B) Deficient Knowledge C) Death Anxiety D) Disturbed Thought Processes

Ans: C Feedback: The sudden hemorrhage that accompanies variceal bleeding is intensely anxiety-provoking. The nurse must address the patient's likely fear of death, which is a realistic possibility. For most patients, anxiety is likely to be a more acute concern than lack of knowledge or decisional conflict. The patient may or may not experience disturbances in thought processes.

A nurse is performing an admission assessment of a patient with a diagnosis of cirrhosis. What technique should the nurse use to palpate the patient's liver? A) Place hand under the right lower abdominal quadrant and press down lightly with the other hand. B) Place the left hand over the abdomen and behind the left side at the 11th rib. C) Place hand under right lower rib cage and press down lightly with the other hand. D) Hold hand 90 degrees to right side of the abdomen and push down firmly.

Ans: C Feedback: To palpate the liver, the examiner places one hand under the right lower rib cage and presses downward with light pressure with the other hand. The liver is not on the left side or in the right lower abdominal quadrant.

A nurse is performing blood pressure screenings at a local health fair. While obtaining subjective assessment data from a patient with hypertension, the nurse learns that the patient has a family history of hypertension and she herself has high cholesterol and lipid levels. The patient says she smokes one pack of cigarettes daily and drinks about a pack of beer every day. The nurse notes what nonmodifiable risk factor for hypertension? A) Hyperlipidemia B) Excessive alcohol intake C) A family history of hypertension D) Closer adherence to medical regimen

Ans: C Feedback:Unlike cholesterol levels, alcohol intake and adherence to treatment, family history is not modifiable

An older adult patient has been treated for a venous ulcer and a plan is in place to prevent the occurrence of future ulcers. What should the nurse include in this plan? A) Use of supplementary oxygen to aid tissue oxygenation B) Daily use of normal saline compresses on the lower limbs C) Daily administration of prophylactic antibiotics D) A high-protein diet that is rich in vitamins

Ans: D Feedback: A diet that is high in protein, vitamins C and A, iron, and zinc is encouraged to promote healing and prevent future ulcers. Prophylactic antibiotics and saline compresses are not used to prevent ulcers. Oxygen supplementation does not prevent ulcer formation.

A nurse is amending a patient's plan of care in light of the fact that the patient has recently developed ascites. What should the nurse include in this patient's care plan? A) Mobilization with assistance at least 4 times daily B) Administration of beta-adrenergic blockers as ordered C) Vitamin B12 injections as ordered D) Administration of diuretics as ordered

Ans: D Feedback: Use of diuretics along with sodium restriction is successful in 90% of patients with ascites. Beta-blockers are not used to treat ascites and bed rest is often more beneficial than increased mobility. Vitamin B12 injections are not necessary.

A patient with newly diagnosed hypertension has come to the clinic for a follow-up visit. The patient asks the nurse why she has to come in so often. What would be the nurse's best response? A) "We do this so you don't suffer a stroke." B) "We do this to determine how your blood pressure changes throughout the day." C) "We do this to see how often you should change your medication dose." D) "We do this to make sure your health is stable. We'll then monitor it at routinely scheduled intervals."

Ans: D Feedback: When hypertension is initially detected, nursing assessment involves carefully monitoring the BP at frequent intervals and then at routinely scheduled intervals. The reference to stroke is frightening and does not capture the overall rationale for the monitoring regimen. Changes throughout the day are not a clinical priority for most patients. The patient must not change his or her medication doses unilaterally.

Gemfibrozil

Antihyperlipidemic

Which topic should the nurse include in patient teaching for a patient with a venous stasis ulcer on the left lower leg?

Application of elastic compression stockings

What will the nurse note about temperature of extremities when the source PVD is arterial? Venous?

Arterial - cool; Venous - warm

PAD pain at rest

As PAD progresses and involves multiple arterial segments, continuous pain develops at rest. Rest pain most often occurs in the foot or toes. It is worse with limb elevation. Rest pain occurs when blood flow is insufficient to meet basic metabolic requirements of the distal tissues. Rest pain occurs more often at night because cardiac output tends to drop during sleep and the limbs are at the level of the heart. Patients often try to achieve pain relief by gravity, dangling the leg over the side of the bed or sleeping in a chair.

ASA therapy is

Aspirin

The pt with advanced cirrhosis asks why his abdomen is so swollen. The nurse's best response is based on the knowledge that a. a lack of clotting factors promotes the collection of blood in the abdominal cavity b. portal hypertension and hypoalbuminemia cause fluid shift into the peritoneal space. c. decreased peristalsis in the GI tract contributes to gas formation and distention of the bowel d. bile salts in the blood irritate the peritoneal membranes, causing edema and pocketing of fluid.

B Ascites is accumulation of serious fluid in peritoneal cavity. With portal hypertension, protein shifts from the blood into the lymph. When the lymph system is unable to carry excess, it leaks thru the liver into the peritoneal cavity. osmotic pressure of the proteins pulls additional fluid into cavity. Second mechanism of ascites if hypoalbuminemia from the liver unable to synthesize albumin, resulting in decreased colloidal oncotic pressure.

The nurse is caring for a patient with a descending aortic dissection. Which assessment finding is mostimportant to report to the health care provider? ​

Blood pressure 137/88 mm Hg

A patient is being evaluated for post-thrombotic syndrome. Which assessment will the nurse perform?

Check for presence of lipodermatosclerosis.

A patient with PVD will need assessment of the affected extremities at regular intervals. What will the nurse check?

Color, temperature, sensation, and pulse quality, pain

The staff educator is teaching ED nurses about hypertensive crisis. The nurse educator should explain that hypertensive urgency differs from hypertensive emergency in what way? A) The BP is always higher in a hypertensive emergency. B) Vigilant hemodynamic monitoring is required during treatment of hypertensive emergencies. C) Hypertensive urgency is treated with rest and benzodiazepines to lower BP. D) Hypertensive emergencies are associated with evidence of target organ damage.

D Hypertensive emergencies are acute, life-threatening BP elevations that require prompt treatment in an intensive care setting because of the serious target organ damage that may occur. Blood pressures are extremely elevated in both urgency and emergencies, but there is no evidence of target organ damage in hypertensive urgency. Extremely close hemodynamic monitoring of the patient's BP is required in both situations. The medications of choice in hypertensive emergencies are those with an immediate effect, such as IV vasodilators. Oral doses of fast-acting agents, such as beta-adrenergic blocking agents, angiotensin-converting enzyme inhibitors, or alpha-agonists, are recommended for the treatment of hypertensive urgencies

What must the nurse teach the patient regarding Tricor and Warfarin prescriptions

Fenofibrate can increase the effects of warfarin and cause you to bleed more easily. If your doctor prescribes these medications together, you may need a dose adjustment in addition to more frequent monitoring of your prothrombin time or International Normalized Ratio (INR) to safely use both medications.

elective aneurysm repair

For elective aneurysm repair surgery, the patient should be well hydrated with normal electrolytes, coagulation, and hematocrit. If the aneurysm ruptures, emergent surgery is required.

Esophageal and Gastric Varices: Interprofessional Care

Goal: prevent bleeding and variceal rupture by reducing portal pressure,Check blood pressure Pt who has esophageal varices should avoid ingesting alcohol, aspirin, and NSAIDs All pts w/ cirrhosis should have an upper endoscopy (esophagogastroduodenoscopy [EGD]) to screen for varices. Diagnosis of esophageal or gastric variceal bleeding needs to be made by endoscopic examination as soon as possible. Pts w/ varices at risk of bleeding are started on a nonselective β-blocker (nadolol [Corgard] or propranolol [Inderal]) to reduce the incidence of hemorrhage. β-Blockers ↓ high portal pressure ↓ risk of rupture When variceal bleeding occurs, the 1st step is to stabilize the pt and manage the airway. IV therapy is initiated and may include administration of blood products Management that involves a comb of drug therapy and endoscopic therapy is more successful than either approach alone

________ is the most common cause of chronic liver disease and liver failure.

HCV About 20% to 30% develop cirrhosis and eventually liver failure and/or liver cancer. Along with chronic HBV, HCV accounts for most cases of liver cancer. HCV hepatitis is the most common reason for liver transplantation in the United States

HCV RNA positive

HCV If recent HCV infection is suspected, HCV RNA testing is usually done because it may take several weeks or longer for HCV antibodies to develop. testing may be used for immunocompromised patients because delayed

Rosuvastatin (Crestor)

HMG-CoA reductase inhibitor is the most potent statin currently available.

What complication does the nurse assess frequently for after surgical intervention for PVD?

Hemorrhage no pulse

A client diagnosed with liver cirrhosis is being treated for an infection. For which complication should the nurse monitor the client? Portal hypertension Hepatic encephalopathy Esophageal varices Wilson disease

Hepatic encephalopathy Hepatic encephalopathy may be aggravated by sepsis secondary to infection, due to increased buildup of toxic substances, in clients with cirrhosis. Portal hypertension, esophageal varices, and Wilson disease (an inherited disorder that causes too much copper to accumulate in the organs) are not caused or aggravated by infection.

S/S hepatic encephalopathy

Hepatic encephalopathy is a neuropsychiatric manifestation of liver disease. Manifestations include: -changes in neurologic and mental responsiveness; - impaired consciousness; and inappropriate behavior, ranging from sleep problems to trouble concentrating to deep coma. -(flapping tremors)asterixis -apraxia (inability to construct simple figures). -hyperventilation, hypothermia, tongue fasciculations, and grimacing and grasping reflexes. -Fetor hepaticus (musty, sweet odor of the patient's breath) Confusion Unkept Mood changes Impaired sleep pattern Somnolence Asterixis

In hepatitis A the presene of IgG without anti-HAV IgM indicates

Hepatitis A IgG without anti-HAV IgM indicates past infection. IgG antibody provides lifelong immunity

What hepatitis can a person has for life

Hepatitis B Carriers continue to be infectious for life

elevated amonia

High ammonia levels in the blood can lead to serious health problems, including brain damage, coma, and even death. High ammonia levels in the blood are most often caused by liver disease. Other causes include kidney failure and genetic disorders.

Risk Factors for CAD

Hypertension (HTN) Normal BP <120/<80 mm Hg Elevated BP 120 to 129/<80 mm Hg Stage 1 HTN 130 to 139/80 to 89 mm Hg Stage 2 HTN >140/>90 mm Hg Lifestyle changes for elevated BP and HTN; treat stage 1 or 2 HTN with drugs Elevated BP—endothelial injury leads to left ventricular hypertrophy and reduced stroke volume

The nurse has started discharge teaching for a patient who is to continue warfarin (Coumadin) following hospitalization for venous thromboembolism (VTE). The nurse determines that additional teaching is needed when the patient says which of the following?

I should reduce the amount of green, leafy vegetables that I eat.

Thrombolytic Agents Assessment Toxicity and Overdose (High Alert):

If local bleeding occurs, apply pressure to site. Do not administer dextran; it has antiplatelet activity. Aminocaproic acid (Amicar) may be used as an antidote.

EVAR complications include

Iliac artery injury Endograft limb thrombosis Endoleak such as paraplegia and death. endoleak

HBV genotyping

Indicates the genotype of HBV

Give several applicable nursing diagnoses for the patient with PVD.

Ineffective tissue perfusion, impaired skin integrity, risk for infection, pain

Thrombolytic Agents Con't Evaluation:

Lysis of thrombi and restoration of blood flow. Prevention of neurologic sequelae in acute ischemic stroke. Cannula or catheter patency

Loop Diuretics bumetanide (Bumex) furosemide (Lasix) torsemide (Demadex)

MOA: Inhibit NaCl reabsorption in the ascending limb of the loop of Henle Increase excretion of Na+ and Cl− More potent diuretic effect than thiazides, but shorter duration of action Monitor for orthostatic hypotension and electrolyte abnormalities Remain effective despite renal insufficiency NC: Diuretic effect increases at higher doses Less effective for hypertension

Potassium-Sparing Diuretics amiloride (Midamor) triamterene (Dyrenium)

MOA: Reduce K+ and Na+ exchange in the distal and collecting tubules Reduce excretion of K+, H+, Ca++, and Mg++ NC: Monitor for orthostatic hypotension and hyperkalemia Contraindicated in patients with renal failure Use with caution in patients on ACE inhibitors or angiotensin II blockers Avoid potassium supplements

HBsAg (hepatitis B surface antigen)

Marker of infectivity Present in acute or chronic infection Positive in chronic carriers

OARs

Open aneurysm repair

Opioid morphine

Used to treat pain/discomfort from an MI • Functions as an analgesic and sedative • Acts as a vasodilator to reduce preload and myocardial O2 consumption

cachexia

a condition of physical wasting away due to the loss of weight and muscle mass that occurs in patients with diseases such as advanced cancer or AIDS

Anti-HAV immunoglobulin M (IgM) indicates

acute infection DIAGNOSTIC TEST FOR HEP A

Supportive drug therapy may include for hepatitis

antihistamines for generalized itching and antiemetics for nausea, such as prochlorperazine (Compazine), promethazine (Phenergan), or ondansetron (Zofran)

supportive drug therapy for hepatitis

antihistamines for generalized itching and antiemetics for nausea. These drugs include promethazine (Phenergan) and ondansetron (Zofran).

What difference will the nurse note when assessing pulses of arterial PVD vs. venous PVD?

arterial - decreased or absent; venous -normal

A 23-year-old patient tells the health care provider about experiencing cold, numb fingers when running during the winter and Raynauds phenomenon is suspected. The nurse will anticipate teaching the patient about tests for

autoimmune disorders. Secondary Raynaud's phenomenon may occur in conjunction with autoimmune diseases such as rheumatoid arthritis. Patients should be screened for autoimmune disorders. Raynaud's phenomenon is not associated with hyperlipidemia, hyperglycemia, or coronary artery disease.DIF: Cognitive Level: Apply (application) REF: 809TOP: Nursing Process: Planning MSC: NCLEX: Physiological

The health care provider prescribes an infusion of heparin (Hep-Lock) and daily partial thromboplastin time (PTT) testing for a patient with venous thromboembolism (VTE). The nurse will plan to

avoid giving any IM medications to prevent localized bleeding.

nonalcoholic fatty liver disease (NAFLD),

can cause cirrhosis.

Describe ulcers associated with PVD of venous origin.

slightly painful, on medial legs and ankles, uneven edges, superficial, marked edema Necrotic crater-like lesion usually found on lower leg at medial malleolus. Characterized by slow wound healing

potassium-sparing diuretics

spironolactone, triamterene, amiloride

cervical lymph node

superficial lymph nodes located on the side of the neck

Palmar erythema

unusual redness of the palms of the hands that can occur during pregnancy

Thrombolytic Agents reteplase (Retavase) alteplase (Activase) tenecteplase (TNKase)

• Breaks up fibrin meshwork in clots • Used only in ST-segment-elevation MI when access to a hospital with PCI capability is not available

goals for a patient undergoing aortic surgery

(1) normal tissue perfusion; (2) intact motor and sensory function; and (3) no complications related to surgical repair, such as thrombosis, infection, or rupture.

Palmar erythema:

(red area that blanches w/ pressure) is located on the palms of the hands. Both of these lesions are attributed to ↑ in circulating estrogen as a result of the damaged liver's inability to metabolize steroid hormones

The five Ps—pain

, pallor, pulselessness, paresthesias, and paralysis—characterize an occlusion. Venous congestion causes tissue changes that indicate an inadequate circulatory flow back to the heart.

chronic thromboembolic pulmonary hypertension

- due to chronic PE - best test is VQ scan, NOT CT scan - PCWP normal - PA and RA increased Pulm HTN can have systolic lift at left sternal border blood vessel are narrowed with recurrent blood cloth Result of recurrent or unresolved PE Sx: dyspnea, exertion - constant Diagnosis: - Unexplained dyspnea on exercise - V/Q scan show multiple large defects

what is the purpose of TIPS

- to lower portal hypertension - avoid gastroesophageal varices - reduce ascites accumulation

PAD treatment

-Lifestyle modification (weight loss, smoking cessation, exercise) -Phosphodiesterase inhibitors (cilostazol) -Antiplatelet agents (aspirin) -Angioplasty and stenting -Vascular surgery Bypass ACE inhibitors • Ramipril (Altace) • Antiplatelet agents • Aspirin • Clopidogrel (Plavix) • Medications prescribed for treatment of intermittent claudication • Cilostazol (Pletal) • Pentoxifylline (Trental) exercise, nutritional therapy, The first treatment goal for patients with PAD is to reduce CVD risk factors stop smoking DASH Statins (e.g., simvastatin [Zocor]) and a fibric acid derivative (gemfibrozil [Lopid]) may be used

Spider angiomas are indicative of what conditions?

-Liver disease -Pregnancy -Vit B deficiency (telangiectasia or spider nevi) are small, dilated blood vessels w/ a bright red center point and spiderlike branches and occur on the nose, cheeks, upper trunk, neck, and shoulders.

Hepatitis C virus (HCV)

-can result in both acute illness and chronic infection.For some people, hepatitis C is a short-term illness, but for more than half of people who become infected with the hepatitis C virus, it becomes a long-term, chronic infection. Chronic hepatitis C can result in serious, even life-threatening health problems like cirrhosis and liver cancer.

Angiotensin Inhibitors Angiotensin-Converting Enzyme (ACE) Inhibitors

-pril Angiotensin-converting enzyme (ACE) inhibitors are medications that help relax the veins and arteries to lower blood pressure. ACE inhibitors prevent an enzyme in the body from producing angiotensin II, a substance that narrows blood vessels. Benazepril (Lotensin) Captopril Enalapril (Vasotec) Fosinopril Lisinopril (Prinivil, Zestril) Moexipril Perindopril Quinapril (Accupril) Ramipril (Altace) Trandolapril

Acute Care before surgery, for aneurysm repair

-provide emotional support and teaching to the patient and caregiver. -Preoperative teaching includes a brief explanation of the disease process, the planned surgical procedure(s), preoperative routines, what to expect right after surgery (e.g., recovery room, tubes, drains), -and usual postoperative timelines -After surgery, patients typically go to an ICU for 24 to 48 hours for close monitoring. When the patient arrives in the ICU, various devices are in place. These include an endotracheal tube for mechanical ventilation, an arterial line, a central venous pressure (CVP) catheter, and a nasogastric (NG) tube. The patient needs continuous ECG and pulse oximetry monitoring. If the thorax is opened during surgery, chest tubes will be in place. The patient may have a lumbar catheter draining cerebrospinal fluid to prevent neurologic deficits. Pain medication is given via subcutaneous infusion into incision site (e.g., On-Q pain pump), epidural catheters, or IV patient-controlled analgesia (PCA). Pain medication is given via subcutaneous infusion into incision site (e.g., On-Q pain pump), epidural catheters, or IV patient-controlled analgesia (PCA). maintaining adequate respiratory function, fluid and electrolyte balance, and pain control check for graft patency and renal perfusion. Watch for and intervene to limit or treat cardiac ischemia, dysrhythmias, infections, VTE, and neurologic complications.

Clinical Manifestations and Complications/Acute Hepatitis

1 to 6 months. intermittent or ongoing anorexia, lethargy,nausea, vomiting, skin rashes, diarrhea or constipation, malaise, fatigue, myalgias, arthralgias, other flu-like symptoms, and right upper quadrant tenderness (caused by liver inflammation) sense of smell and find food repugnant Smokers may have distaste for cigarettes. Physical examination often reveals hepatomegaly, lymphadenopathy, abdominal tenderness, and sometimes splenomegaly. icteric (jaundiced) or anicteric. urine may appear darker stools will be clay colored Pruritus (intense generalized itching) convalescent phase: malaise and easy fatigability. Hepatomegaly,,Splenomegaly

Evolocumab (Repatha) Alirocumab (Praluent)

2 PCSK9 inhibitors. reduces the number of receptors on the liver that remove LDL cholesterol from the blood. By blocking PCSK9's ability to work, more receptors are available and can get rid of LDL, thus decreasing circulating LDL levels given subcutaneously every 2 weeks. Evolocumab may be given every 4 weeks at a higher dose, depending on patient preference. They are not a first-line therapy. They are used in addition to diet and maximum statin therapy for adults with familial hypercholesterolemia and for patients intolerant of statin drugs. They also can be used for patients with CAD who need added LDL lowering because of inadequate treatment with statins.

A pt with type 2 diabetes and cirrhosis asks if it would be okay to take silymarin (milk thistle) to help minimize liver damage. The nurse responds based on what knowledge? a. milk thistle may affect liver enzymes and thus alter drug metabolism b. milk thistle is generally safe in recommended doses for up to 10 yrs c. there is unclear scientific evidence for the use of milk thistle in treating cirrhosis d. milk thislte may elevate the serum glucose level and is thus contraindicated in diabetes.

A There is evidence that there is no real benefit from using milk thistle to protect liver enzyme from toxic damage. Milk thistle does affect liver enzyme and could alter drug metabolism. Pt will need to be monitored for drug interactions. It is dafe for up to 6 yrs not 10, and it may lower, not elevate, blood glucose levels.

The pt with cirrhosis is being taught self care. which statement indicates the pt needs MORE teaching? a. if i notice a fast heart rate and irregular beats, this is normal for cirrhosis b. i need to take good care of my belly and ankle skin where it is swollen. c. a scrotal support may be more comfortable when i have scrotal edema d. i can use pillows to support my head to help me breathe when i am in bed.

A This may indicative of hypokalemia and should be reported to healthcare provider, this is not normal for cirrhosis.

Lovastatin (Mevacor) is prescribed for a patient for the first time. The nurse should provide the patient with which instruction? a. "Take lovastatin with your evening meal." b. "Take this medicine before breakfast." c. "You may take lovastatin without regard to meals." d. "Take this medicine on an empty stomach."

A Patients should be instructed to take lovastatin with the evening meal. Statins should be taken with the evening meal, not before breakfast. Statins should not be administered without regard to meals and should not be taken on an empty stomach.

A nurse is reviewing the medications of a patient with diabetes before discharge. The nurse realizes that the patient will be going home on colesevelam, a bile acid sequestrant, and insulin. What patient education should the nurse provide in the discharge teaching for this patient? a. The patient needs to monitor the blood sugar carefully, because colesevelam can cause hypoglycemia. b. The patient needs to monitor the blood sugar carefully, because colesevelam can cause hyperglycemia. c. The patient needs to take the insulin at least 3 hours before the colesevelam. d. The patient needs to use an oral antidiabetic agent or agents, not insulin, with colesevelam.

A Colesevelam can help control hyperglycemia in patients with diabetes; therefore, hypoglycemia is a possible risk. Hyperglycemia is not a risk for patients with diabetes who take colesevelam. Insulin and colesevelam do not interact; therefore, the insulin can be taken at the patient's preferred time or times. Either insulin or oral antidiabetic agents can be taken with colesevelam.

A patient with a history of angina and hypertension is being started on nicotinic acid (Niacin). The nurse is providing patient education. What statement made by the patient demonstrates a need for further teaching? a. "I will take Tylenol with my medication to reduce the inflammatory effects." b. "I will be cautious taking this medication, because I have mild liver damage." c. "I will take an aspirin 30 minutes before my niacin to reduce flushing." d. "This medication will lower my triglyceride levels."

A Intense flushing of the face, neck, and ears occurs in practically all patients taking nicotinic acid in pharmacologic doses. Tylenol will not reduce the flushing or inflammatory effects of the medication; further patient teaching is required. The flushing reaction diminishes in several weeks and can be attenuated by taking 325 mg of aspirin 30 minutes before each dose. Nicotinic acid is hepatotoxic; therefore, the patient should be taught to have the liver enzymes checked and to self-monitor for signs and symptoms. Nicotinic acid reduces low-density lipoprotein (LDL) cholesterol and triglyceride (TG) levels.

intraabdominal hypertension

A potentially lethal complication in an emergency repair of a ruptured AAA is the development of intraabdominal hypertension (IAH) with associated abdominal compartment syndrome. Persistent IAH reduces blood flow to the viscera. Abdominal compartment syndrome refers to the impaired organ perfusion caused by IAH and resulting multisystem organ failure. I

Prevention of infection after aneurysm repair

A prosthetic vascular graft infection is a rare but potentially life-threatening complication. Nursing interventions to prevent infection include giving a broad-spectrum antibiotic as prescribed. Assess temperature regularly, and promptly report elevations. Monitor laboratory results for a high WBC count, which may be the first sign of an infection. Ensure adequate nutrition and assess the surgical incision for signs of infection (e.g., redness, swelling, drainage). Keep surgical incisions clean and dry and perform wound care as prescribed. Use good hand-washing and strict aseptic technique in the care of all peripheral, arterial, and CVP catheter insertion sites, since these are ports of entry for bacteria. Meticulous perineal care for the patient with an indwelling urinary catheter and early catheter removal are essential to minimize the risk for urinary tract infection.

aortic aneurysms.

A variety of disorders are associated with aortic aneurysms. The main causes are classified as degenerative, congenital, mechanical (e.g., penetrating or blunt trauma), inflammatory (e.g., aortitis [Takayasu's arteritis]), or infectious (e.g., aortitis [Chlamydia pneumoniae, human immunodeficiency virus]). AAAs are often asymptomatic. They are often found during routine physical examinations or evaluations for an unrelated problem (e.g., abdominal x-ray, CT scan). A pulsatile mass in the periumbilical area slightly to the left of the midline may be present. Bruits may be auscultated over the aneurysm. Physical findings may be hard to detect in obese persons.

Which person should the nurse identify as having the highest risk for abdominal aortic aneurysm? A) A 70-year-old male, with high cholesterol and hypertension B) A 40-year-old female with obesity and metabolic syndrome C) A 60-year-old male with renal insufficiency who is physically inactive D) A 65-year-old female with hyperhomocysteinemia and substance abuse

A) A 70-year-old male, with high cholesterol and hypertension The most common etiology of descending abdominal aortic aneurysm (AAA) is atherosclerosis. Male gender, age 65 years or older, and tobacco use are the major risk factors for AAAs of atherosclerotic origin. Other risk factors include the presence of coronary or peripheral artery disease, high blood pressure, and high cholesterol.

The patient has CVI and a venous ulcer. The unlicensed assistive personnel (UAP) decides to apply compression stockings because that is what these patients always have ordered. What assessment by the nurse would cause the application of compression stockings to harm the patient? A) Rest pain B) High blood pressure C) Elevated blood sugar D) Dry, itchy, flaky skin

A) Rest pain Rest pain occurs as peripheral artery disease (PAD) progresses and involves multiple arterial segments. Compression stockings should not be used on patients with PAD. Elevated blood glucose, possibly indicating uncontrolled diabetes mellitus, and hypertension may or may not indicate arterial problems. Dry, itchy, flaky skin indicates venous insufficiency. The RN should be the one to obtain the order and instruct the UAP to apply compression stockings if they are ordered.

The pt with cirrhosis has increased ab girth from ascites. The nurse should know that this fluid gathers in the ad for which reasons? Select all that apply. a. there is decreased colloid oncotic pressure from the liver's inability to synthesize albumin. b. hyperaldosteronsim related to damaged hepatocytes increases sodium and fluid retention c. portal hypertension pushes proteins from the blood vessels, causing leaking into the peritoneal cavity d. osmoreceptors in the hypothalamus stimulate thirst, which causes the stimulation to take in fluid orally e. overactivity of the enlarged spleen results in increased removal of blood cells from the circulation, which decreases the vascular pressure.

A, B, C Ascites related to cirrhosis is caused by decreased colloid oncotic pressure from the lack of albumin form liver inability to synthesize it and the portal hypertension shifts protein into peritoneal cavity, and hyperaldosteronism which increases sodium and fluid retention.

When caring for a pt with liver disease, the nurse recognizes the need to prevent bleeding resulting from altered clotting factors and rupture of varices. Which nursing interventions would be appropriate to achieve this outcome? Select all that apply. a. use smallest gauge needle possible when giving injections or drawing blood. b. teach pt to avoid straining at stool, vigorous blowing of nose, and coughing c. advise pt to use soft-bristle toothbrush and avoid ingestion of irritating food. d. apply gentle pressure for the shortest possible time period after performing venipuncture e. instruct pt to avoid aspirin and NSAIDs to prevent hemorrhage when varices are present.

A, B, C, E - small gauge minimize risk of bleeding into tissues. - avoiding strain reduces hemorrhage - soft bristle reduce injury to highly vascular mucous membranes - apply gentle but prolonged pressure to venipuncture - aspirin and NSAIDs should not be used in pt with liver disease b/c they interfere w/ platelet aggregation, increasing bleeding risk

The patient has a homocysteine level ordered. What aspects of this test should inform the nurse's care? Select all that apply. A) A 12-hour fast is necessary before drawing the blood sample. B) Recent inactivity can depress homocysteine levels. C) Genetic factors can elevate homocysteine levels. D) A diet low in folic acid elevates homocysteine levels. E) An ECG should be performed immediately before drawing a sample.

A, C, D Feedback:Genetic factors and a diet low in folic acid, vitamin B6, and vitamin B12are associatedwith elevated homocysteine levels. A 12-hour fast is necessary before drawing a bloodsample for an accurate serum measurement. An ECG is unnecessary and recent inactivitydoes not influence the results of the test

A patient is being started on nicotinic acid (Niaspan) to reduce triglyceride levels. The nurse is providing patient education and should include which adverse effects? (SATA) a. Facial flushing b. Constipation c. Hypoglycemia d. Gastric upset e. Itching

A, D, E ~ Adverse effects of nicotinic acid include intense flushing of the face, neck, and ears; itching; and GI upset (nausea, vomiting, and diarrhea). Constipation and hypoglycemia are not adverse effects of niacin therapy.

The nurse is developing a discharge teaching plan for a patient diagnosed with thromboangiitis obliterans (Buerger's disease). Which expected outcome has the highest priority for this patient? a. Cessation of all tobacco use b. Control of serum lipid levels c. Maintenance of appropriate weight d. Demonstration of meticulous foot care

ANS: A Absolute cessation of nicotine use is needed to reduce the risk for amputation in patients with Buerger's disease. Other therapies have limited success in treatment of this disease

An older patient with a history of an abdominal aortic aneurysm arrives at the emergency department (ED) with severe back pain and absent pedal pulses. Which action should the nurse take first? a. Check the blood pressure. b. Draw blood for laboratory testing. c. Assess for the presence of an abdominal bruit. d. Determine any family history of heart disease.

ANS: A Because the patient appears to be experiencing aortic dissection, the nurse's first action should be to determine the hemodynamic status by assessing blood pressure. The other actions may also be done, but they will not provide information to determine what interventions are needed immediately

When evaluating the discharge teaching for a patient with chronic peripheral artery disease (PAD), the nurse determines a need for further instruction when the patient says, "I will a. use a heating pad on my feet at night to increase the circulation." b. buy some loose clothes that do not bind across my legs or waist." c. walk to the point of pain, rest, and walk again for at least 30 minutes 3 times a week." d. change my position every hour and avoid long periods of sitting with my legs crossed."

ANS: A Because the patient has impaired circulation and sensation to the feet, the use of a heating pad could lead to burns. The other patient statements are correct and indicate that teaching has been successfu

A patient in the outpatient clinic is diagnosed with acute hepatitis C (HCV) infection. Which action by the nurse is appropriate? a.Schedule the patient for HCV genotype testing. b.Administer the HCV vaccine and immune globulin. c.Teach the patient about ribavirin (Rebetol) treatment. d.Explain that the infection will resolve over a few months

ANS: A Genotyping of HCV has an important role in managing treatment and is done before drug therapy is initiated. Because most patients with acute HCV infection convert to the chronic state, the nurse should not teach the patient that the HCV will resolve in a few months. Immune globulin or vaccine is not available for HCV. Ribavirin is used for chronic HCV infection

The nurse is caring for a patient immediately after repair of an abdominal aortic aneurysm. On assessment, the patient has absent popliteal, posterior tibial, and dorsalis pedis pulses. The legs are cool and mottled. Which action should the nurse take first? a. Notify the surgeon and anesthesiologist. b. Wrap both the legs in a warming blanket. c. Document the findings and recheck in 15 minutes. d. Compare findings to the preoperative assessment of the pulses.

ANS: A Lower extremity pulses may be absent for a short time after surgery because of vasospasm and hypothermia. Decreased or absent pulses together with a cool and mottled extremity may indicate embolization or graft occlusion. These findings should be reported to the surgeon immediately because this is an emergency situation. Because pulses are marked before surgery, the nurse would know whether pulses were present before surgery before notifying the health care providers about the absent pulses. Because the patient's symptoms may indicate graft occlusion or multiple emboli and a possible need to return to surgery, it is not appropriate to wait 15 minutes before taking action. A warming blanket will not improve the circulation to the patient's legs.

After teaching a patient with newly diagnosed Raynaud's phenomenon about how to manage the condition, which action by the patient best demonstrates that the teaching has been effective? a. The patient exercises indoors during the winter months. b. The patient immerses hands in hot water when they turn pale. c. The patient takes pseudoephedrine (Sudafed) for cold symptoms. d. The patient avoids taking nonsteroidal antiinflammatory drugs (NSAIDs)

ANS: A Patients should avoid temperature extremes by exercising indoors when it is cold. To avoid burn injuries, the patient should use warm rather than hot water to warm the hands. Pseudoephedrine is a vasoconstrictor and should be avoided. There is no reason to avoid taking NSAIDs with Raynaud's phenomenon

A patient in the outpatient clinic has a new diagnosis of peripheral artery disease (PAD). Which group of drugs will the nurse plan to include when teaching about PAD management? a. Statins b. Antibiotics c. Thrombolytics d. Anticoagulants

ANS: A Research indicates that statin use by patients with PAD improves multiple outcomes. There is no research that supports the use of the other drug categories in PAD

A patient who is 2 days post femoral popliteal bypass graft to the right leg is being cared for on the vascular unit. Which action by a licensed practical/vocational nurse (LPN/LVN) caring for the patient requires the registered nurse (RN) to intervene? a. The LPN/LVN has the patient to sit in a chair for 2 hours. b. The LPN/LVN gives the prescribed aspirin after breakfast. c. The LPN/LVN assists the patient to walk 40 feet in the hallway. d. The LPN/LVN places the patient in Fowler's position for meals

ANS: A The patient should avoid sitting for long periods because of the increased stress on the suture line caused by leg edema and because of the risk for venous thromboembolism (VTE). The other actions by the LPN/LVN are appropriate

A patient has been admitted with acute liver failure. Which assessment data are most important for the nurse to communicate to the health care provider? a. Asterixis and lethargy b. Jaundiced sclera and skin c. Elevated total bilirubin level d. Liver 3 cm below costal margin

ANS: A The patient's findings of asterixis and lethargy are consistent with grade 2 hepatic encephalopathy. Patients with acute liver failure can deteriorate rapidly from grade 1 or 2 to grade 3 or 4 hepatic encephalopathy and need early transfer to a transplant center. The other findings are typical of patients with hepatic failure and would be reported but would not indicate a need for an immediate change in the therapeutic plan

A patient has been admitted with acute liver failure. Which assessment data are most important for the nurse to communicate to the health care provider? a. Asterixis and lethargy b. Jaundiced sclera and skin c. Elevated total bilirubin level d. Liver 3 cm below costal margin

ANS: A The patient's findings of asterixis and lethargy are consistent with grade 2 hepatic encephalopathy. Patients with acute liver failure can deteriorate rapidly from grade 1 or 2 to grade 3 or 4 hepatic encephalopathy and need early transfer to a transplant center. The other findings are typical of patients with hepatic failure and would be reported but would not indicate a need for an immediate change in the therapeutic plan. DIF: Cognitive Level: Analyze (analysis) REF: 990 OBJ: Special Questions: Prioritization TOP: Nursing Process: Assessment MSC: NCLEX: Physiological Integrity

Which finding indicates to the nurse that lactulose is effective for an older adult who has advanced cirrhosis? a. The patient is alert and oriented. b. The patient denies nausea or anorexia. c. The patient's bilirubin level decreases. d. The patient has at least one stool daily.

ANS: A The purpose of lactulose in the patient with cirrhosis is to lower ammonia levels and prevent encephalopathy. Although lactulose may be used to treat constipation, that is not the purpose for this patient. Lactulose will not decrease nausea and vomiting or lower bilirubin levels

Which finding indicates to the nurse that lactulose is effective for an older adult who has advanced cirrhosis? a. The patient is alert and oriented. b. The patient denies nausea or anorexia. c. The patient's bilirubin level decreases. d. The patient has at least one stool daily.

ANS: A The purpose of lactulose in the patient with cirrhosis is to lower ammonia levels and prevent encephalopathy. Although lactulose may be used to treat constipation, that is not the purpose for this patient. Lactulose will not decrease nausea and vomiting or lower bilirubin levels. DIF: Cognitive Level: Apply (application) REF: 992 TOP: Nursing Process: Evaluation MSC: NCLEX: Physiological Integrity

The nurse administering -interferon and ribavirin (Rebetol) to a patient with chronic hepatitis C will plan to monitor for: a.leukopenia. b.hypokalemia. c.polycythemia. d.hypoglycemia.

ANS: A Therapy with ribavirin and -interferon may cause leukopenia. The other problems are not associated with this drug therapy

The nurse administering a-interferon and ribavirin (Rebetol) to a patient with chronic hepatitis C will plan to monitor for a. leukopenia. b. hypokalemia. c. polycythemia. d. hypoglycemia.

ANS: A Therapy with ribavirin and -interferon may cause leukopenia. The other problems are notassociated with this drug therapy.DIF: Cognitive Level: Apply (application) REF: 981TOP: Nursing Process: Planning MSC: NCLEX: Physiological Integrity

After an unimmunized person is exposed to hepatitis B through a needle-stick injury, which actions will the nurse plan to take? (Select all that apply.) a. Administer hepatitis B vaccine. b. Test for antibodies to hepatitis B. c. Teach about alpha-interferon therapy. d. Give hepatitis B immune globulin. e. Explain options for oral antiviral therapy.

ANS: A, B, D The recommendations for hepatitis B exposure include both vaccination and immune globulin administration. In addition, baseline testing for hepatitis B antibodies will be needed. Interferon and oral antivirals are not used for hepatitis B prophylaxis.

After an unimmunized individual is exposed to hepatitis B through a needle-stick injury, which actions will the nurse plan to take (select all that apply)? a. Administer hepatitis B vaccine. b. Test for antibodies to hepatitis B. c. Teach about a-interferon therapy. d. Give hepatitis B immune globulin. e. Teach about choices for oral antiviral therapy.

ANS: A, B, D The recommendations for hepatitis B exposure include both vaccination and immune globulin administration. In addition, baseline testing for hepatitis B antibodies will be needed. Interferon and oral antivirals are not used for hepatitis B prophylaxis. DIF: Cognitive Level: Apply (application) REF: 983 TOP: Nursing Process: Planning MSC: NCLEX: Safe and Effective Care Environment

Which information given by a 70-yr-old patient during a health history indicates to the nurse that the patient should be screened for hepatitis C? a. The patient had a blood transfusion in 2005. b. The patient used IV drugs about 20 years ago. c. The patient frequently eats in fast-food restaurants. d. The patient traveled to a country with poor sanitation.

ANS: B Any patient with a history of IV drug use should be tested for hepatitis C. Blood transfusionsgiven after 1992 (when an antibody test for hepatitis C became available) do not pose a riskfor hepatitis C. Hepatitis C is not spread by the oral-fecal route and therefore is not caused bycontaminated food or by traveling in underdeveloped countries.DIF: Cognitive Level: Apply (application) REF: 976TOP: Nursing Process: Assessment MSC: NCLEX: Health Promotion and Maintenance

The nurse is caring for a patient who has cirrhosis. Which data obtained by the nurse during the assessment will be of most concern? a. The patient complains of right upper-quadrant pain with palpation. b. The patient's hands flap back and forth when the arms are extended. c. The patient has ascites and a 2-kg weight gain from the previous day. d. The patient's abdominal skin has multiple spider-shaped blood vessels.

ANS: B Asterixis indicates that the patient has hepatic encephalopathy, and hepatic coma may occur.The spider angiomas and right upper quadrant abdominal pain are not unusual for the patientwith cirrhosis and do not require a change in treatment. The ascites and weight gain indicatethe need for treatment but not as urgently as the changes in neurologic status.DIF: Cognitive Level: Analyze (analysis) REF: 990OBJ: Special Questions: Prioritization TOP: Nursing Process: AssessmentMSC: NCLEX: Physiological Integrity

A patient admitted with an abrupt onset of jaundice and nausea has abnormal liver function studies but serologic testing is negative for viral causes of hepatitis. Which question by the nurse is appropriate? a. "Do you have a history of IV drug use?" b. "Do you use any over-the-counter drugs?" c. "Have you used corticosteroids for any reason?" d. "Have you recently traveled to a foreign country?"

ANS: B The patient's symptoms, lack of antibodies for hepatitis, and the abrupt onset of symptomssuggest toxic hepatitis, which can be caused by commonly used over-the-counter drugs suchas acetaminophen (Tylenol). Travel to a foreign country and a history of IV drug use are riskfactors for viral hepatitis. Corticosteroid use does not cause the symptoms listed.DIF: Cognitive Level: Apply (application) REF: 984TOP: Nursing Process: Assessment MSC: NCLEX: Physiological Integrity

Which information from a 70-yr-old patient during a health history indicates to the nurse that the patient should be screened for hepatitis C? a.The patient had a blood transfusion in 2005. b.The patient used IV drugs about 20 years ago. c.The patient frequently eats in fast-food restaurants. d.The patient traveled to a country with poor sanitation.

ANS: B Any patient with a history of IV drug use should be tested for hepatitis C. Blood transfusions given after 1992 (when an antibody test for hepatitis C became available) do not pose a risk for hepatitis C. Hepatitis C is not spread by the oral-fecal route and therefore is not caused by contaminated food or by traveling in underdeveloped countries

The nurse is caring for a patient who has cirrhosis. Which data obtained by the nurse during the assessment will be of most concern? a.The patient reports right upper-quadrant pain with palpation. b.The patient's hands flap back and forth when the arms are extended. c.The patient has ascites and a 2-kg weight gain from the previous day. d.The patient's abdominal skin has multiple spider-shaped blood vessels.

ANS: B Asterixis indicates that the patient has hepatic encephalopathy, and hepatic coma may occur. The spider angiomas and right upper quadrant abdominal pain are not unusual for the patient with cirrhosis and do not require a change in treatment. The ascites and weight gain indicate the need for treatment but not as urgently as the changes in neurologic status.

Which nursing action should be included in the plan of care after endovascular repair of an abdominal aortic aneurysm? a. Record hourly chest tube drainage. b. Monitor fluid intake and urine output. c. Assess the abdominal incision for redness. d. Teach the patient to plan for a long recovery period.

ANS: B Because renal artery occlusion can occur after endovascular repair, the nurse should monitor parameters of renal function such as intake and output. Chest tubes will not be needed for endovascular surgery, the recovery period will be short, and there will not be an abdominal wound

The nurse is admitting a patient newly diagnosed with peripheral artery disease. Which admission order should the nurse question? a. Cilostazol drug therapy b. Omeprazole drug therapy c. Use of treadmill for exercise d. Exercise to the point of discomfort

ANS: B Because the antiplatelet effect of clopidogrel is reduced when it is used with omeprazole, the nurse should clarify this order with the health care provider. The other interventions are appropriate for a patient with peripheral artery disease

While working in the outpatient clinic, the nurse notes that a patient has a history of intermittent claudication. Which statement by the patient would support this information? a. "When I stand too long, my feet start to swell." b. "My legs cramp when I walk more than a block." c. "I get short of breath when I climb a lot of stairs." d. "My fingers hurt when I go outside in cold weather."

ANS: B Cramping that is precipitated by a consistent level of exercise is descriptive of intermittent claudication. Finger pain associated with cold weather is typical of Raynaud's phenomenon. Shortness of breath that occurs with exercise is not typical of intermittent claudication, which is reproducible. Swelling associated with prolonged standing is typical of venous disease

A patient has a 6-cm thoracic aortic aneurysm that was discovered during routine chest x-ray. When obtaining an admission history from the patient, it will be most important for the nurse to ask about a. low back pain. b. trouble swallowing. c. abdominal tenderness. d. changes in bowel habits

ANS: B Difficulty swallowing may occur with a thoracic aneurysm because of pressure on the esophagus. The other symptoms will be important to assess for in patients with abdominal aortic aneurysms

Which action should the nurse take to evaluate treatment effectiveness for a patient who has hepatic encephalopathy? a. Request that the patient stand on one foot. b. Ask the patient to extend both arms forward. c. Request that the patient walk with eyes closed. d. Ask the patient to perform the Valsalva maneuver.

ANS: B Extending the arms allows the nurse to check for asterixis, a classic sign of hepatic encephalopathy. The other tests might also be done as part of the neurologic assessment but would not be diagnostic for hepatic encephalopathy. DIF: Cognitive Level: Apply (application) REF: 990 TOP: Nursing Process: Assessment MSC: NCLEX: Physiological Integrity

Which action should the nurse take to evaluate treatment effectiveness for a patient who has hepatic encephalopathy? a. Request that the patient stand on one foot. b. Ask the patient to extend both arms forward. c. Request that the patient walk with eyes closed. d. Ask the patient to perform the Valsalva maneuver.

ANS: B Extending the arms allows the nurse to check for asterixis, a classic sign of hepatic encephalopathy. The other tests might be done as part of the neurologic assessment but would not be diagnostic for hepatic encephalopathy

The health care provider prescribes an infusion of heparin and daily partial thromboplastin time (PTT) testing for a patient with venous thromboembolism (VTE). The nurse will plan to a. decrease the infusion when the PTT value is 65 seconds. b. avoid giving IM medications to prevent localized bleeding. c. have vitamin K available in case reversal of the heparin is needed. d. monitor posterior tibial and dorsalis pedis pulses with the Doppler.

ANS: B Intramuscular injections are avoided in patients receiving anticoagulation to prevent hematoma formation and bleeding from the site. A PTT of 65 seconds is within the therapeutic range. Vitamin K is used to reverse warfarin. Pulse quality is not affected by VTE.

The nurse who works in the vascular clinic has several patients with venous insufficiency scheduled today. Which patient should the nurse assign to an experienced licensed practical/vocational nurse (LPN/LVN)? a. Patient who has been complaining of increased edema and skin changes in the legs b. Patient who needs wound care for a chronic venous stasis ulcer on the right lower leg c. Patient who has a history of venous thromboembolism and is complaining of dyspnea d. Patient who needs teaching about elastic compression stockings for venous insufficiency

ANS: B LPN education and scope of practice includes wound care. The other patients, which require more complex assessments or education, should be managed by the RN

A patient with a venous thromboembolism (VTE) is started on enoxaparin (Lovenox) and warfarin (Coumadin). The patient asks the nurse why two medications are necessary. Which response by the nurse is most accurate? a. "Taking two blood thinners greatly reduces the risk for another clot to form." b. "Enoxaparin will work right away, but warfarin takes several days to begin preventing clots." c. "Enoxaparin will start to dissolve the clot, and warfarin will prevent any more clots from forming." d. "Because of the risk for a blood clot in the lungs, it is important for you to take more than one blood thinner."

ANS: B Low molecular weight heparin (LMWH) is used because of the immediate effect on coagulation and discontinued once the international normalized ratio (INR) value indicates that the warfarin has reached a therapeutic level. LMWH has no thrombolytic properties. The use of two anticoagulants is not related to the risk for pulmonary embolism, and two are not necessary to reduce the risk for another VTE. Anticoagulants do not thin the blood

What topic should the nurse plan to teach the patient diagnosed with acute hepatitis B? a.Administering alpha -interferon b.Measures for improving appetite c.Side effects of nucleotide analogs d.Ways to increase activity and exercise

ANS: B Maintaining adequate nutritional intake is important for regeneration of hepatocytes. Interferon and antivirals may be used for chronic hepatitis B, but they are not prescribed foracute hepatitis B infection. Rest is recommended.

The nurse has started discharge teaching for a patient who is to continue warfarin (Coumadin) after hospitalization for venous thromboembolism (VTE). The nurse determines that additional teaching is needed when the patient says which of the following? a. "I should get a Medic Alert device stating that I take warfarin." b. "I should reduce the amount of green, leafy vegetables that I eat." c. "I will need routine blood tests to monitor the effects of the warfarin." d. "I will check with my health care provider before I begin any new drugs."

ANS: B Patients taking warfarin are taught to follow a consistent diet with regard to foods that are high in vitamin K, such as green, leafy vegetables. The other patient statements are accurate.

For a patient who has cirrhosis, which nursing action can the registered nurse (RN) delegate to unlicensed assistive personnel (UAP)? a. Assessing the patient for jaundice b. Providing oral hygiene after a meal c. Palpating the abdomen for distention d. Teaching the patient the prescribed diet

ANS: B Providing oral hygiene is within the scope of UAP. Assessments and assisting patients to choose therapeutic diets are nursing actions that require higher level nursing education and scope of practice and would be delegated to licensed practical/vocational nurses (LPNs/VNs) or RNs

Which instructions should the nurse include in a teaching plan for an older patient newly diagnosed with peripheral artery disease (PAD)? a. "Exercise only if you do not experience any pain." b. "It is very important that you stop smoking cigarettes." c. "Try to keep your legs elevated whenever you are sitting." d. "Put elastic compression stockings on early in the morning."

ANS: B Smoking cessation is essential for slowing the progression of PAD to critical limb ischemia and reducing the risk of myocardial infarction and death. Circulation to the legs will decrease if the legs are elevated. Patients with PAD are taught to exercise to the point of feeling pain, rest, and then resume walking. Support hose are not used for patients with PAD.

Which topic is most important to include in teaching for a 41-yr-old patient diagnosed with early alcoholic cirrhosis? a. Taking lactulose b. Avoiding all alcohol use c. Maintaining good nutrition d. Using vitamin B supplements

ANS: B The disease progression can be stopped or reversed by alcohol abstinence. The other interventions may be used when cirrhosis becomes more severe to decrease symptoms or complications, but the priority for this patient is to stop the progression of the disease

Which goal has the highest priority in the plan of care for a 26-yr-old patient who was admitted with viral hepatitis, has severe anorexia and fatigue, and is homeless? a.Increase activity level. b.Maintain adequate nutrition. c.Establish a stable environment. d.Identify source of hepatitis exposure

ANS: B The highest priority outcome is to maintain nutrition because adequate nutrition is needed for hepatocyte regeneration. Finding a home for the patient and identifying the source of the infection would be appropriate activities, but they do not have as high a priority as ensuring adequate nutrition. Although the patient's activity level will be gradually increased, rest is indicated during the acute phase of hepatitis

A patient is being treated for bleeding esophageal varices with balloon tamponade. Which nursing action will be included in the plan of care? a. Instruct the patient to cough every hour. b. Monitor the patient for shortness of breath. c. Verify the position of the balloon every 4 hours. d. Deflate the gastric balloon if the patient reports nausea.

ANS: B The most common complication of balloon tamponade is aspiration pneumonia. In addition, if the gastric balloon ruptures, the esophageal balloon may slip upward and occlude the airway. Coughing increases the pressure on the varices and increases the risk for bleeding. Balloon position is verified after insertion and does not require further verification. Balloons may be deflated briefly every 8 to 12 hours to avoid tissue necrosis, but if only the gastric balloon is deflated, the esophageal balloon may occlude the airway. Balloons are not deflated for nausea

A patient is being treated for bleeding esophageal varices with balloon tamponade. Which nursing action will be included in the plan of care? a. Instruct the patient to cough every hour. b. Monitor the patient for shortness of breath. c. Verify the position of the balloon every 4 hours. d. Deflate the gastric balloon if the patient reports nausea.

ANS: B The most common complication of balloon tamponade is aspiration pneumonia. In addition, if the gastric balloon ruptures, the esophageal balloon may slip upward and occlude the airway. Coughing increases the pressure on the varices and increases the risk for bleeding. Balloon position is verified after insertion and does not require further verification. Balloons may be deflated briefly every 8 to 12 hours to avoid tissue necrosis, but if only the gastric balloon is deflated, the esophageal balloon may occlude the airway. Balloons are not deflated for nausea. DIF: Cognitive Level: Apply (application) REF: 992 TOP: Nursing Process: Implementation MSC: NCLEX: Physiological Integrity

After receiving change of shift report, which patient admitted to the emergency department should the nurse assess first? a. A 67-yr-old patient who has a gangrenous left foot ulcer with a weak pedal pulse b. A 50-yr-old patient who is complaining of sudden sharp and severe upper back pain c. A 39-yr-old patient who has right calf tenderness, redness, and swelling after a plane ride d. A 58-yr-old patient who is taking anticoagulants for atrial fibrillation and has black stools

ANS: B The patient's presentation of sudden sharp and severe upper back pain is consistent with dissecting thoracic aneurysm, which will require the most rapid intervention. The other patients also require rapid intervention but not before the patient with severe pain.

What is most important for the nurse to monitor to detect possible complications in a patient with severe cirrhosis who has bleeding esophageal varices? a. Bilirubin levels b. Ammonia levels c. Potassium levels d. Prothrombin time

ANS: B The protein in the blood in the gastrointestinal tract will be absorbed and may result in an increase in the ammonia level because the liver cannot metabolize protein very well. The prothrombin time, bilirubin, and potassium levels should also be monitored, but they will not be affected by the bleeding episode

When caring for a patient on the first postoperative day after an abdominal aortic aneurysm repair, which assessment finding is most important for the nurse to communicate to the health care provider? a. Presence of flatus b. Hypoactive bowel sounds c. Maroon-colored liquid stool d. Abdominal pain with palpation

ANS: C Loose, bloody (maroon colored) stools at this time may indicate intestinal ischemia or infarction and should be reported immediately because the patient may need an emergency bowel resection. The other findings are normal on the first postoperative day after abdominal surgery.

Which assessment finding would the nurse need to report most quickly to the health care provider regarding a patient who has acute pancreatitis? a. Nausea and vomiting b. Hypotonic bowel sounds c. Muscle twitching and finger numbness d. Upper abdominal tenderness and guarding

ANS: C Muscle twitching and finger numbness indicate hypocalcemia, which may lead to tetany unless calcium gluconate is administered. Although the other findings should also be reported to the health care provider, they do not indicate complications that require rapid action

A patient with chronic hepatitis C infection has several medications prescribed. Which medication requires further discussion with the health care provider before administration? a. Ribavirin (Rebetol, Copegus) 600 mg PO bid b. Diphenhydramine 25 mg PO every 4 hours PRN itching c. Pegylated a-interferon (PEG-Intron, Pegasys) 1.5 mcg/kg PO daily d. Dimenhydrinate (Dramamine) 50 mg PO every 6 hours PRN nausea

ANS: C Pegylated a-interferon is administered subcutaneously, not orally. The medications are allappropriate for a patient with chronic hepatitis C infection.DIF: Cognitive Level: Understand (comprehension) REF: 981TOP: Nursing Process: Implementation MSC: NCLEX: Physiological Integrity

A 36-yr-old female patient is receiving treatment for chronic hepatitis B with pegylated interferon (Pegasys). Which finding is important to communicate to the health care provider to suggest a change in therapy? a.Nausea and anorexia b.Weight loss of 2 lb (1 kg) c.Positive urine pregnancy test d.Hemoglobin level of 10.4 g/dL

ANS: C Because ribavirin is teratogenic, the medication will need to be discontinued immediately. Anemia, weight loss, and nausea are common adverse effects of the prescribed regimen and may require actions such as patient teaching, but they would not require immediate cessation of the therapy

A patient born in 1955 had hepatitis A infection 1 year ago. According to Centers for Disease Control and Prevention (CDC) guidelines, which action should the nurse include in care when the patient is seen for a routine annual physical examination? a. Start the hepatitis B immunization series. b. Teach the patient about hepatitis A immune globulin. c. Ask whether the patient has been screened for hepatitis C. d. Test for anti-hepatitis-A virus immune globulin M (anti-HAV-IgM).

ANS: C Current CDC guidelines indicate that all patients who were born between 1945 and 1965 should be screened for hepatitis C because many individuals who are positive have not been diagnosed. Although routine hepatitis B immunization is recommended for infants, children, and adolescents, vaccination for hepatitis B is recommended only for adults at risk for blood-borne infections. Because the patient has already had hepatitis A, immunization and anti-HAV IgM levels will not be needed. DIF: Cognitive Level: Apply (application) REF: 983 TOP: Nursing Process: Planning MSC: NCLEX: Health Promotion and Maintenance

A patient born in 1955 had hepatitis A infection 1 year ago. According to Centers for Disease Control and Prevention (CDC) guidelines, which action should the nurse include in care when the patient is seen for a routine annual physical examination? a.Start the hepatitis B immunization series. b.Teach the patient about hepatitis A immune globulin. c.Ask whether the patient has been screened for hepatitis C. d. Test for anti-hepatitis-A virus immune globulin M (anti-HAV-IgM)

ANS: C Current CDC guidelines indicate that all patients who were born between 1945 and 1965 should be screened for hepatitis C because many persons who are positive have not been diagnosed. Although routine hepatitis B immunization is recommended for infants, children, and adolescents, vaccination for hepatitis B is recommended only for adults at risk for blood-borne infections. Because the patient has already had hepatitis A, immunization, and anti-HAV IgM levels will not be needed.

Which response by the nurse best explains the purpose of ranitidine (Zantac) for a patient who was admitted with bleeding esophageal varices? a. The medication will reduce the risk for aspiration. b. The medication will inhibit development of gastric ulcers. c. The medication will prevent irritation of the enlarged veins. d. The medication will decrease nausea and improve the appetite.

ANS: C Esophageal varices are dilated submucosal veins. The therapeutic action of H2-receptor blockers in patients with esophageal varices is to prevent irritation and bleeding from the varices caused by reflux of acid gastric contents. Although ranitidine does decrease the risk for peptic ulcers, reduce nausea, and help prevent aspiration pneumonia, these are not the primary purposes for H2-receptor blockade in this patient

Which response by the nurse best explains the purpose of ranitidine (Zantac) for a patient admitted with bleeding esophageal varices? a. The medication will reduce the risk for aspiration. b. The medication will inhibit development of gastric ulcers. c. The medication will prevent irritation of the enlarged veins. d. The medication will decrease nausea and improve the appetite.

ANS: C Esophageal varices are dilated submucosal veins. The therapeutic action of H2-receptor blockers in patients with esophageal varices is to prevent irritation and bleeding from the varices caused by reflux of acid gastric contents. Although ranitidine does decrease the risk for peptic ulcers, reduce nausea, and help prevent aspiration pneumonia, these are not the primary purposes for H2-receptor blockade in this patient. DIF: Cognitive Level: Analyze (analysis) REF: 1004 TOP: Nursing Process: Implementation MSC: NCLEX: Physiological Integrity

The nurse will plan to teach the patient diagnosed with acute hepatitis B about a. administering a-interferon b. side effects of nucleotide analogs. c. measures for improving the appetite. d. ways to increase activity and exercise.

ANS: C Maintaining adequate nutritional intake is important for regeneration of hepatocytes.Interferon and antivirals may be used for chronic hepatitis B, but they are not prescribed foracute hepatitis B infection. Rest is recommended.DIF: Cognitive Level: Apply (application) REF: 980TOP: Nursing Process: Planning MSC: NCLEX: Physiological Integrity

A young adult patient tells the health care provider about experiencing cold, numb fingers when running during the winter, and Raynaud's phenomenon is suspected. The nurse will anticipate teaching the patient about tests for a. hyperglycemia. b. hyperlipidemia. c. autoimmune disorders. d. coronary artery disease.

ANS: C Secondary Raynaud's phenomenon may occur in conjunction with autoimmune diseases such as rheumatoid arthritis. Patients should be screened for autoimmune disorders. Raynaud's phenomenon is not associated with hyperlipidemia, hyperglycemia, or coronary artery disease

The nurse is caring for a patient with a descending aortic dissection. Which assessment finding is most important to report to the health care provider? a. Weak pedal pulses b. Absent bowel sounds c. Blood pressure of 138/88 mm Hg d. 25 mL of urine output over the past hour

ANS: C The blood pressure is typically kept at less than 120 mm Hg systolic to minimize extension of the dissection. The nurse will need to notify the health care provider so that b-blockers or other antihypertensive drugs can be prescribed. The other findings are typical with aortic dissection and should also be reported but do not require immediate action

Which topic is most important to include in patient teaching for a 41-yr-old patient diagnosed with early alcoholic cirrhosis? a. Taking lactulose b. Maintaining good nutrition c. Avoiding alcohol ingestion d. Using vitamin B supplements

ANS: C The disease progression can be stopped or reversed by alcohol abstinence. The other interventions may be used when cirrhosis becomes more severe to decrease symptoms or complications, but the priority for this patient is to stop the progression of the disease. DIF: Cognitive Level: Analyze (analysis) REF: 995 TOP: Nursing Process: Planning MSC: NCLEX: Physiological Integrity

Which action should the nurse in the emergency department take first for a new patient who is vomiting blood? a. Insert a large-gauge IV catheter. b. Draw blood for coagulation studies. c. Check blood pressure and heart rate. d. Place the patient in the supine position.

ANS: C The nurse's first action should be to determine the patient's hemodynamic status by assessing vital signs. Drawing blood for coagulation studies and inserting an IV catheter are also appropriate. However, the vital signs may indicate the need for more urgent actions. Because aspiration is a concern for this patient, the nurse will need to assess the patient's vital signs and neurologic status before placing the patient in a supine position. DIF: Cognitive Level: Analyze (analysis) REF: 1002 OBJ: Special Questions: Prioritization TOP: Nursing Process: Implementation MSC: NCLEX: Physiological Integrity

How should the nurse prepare a patient with ascites for paracentesis? a. Place the patient on NPO status. b. Assist the patient to lie flat in bed. c. Ask the patient to empty the bladder. d. Position the patient on the right side.

ANS: C The patient should empty the bladder to decrease the risk of bladder perforation during the procedure. The patient would be positioned in Fowler's position and would not be able to lie flat without compromising breathing. Because no sedation is required for paracentesis, the patient does not need to be NPO

A patient admitted with an abrupt onset of jaundice and nausea has abnormal liver function studies, but serologic testing is negative for viral causes of hepatitis. Which question by the nurse is appropriate? a."Have you taken corticosteroids?" b."Do you have a history of IV drug use?" c."Do you use any over-the-counter drugs?" d."Have you recently traveled to another country?"

ANS: C The patient's symptoms, lack of antibodies for hepatitis, and the abrupt onset of symptoms suggest toxic hepatitis, which can be caused by commonly used over-the-counter drugs such as acetaminophen (Tylenol). Travel to a foreign country and a history of IV drug use are risk factors for viral hepatitis. Corticosteroid use does not cause the symptoms listed

Which finding is most important for the nurse to communicate to the health care provider about a patient who received a liver transplant 1 week ago? a. Dry palpebral and oral mucosa b. Crackles at bilateral lung bases c. Temperature 100.8° F (38.2° C) d. No bowel movement for 4 days

ANS: C The risk of infection is high in the first few months after liver transplant, and fever is frequently the only sign of infection. The other patient data indicate the need for further assessment or nursing actions and might be communicated to the health care provider, but they do not indicate a need for urgent action.

Which finding is most important for the nurse to communicate to the health care provider about a patient who received a liver transplant 1 week ago? a. Dry palpebral and oral mucosa b. Crackles at bilateral lung bases c. Temperature 100.8° F (38.2° C) d. No bowel movement for 4 days

ANS: C The risk of infection is high in the first few months after liver transplant, and fever is frequently the only sign of infection. The other patient data indicate the need for further assessment or nursing actions and might be communicated to the health care provider, but they do not indicate a need for urgent action. DIF: Cognitive Level: Analyze (analysis) REF: 998 OBJ: Special Questions: Prioritization TOP: Nursing Process: Implementation MSC: NCLEX: Physiological Integrity

During change-of-shift report, the nurse learns about the following four patients. Which patient requires assessment first? a. A 58-yr-old patient who has compensated cirrhosis and reports anorexia b. A 40-yr-old patient with chronic pancreatitis who has gnawing abdominal pain c. A 55-yr-old patient with cirrhosis and ascites who has an oral temperature of 102° F (38.8° C) d. A 36-yr-old patient recovering from a laparoscopic cholecystectomy who has severe shoulder pain

ANS: C This patient's history and fever suggest possible spontaneous bacterial peritonitis, which would require rapid assessment and interventions such as antibiotic therapy. The clinical manifestations for the other patients are consistent with their diagnoses and do not indicate complications are occurring.

During change-of-shift report, the nurse learns about the following four patients. Which patient requires assessment first? a. A 40-yr-old patient with chronic pancreatitis who has gnawing abdominal pain b. A 58-yr-old patient who has compensated cirrhosis and is complaining of anorexia c. A 55-yr-old patient with cirrhosis and ascites who has an oral temperature of 102° F (38.8° C) d. A 36-yr-old patient recovering from a laparoscopic cholecystectomy who has severe shoulder pain

ANS: C This patient's history and fever suggest possible spontaneous bacterial peritonitis, which would require rapid assessment and interventions such as antibiotic therapy. The clinical manifestations for the other patients are consistent with their diagnoses and do not indicate complications are occurring. DIF: Cognitive Level: Analyze (analysis) REF: 989 OBJ: Special Questions: Multiple Patients TOP: Nursing Process: Assessment MSC: NCLEX: Safe and Effective Care Environment

A serum potassium level of 3.2 mEq/L (3.2 mmol/L) is reported for a patient with cirrhosis who has scheduled doses of spironolactone (Aldactone) and furosemide (Lasix) due. Which action should the nurse take? a. Withhold both drugs. b. Administer both drugs c. Administer the furosemide. d. Administer the spironolactone.

ANS: D Spironolactone is a potassium-sparing diuretic and will help increase the patient's potassiumlevel. The nurse does not need to talk with the doctor before giving the spironolactone,although the health care provider should be notified about the low potassium value. Thefurosemide will further decrease the patient's potassium level and should be held until thenurse talks with the health care provider.DIF: Cognitive Level: Apply (application) REF: 991TOP: Nursing Process: Implementation MSC: NCLEX: Physiological Integrity

Which nursing intervention for a patient who had an open repair of an abdominal aortic aneurysm 2 days previously is appropriate for the nurse to delegate to unlicensed assistive personnel (UAP)? a. Monitor the quality and presence of the pedal pulses. b. Teach the patient the signs of possible wound infection. c. Check the lower extremities for strength and movement. d. Help the patient to use a pillow to splint while coughing.

ANS: D Assisting a patient who has already been taught how to cough is part of routine postoperative care and within the education and scope of practice for UAP. Patient teaching and assessment of essential postoperative functions such as circulation and movement should be done by RNs

A patient is being evaluated for postthrombotic syndrome. Which assessment will the nurse perform? a. Ask about leg pain with exercise. b. Determine the ankle-brachial index. c. Assess capillary refill in the patient's toes. d. Inspect for presence of lipodermatosclerosis.

ANS: D Clinical signs of postthrombotic syndrome include lipodermatosclerosis. In this situation, the skin on the lower leg becomes scarred, and the leg becomes tapered like an "inverted bottle." The other assessments would be done for patients with peripheral arterial disease

Which topic should the nurse include in patient teaching for a patient with a venous stasis ulcer on the left lower leg? a. Need to increase carbohydrate intake b. Methods of keeping the wound area dry c. Purpose of prophylactic antibiotic therapy d. Application of elastic compression stockings

ANS: D Compression of the leg is essential to healing of venous stasis ulcers. High dietary intake of protein, rather than carbohydrates, is needed. Prophylactic antibiotics are not routinely used for venous ulcers. Moist dressings are used to hasten wound healing

A young adult contracts hepatitis from contaminated food. What should the nurse expect serologic testing to reveal during the acute (icteric) phase of the patient's illness? a. Antibody to hepatitis D (anti-HDV) b. Hepatitis B surface antigen (HBsAg) c. Anti-hepatitis A virus immunoglobulin G (anti-HAV IgG) d. Anti-hepatitis A virus immunoglobulin M (anti-HAV IgM)

ANS: D Hepatitis A is transmitted through the oral-fecal route, and antibody to HAV IgM appears during the acute phase of hepatitis A. The patient would not have antigen for hepatitis B or antibody for hepatitis D. Anti-HAV IgG would indicate past infection and lifelong immunity

A young adult contracts hepatitis from contaminated food. During the acute (icteric) phase of the patient's illness, the nurse would expect serologic testing to reveal a. antibody to hepatitis D (anti-HDV). b. hepatitis B surface antigen (HBsAg). c. anti-hepatitis A virus immunoglobulin G (anti-HAV IgG). d. anti-hepatitis A virus immunoglobulin M (anti-HAV IgM).

ANS: D Hepatitis A is transmitted through the oral-fecal route, and antibody to HAV IgM appearsduring the acute phase of hepatitis A. The patient would not have antigen for hepatitis B orantibody for hepatitis D. Anti-HAV IgG would indicate past infection and lifelong immunity.DIF: Cognitive Level: Apply (application) REF: 980TOP: Nursing Process: Assessment MSC: NCLEX: Physiological Integrity

Which action will be included in the care for a patient who has recently been diagnosed with asymptomatic nonalcoholic fatty liver disease (NAFLD)? a. Teach symptoms of variceal bleeding. b. Draw blood for hepatitis serology testing. c. Discuss the need to increase caloric intake. d. Review the patient's current medication list.

ANS: D Some medications can increase the risk for NAFLD, and they should be discontinued. NAFLD is not associated with hepatitis, weight loss is usually indicated, and variceal bleeding would not be a concern in a patient with asymptomatic NAFLD

Which action will be included in the care for a patient who has recently been diagnosed with asymptomatic nonalcoholic fatty liver disease (NAFLD)? a. Teach symptoms of variceal bleeding. b. Draw blood for hepatitis serology testing. c. Discuss the need to increase caloric intake. d. Review the patient's current medication list.

ANS: D Some medications can increase the risk for NAFLD, and they should be eliminated. NAFLD is not associated with hepatitis, weight loss is usually indicated, and variceal bleeding would not be a concern in a patient with asymptomatic NAFLD. DIF: Cognitive Level: Apply (application) REF: 985 TOP: Nursing Process: Planning MSC: NCLEX: Physiological Integrity

A serum potassium level of 3.2 mEq/L (3.2 mmol/L) is reported for a patient with cirrhosis who has scheduled doses of spironolactone (Aldactone) and furosemide (Lasix) due. Which action should the nurse take? a. Withhold both drugs. b. Administer both drugs. c. Administer the furosemide. d. Administer the spironolactone.

ANS: D Spironolactone is a potassium-sparing diuretic and will help increase the patient's potassium level. The furosemide will further decrease the patient's potassium level and should be held until the nurse talks with the health care provider.

Which finding indicates to the nurse that a patient's transjugular intrahepatic portosystemic shunt (TIPS) placed 3 months ago has been effective? a. Increased serum albumin level b. Decreased indirect bilirubin level c. Improved alertness and orientation d. Fewer episodes of bleeding varices

ANS: D TIPS is used to lower pressure in the portal venous system and decrease the risk of bleeding from esophageal varices. Indirect bilirubin level and serum albumin levels are not affected by shunting procedures. TIPS will increase the risk for hepatic encephalopathy

Which finding indicates to the nurse that a patient's transjugular intrahepatic portosystemic shunt (TIPS) placed 3 months ago has been effective? a. Increased serum albumin level b. Decreased indirect bilirubin level c. Improved alertness and orientation d. Fewer episodes of bleeding varices

ANS: D TIPS is used to lower pressure in the portal venous system and decrease the risk of bleeding from esophageal varices. Indirect bilirubin level and serum albumin levels are not affected by shunting procedures. TIPS will increase the risk for hepatic encephalopathy. DIF: Cognitive Level: Apply (application) REF: 992 TOP: Nursing Process: Evaluation MSC: NCLEX: Physiological Integrity

Several hours after a patient had an open surgical repair of an abdominal aortic aneurysm, the UAP reports to the nurse that urinary output for the past 2 hours has been 45 mL. The nurse notifies the health care provider and anticipates an order for a(n) a. hemoglobin count. b. additional antibiotic. c. serum creatinine level. d. increased IV infusion rate.

ANS: D The decreased urine output suggests decreased renal perfusion and monitoring of renal function is needed. There is no indication that infection is a concern, so antibiotic therapy and a WBC count are not needed. The IV rate may be increased because hypovolemia may be contributing to the patient's decreased urinary output

Which focused data should the nurse assess after identifying 4+ pitting edema on a patient who has cirrhosis? a. Hemoglobin b. Temperature c. Activity level d. Albumin level

ANS: D The low oncotic pressure caused by hypoalbuminemia is a major pathophysiologic factor in the development of edema. The other parameters are not directly associated with the patient's edema

Which focused data will the nurse monitor in relation to the 4+ pitting edema assessed in a patient with cirrhosis? a. Hemoglobin b. Temperature c. Activity level d. Albumin level

ANS: D The low oncotic pressure caused by hypoalbuminemia is a major pathophysiologic factor in the development of edema. The other parameters are not directly associated with the patient's edema. DIF: Cognitive Level: Apply (application) REF: 988 TOP: Nursing Process: Assessment MSC: NCLEX: Physiological Integrity

A patient at the clinic says, "I always walk after dinner, but lately my leg cramps and hurts after just a few minutes of starting. The pain goes away after I stop walking, though." The nurse should a. look for the presence of tortuous veins bilaterally on the legs. b. ask about any skin color changes that occur in response to cold. c. assess for unilateral swelling, redness, and tenderness of either leg. d. palpate for the presence of dorsalis pedis and posterior tibial pulses

ANS: D The nurse should assess for other clinical manifestations of peripheral arterial disease in a patient who describes intermittent claudication. Changes in skin color that occur in response to cold are consistent with Raynaud's phenomenon. Tortuous veins on the legs suggest venous insufficiency. Unilateral leg swelling, redness, and tenderness indicate venous thromboembolism

A nurse is considering which patient to admit to the same room as a patient who is hospitalized with acute rejection 3 weeks after a liver transplant. Which patient would be the best choice? a.Patient who is receiving chemotherapy for liver cancer b.Patient who is receiving treatment for acute hepatitis C c.Patient who has a wound infection after cholecystectomy d.Patient who requires pain management for chronic pancreatitis

ANS: D The patient with chronic pancreatitis does not present an infection risk to the immunosuppressed patient who had a liver transplant. The other patients either are at risk for infection or currently have an infection, which will place the immunosuppressed patient at risk for infection.

A nurse is considering which patient to admit to the same room as a patient who had a liver transplant 3 weeks ago and is now hospitalized with acute rejection. Which patient would be the best choice? a. Patient who is receiving chemotherapy for liver cancer b. Patient who is receiving treatment for acute hepatitis C c. Patient who has a wound infection after cholecystectomy d. Patient who requires pain management for chronic pancreatitis

ANS: D The patient with chronic pancreatitis does not present an infection risk to the immunosuppressed patient who had a liver transplant. The other patients either are at risk for infection or currently have an infection, which will place the immunosuppressed patient at risk for infection. DIF: Cognitive Level: Analyze (analysis) REF: 1003 OBJ: Special Questions: Multiple Patients TOP: Nursing Process: Planning MSC: NCLEX: Safe and Effective Care Environment

An older patient with chronic atrial fibrillation develops sudden severe pain, pulselessness, pallor, and coolness in the right leg. The nurse should notify the health care provider and immediately a. apply a compression stocking to the leg. b. elevate the leg above the level of the heart. c. assist the patient in gently exercising the leg. d. keep the patient in bed in the supine position

ANS: D The patient's history and clinical manifestations are consistent with acute arterial occlusion, and resting the leg will decrease the O2 demand of the tissues and minimize ischemic damage until circulation can be restored. Elevating the leg or applying an elastic wrap will further compromise blood flow to the leg. Exercise will increase oxygen demand for the tissues of the leg

A patient with cirrhosis has ascites and 4+ edema of the feet and legs. Which nursing action will be included in the plan of care? a. Restrict daily dietary protein intake. b. Reposition the patient every 4 hours. c. Perform passive range of motion twice daily. d. Place the patient on a pressure-relief mattress.

ANS: D The pressure-relieving mattress will decrease the risk for skin breakdown for this patient. Adequate dietary protein intake is necessary in patients with ascites to improve oncotic pressure. Repositioning the patient every 4 hours will not be adequate to maintain skin integrity. Passive range of motion will not take the pressure off areas such as the sacrum that are vulnerable to breakdown

A patient with cirrhosis has ascites and 4+ edema of the feet and legs. Which nursing action will be included in the plan of care? a. Restrict daily dietary protein intake. b. Reposition the patient every 4 hours. c. Perform passive range of motion twice daily. d. Place the patient on a pressure-relief mattress.

ANS: D The pressure-relieving mattress will decrease the risk for skin breakdown for this patient. Adequate dietary protein intake is necessary in patients with ascites to improve oncotic pressure. Repositioning the patient every 4 hours will not be adequate to maintain skin integrity. Passive range of motion will not take the pressure off areas such as the sacrum that are vulnerable to breakdown. DIF: Cognitive Level: Apply (application) REF: 994 TOP: Nursing Process: Implementation MSC: NCLEX: Physiological Integrity

The health care provider has prescribed bed rest with the feet elevated for a patient admitted to the hospital with venous thromboembolism. Which action by the nurse to elevate the patient's feet is best? a. The patient is placed in the Trendelenburg position. b. Two pillows are positioned under the affected leg. c. The bed is elevated at the knee and pillows are placed under the feet. d. One pillow is placed under the thighs and two pillows are placed under the lower legs

ANS: D The purpose of elevating the feet is to enhance venous flow from the feet to the right atrium, which is best accomplished by placing two pillows under the feet and one under the thighs. Placing the patient in the Trendelenburg position will lower the head below heart level, which is not indicated for this patient. Placing pillows under the calf or elevating the bed at the knee may cause blood stasis at the calf level.

When discussing risk factor modification for a patient who has a 5-cm abdominal aortic aneurysm, the nurse will focus teaching on which patient risk factor?

ANS: D. Uncontrolled hypertension hypertension can potentially be modified to decrease the patient's risk for further expansion of the aneurysm.DIF: Cognitive Level: Apply (application) REF: 810

The nurse provides discharge instructions for a 64 y.o. women with ascites and peripheral edema related to cirrhosis. Which statement, if made by the pt, indicates teaching WAS effective? a. it is safe to take acetaminophen up to four times a day for pain b. lactulose (cephulac) should be taken everyday to prevent constipation c. herbs and other spices should be used to season my foods instead of salt d. i will eat foods high in potassium while taking spironolactone (aldactone)

C low sodium diet is indicated for pt with ascites and edema related to cirrhosis

The nurse performing an assessment of a patient who has chronic peripheral artery disease (PAD) of the legs and an ulcer on the right second toe would expect to find a. dilated superficial veins. b. swollen, dry, scaly ankles. c. prolonged capillary refill in all the toes. d. serosanguineous drainage from the ulcer

ANS:C Capillary refill is prolonged in PAD because of the slower and decreased blood flow to the periphery. The other listed clinical manifestations are consistent with chronic venous disease

A patient presents to the clinic with complaints of muscle aches, muscle pain, and weakness. Upon review of the individual's medications, the nurse notes that the patient is concurrently taking gemfibrozil (Lopid) and atorvastatin (Lipitor). The nurse should assess the patient for the development of: a. migraines. b. hypothyroidism. c. myopathy. d. heart failure.

C When gemfibrozil and a statin are taken concurrently, the risk of myopathy is increased more than with either agent alone. Migraines, hypothyroidism, and heart failure are not drug-to-drug interactions.

HBV core Ab (HBVcAb) IgG indicates

Ab (HBVcAb) IgG indicates that the patient has a history of HBV infection.

abdominal aorta aneurysm

Abdominal aortic aneurysm Also called: AAA -An aneurysm that is below renal arteries and above bifurcation of aorta. pulsations can be felt -Most abdominal aortic aneurysms (AAAs) occur below the renal arteries -Risk factors for aortic aneurysms include age, male gender, hypertension, CAD, family history, tobacco use, high cholesterol, lower extremity PAD, carotid artery disease, previous stroke, and obesity. Tobacco use is the most important modifiable risk factor.1,13 The larger the aneurysm, the greater is the risk for rupture.

A nurse is caring for a client with cirrhosis. Which assessment finding warrants immediate attention? Pulse of 60 bpm Oxygen saturation of 92% Blood pressure of 110/72 mmHg Abdominal distention

Abdominal distention Rationale Abdominal distention, which is an imbalance of fluid within the portal system, might mean ascites in a client with cirrhosis. The vital signs are all within normal limits.

anorexia, lethargy, nausea, vomiting, skin rashes, diarrhea or constipation, malaise, fatigue, myalgias, arthralgias, other flu-like symptoms, and right upper quadrant tenderness (caused by liver inflammation). icteric (jaundiced) or anicteric,urine may appear darker,stools will be clay colored.

Acute Hepatitis -1 to 6 months. -patient may have a decreased sense of smell and find food repugnant. -Smokers may have distaste for cigarettes. -Physical examination often reveals hepatomegaly, lymphadenopathy, abdominal tenderness, and sometimes splenomegaly. - The acute phase is the period of maximal infectivity.

Overall goals for patient with PAD

Adequate tissue perfusion Relief of pain Increased exercise tolerance Intact, healthy skin on extremities Increased knowledge of disease and treatment plan

A prescriber has ordered rosuvastatin (Crestor) for a patient with non-alcoholic-related cirrhosis. Which intervention would be most appropriate for the nurse before administration of this drug? a. Question the order, because rosuvastatin is contraindicated in patients with liver disease. b. No intervention is necessary; just administer the drug as ordered. c. Review the baseline liver function test results. d. Assess the patient for liver disease.

C Before administering rosuvastatin, the nurse should review the results of the baseline liver function tests (LFTs). Statins can be used by patients with liver disease but not by those with alcoholic or viral hepatitis. Administering the drug before obtaining baseline LFT results would not allow for an accurate baseline. The patient clearly already has liver disease

A patient with malaise has been taking daptomycin (Cubicin) for an infection and is concurrently taking simvastatin (Zocor). The nurse should be concerned if the patient complains of: a. nausea. b. tiredness. c. muscle pain. d. headache.

C Statins can injure muscle tissue, causing muscle aches and pain known as myopathy/rhabdomyolysis. Daptomycin also can cause myopathy and therefore should be used with caution in patients concurrently taking simvastatin. Nausea, tiredness, and headache would not cause the nurse as much concern as the likelihood of myopathy.

GI status after aneurysm repair

After OAR, postoperative ileus may develop because of anesthesia and the handling of the bowel during surgery. The intestines may become swollen and bruised, and peristalsis ceases for variable intervals. A retroperitoneal surgical approach decreases the risk for bowel complications. NG tube may be present and connected to low, intermittent suction to decompress the stomach, prevent aspiration of stomach contents, and decrease pressure on suture lines. Record the amount and character of the NG output. While the patient is NPO, provide frequent oral care. Ice chips or lozenges can help soothe a dry or irritated throat. Assess for bowel sounds every 4 hours. Note the passing of flatus as it signals returning bowel function. Encourage early ambulation since this will help the return of bowel function. A postoperative ileus rarely lasts beyond the 4th postoperative day.

Ambulatory and Home Care Hep

All pts w/ chronic HBV or HCV should avoid alcohol, since it can accelerate disease progression. Pts who are positive for HBsAg (chronic carrier status) or HCV antibody should not be blood donors

Endovascular Graft Procedure

Alternative to conventional surgical repair Involves placement of sutureless aortic graft into abdominal aorta inside aneurysm Minimally invasive Done through femoral artery cutdown Minimally invasive endovascular aneurysm repair (EVAR) is an alternative to OAR for select patients. Eligibility criteria include iliofemoral vessels that allow for safe graft insertion and vessels of sufficient length and width to support the graft. TEVAR does not prevent the risk for renal failure, paraplegia, or stroke Patients undergoing EVAR need periodic imaging for the rest of their lives to monitor for an endoleak, document stability of the aneurysm sac, and determine the need for surgical intervention. has fewer complications, such as paraplegia and death. A potentially lethal complication:development of intraabdominal hypertension (IAH) with associated abdominal compartment syndrome. resulting multisystem organ failure. Monitor the patient for signs of aneurysm rupture. These include diaphoresis; pallor; weakness; tachycardia; hypotension; abdominal, back, groin, or periumbilical pain; changes in level of consciousness; or a pulsating abdominal mass. Avoid severe hypertension, which may stress the arterial anastomoses, resulting in leakage of blood or rupture at the suture lines. Drug therapy with IV diuretics (e.g., furosemide) or IV antihypertensive agents (e.g., labetalol, metoprolol, hydralazine, sodium nitroprusside [Nipride]) may be indicated. Nursing interventions include continuous ECG monitoring; frequent electrolyte and arterial blood gas determinations; O2 administration, IV antidysrhythmic and antihypertensive drugs, and electrolytes as needed; adequate pain control; and resumption of cardiac drugs. Assess temperature regularly, and promptly report elevations. Monitor laboratory results for a high WBC count, which may be the first sign of an infection. Ensure adequate nutrition and assess the surgical incision for signs of infection (e.g., redness, swelling, drainage). Keep surgical incisions clean and dry and perform wound care as prescribed. Use good hand-washing and strict aseptic technique in the care of all peripheral, arterial, and CVP catheter insertion sites, since these are ports of entry for bacteria. Meticulous perineal care for the patient with an indwelling urinary catheter and early catheter removal are essential to minimize the risk for urinary tract infection. Assess for bowel sounds every 4 hours. Note the passing of flatus as it signals returning bowel function. Encourage early ambulation since this will help the return of bowel function. A postoperative ileus rarely lasts beyond the 4th postoperative day. Manifestations of this rare, but serious, complication include absent bowel sounds, fever, abdominal distention and pain, diarrhea, and bloody stools. If bowel infarction occurs, immediate reoperation is needed to restore blood flow, with resection of the infarcted bowel. When the ascending aorta and aortic arch are involved, assess the patient's level of consciousness, pupil size and response to light, facial symmetry, tongue position, speech, upper extremity movement, and quality of hand grasps). When the descending aorta is involved, perform a neurovascular assessment of the lower extremities. Record all assessments and report changes from baseline to the HCP immediately. Pulses: A decreased or absent pulse together with a cool, pale, mottled, or painful extremity may indicate embolization or graft occlusion. Report these findings to the HCP at once.

Graft Patency anuerysm repair

An adequate BP is important to maintain graft patency. Prolonged low BP may result in graft thrombosis. Give IV fluids and blood components as ordered to maintain adequate blood flow. Monitor CVP or PA pressures and urine output hourly in the immediate postoperative period to assess the patient's hydration and perfusion status. Avoid severe hypertension, which may stress the arterial anastomoses, resulting in leakage of blood or rupture at the suture lines. Drug therapy with IV diuretics (e.g., furosemide) or IV antihypertensive agents (e.g., labetalol, metoprolol, hydralazine, sodium nitroprusside [Nipride]) may be indicated.

azilsartan (Edarbi) candesartan (Atacand) eprosartan (Teveten) irbesartan (Avapro) losartan (Cozaar) olmesartan (Benicar) telmisartan (Micardis) valsartan (Diovan)

Angiotensin II Receptor Blockers (ARBs) MOA:Prevent action of A-II and produce vasodilation and increased Na+ and water excretion Full effect on BP may not be seen for 3-6 wk nursing consideration: Do not affect bradykinin levels, therefore an acceptable alternative to ACE inhibitors in people who develop a dry cough In patients with kidney disease, ACE inhibitors and ARBs should not be used together due to adverse renal effects

benazepril (Lotensin) captopril enalapril (Vasotec) fosinopril lisinopril (Zestril) moexipril perindopril quinapril (Accupril) ramipril (Altace) trandolapril (Mavik)

Angiotensin Inhibitors Angiotensin-Converting Enzyme (ACE) Inhibitors: MOA: Inhibit ACE, reduce conversion of angiotensin I to angiotensin II (A-II) Inhibit A-II-mediated vasoconstriction consideations: Aspirin and NSAIDs may reduce effectiveness Adding a diuretic enhances effect, but should not be used with potassium-sparing diuretics Can cause an increase in serum creatinine Inhibit breakdown of bradykinin, which may cause a dry, hacking cough that can occur at any point during treatment, even years later Captopril may be given orally for hypertensive crisis

Drug Therapy PAD

Angiotensin-converting enzyme (ACE) inhibitors (e.g., ramipril [Altace]) can reduce PAD symptoms. Antiplatelet agents are critical for reducing the risks for CVD events and death. Oral antiplatelet therapy should include low-dose aspirin therapy. Aspirin-intolerant patients may take clopidogrel (Plavix) daily. Combination antiplatelet therapy with aspirin and clopidogrel may be used by select high-risk patients. Anticoagulants (e.g., warfarin [Coumadin]) are not recommended for prevention of CVD events in patients with PAD. Two drugs are available to treat intermittent claudication: cilostazol and pentoxifylline. Cilostazol, a phosphodiesterase inhibitor, inhibits platelet aggregation and increases vasodilation. Pentoxifylline, a xanthine derivative, improves the flexibility of RBCs and WBCs and decreases fibrinogen concentration, platelet adhesiveness, and blood viscosity. It is not as effective as cilostazol. Cilostazol is usually stopped within 3 months due to side effects.3

During a health education session, a participant has asked about the hepatitis E virus. What prevention measure should the nurse recommend for preventing infection with this virus? A) Following proper hand-washing techniques B) Avoiding chemicals that are toxic to the liver C) Wearing a condom during sexual contact D) Limiting alcohol intake

Ans: A Feedback: Avoiding contact with the hepatitis E virus through good hygiene, including hand-washing, is the major method of prevention. Hepatitis E is transmitted by the fecal-oral route, principally through contaminated water in areas with poor sanitation. Consequently, none of the other listed preventative measures is indicated.

A patient with primary hypertension comes to the clinic complaining of a gradual onset of blurry vision and decreased visual acuity over the past several weeks. The nurse is aware that these symptoms could be indicative of what? A) Retinal blood vessel damage B) Glaucoma C) Cranial nerve damage D) Hypertensive emergency

Ans: A Feedback: Blurred vision, spots in front of the eyes, and diminished visual acuity can mean retinal blood vessel damage indicative of damage elsewhere in the vascular system as a result of hypertension. Glaucoma and cranial nerve damage do not normally cause these symptoms. A hypertensive emergency would have a more rapid onset

The nurse is developing a nursing care plan for a patient who is being treated for hypertension. What is a measurable patient outcome that the nurse should include? A) Patient will reduce Na+ intake to no more than 2.4 g daily. B) Patient will have a stable BUN and serum creatinine levels. C) Patient will abstain from fat intake and reduce calorie intake. D) Patient will maintain a normal body weight.

Ans: A Feedback: Dietary sodium intake of no more than 2.4 g sodium is recommended as a dietary lifestyle modification to prevent and manage hypertension. Giving a specific amount of allowable sodium intake makes this a measurable goal. None of the other listed goals is quantifiable and measurable.

A group of nurses have attended an inservice on the prevention of occupationally acquired diseases that affect healthcare providers. What action has the greatest potential to reduce a nurse's risk of acquiring hepatitis C in the workplace? A) Disposing of sharps appropriately and not recapping needles B) Performing meticulous hand hygiene at the appropriate moments in care C) Adhering to the recommended schedule of immunizations D) Wearing an N95 mask when providing care for patients on airborne precautions

Ans: A Feedback: HCV is bloodborne. Consequently, prevention of needlestick injuries is paramount. Hand hygiene, immunizations and appropriate use of masks are important aspects of overall infection control, but these actions do not directly mitigate the risk of HCV.

A patient with advanced venous insufficiency is confined following orthopedic surgery. How can the nurse best prevent skin breakdown in the patient's lower extremities? A) Ensure that the patient's heels are protected and supported. B) Closely monitor the patient's serum albumin and prealbumin levels. C) Perform gentle massage of the patient's lower legs, as tolerated. D) Perform passive range-of-motion exercises once per shift.

Ans: A Feedback: If the patient is on bed rest, it is important to relieve pressure on the heels to prevent pressure ulcerations, since the heels are among the most vulnerable body regions. Monitoring blood work does not directly prevent skin breakdown, even though albumin is related to wound healing. Massage is not normally indicated and may exacerbate skin breakdown. Passive range- of-motion exercises do not directly reduce the risk of skin breakdown.

A nurse on a solid organ transplant unit is planning the care of a patient who will soon be admitted upon immediate recovery following liver transplantation. What aspect of nursing care is the nurse's priority? A) Implementation of infection-control measures B) Close monitoring of skin integrity and color C) Frequent assessment of the patient's psychosocial status D) Administration of antiretroviral medications

Ans: A Feedback: Infection control is paramount following liver transplantation. This is a priority over skin integrity and psychosocial status, even though these are valid areas of assessment and intervention. Antiretrovirals are not indicated.

A patient is being discharged after a liver transplant and the nurse is performing discharge education. When planning this patient's continuing care, the nurse should prioritize which of the following risk diagnoses? A) Risk for Infection Related to Immunosuppressant Use B) Risk for Injury Related to Decreased Hemostasis C) Risk for Unstable Blood Glucose Related to Impaired Gluconeogenesis D) Risk for Contamination Related to Accumulation of Ammonia

Ans: A Feedback: Infection is the leading cause of death after liver transplantation. Pulmonary and fungal infections are common; susceptibility to infection is increased by the immunosuppressive therapy that is needed to prevent rejection. This risk exceeds the threats of injury and unstable blood glucose. The diagnosis of Risk for Contamination relates to environmental toxin exposure.

A patient has developed hepatic encephalopathy secondary to cirrhosis and is receiving care on the medical unit. The patient's current medication regimen includes lactulose (Cephulac) four times daily. What desired outcome should the nurse relate to this pharmacologic intervention? A) Two to 3 soft bowel movements daily B) Significant increase in appetite and food intake C) Absence of nausea and vomiting D) Absence of blood or mucus in stool

Ans: A Feedback: Lactulose (Cephulac) is administered to reduce serum ammonia levels. Two or three soft stools per day are desirable; this indicates that lactulose is performing as intended. Lactulose does not address the patient's appetite, symptoms of nausea and vomiting, or the development of blood and mucus in the stool.

A nurse is teaching an adult female patient about the risk factors for hypertension. What should the nurse explain as risk factors for primary hypertension? A) Obesity and high intake of sodium and saturated fat B) Diabetes and use of oral contraceptives C) Metabolic syndrome and smoking D) Renal disease and coarctation of the aorta

Ans: A Feedback: Obesity, stress, high intake of sodium or saturated fat, and family history are all risk factors for primary hypertension. Diabetes and oral contraceptives are risk factors for secondary hypertension. Metabolic syndrome, renal disease, and coarctation of the aorta are causes of secondary hypertension.

The nurse is admitting a 32-year-old woman to the presurgical unit. The nurse learns during the admission assessment that the patient takes oral contraceptives. Consequently, the nurse's postoperative plan of care should include what intervention? A) Early ambulation and leg exercises B) Cessation of the oral contraceptives until 3 weeks postoperative C) Doppler ultrasound of peripheral circulation twice daily D) Dependent positioning of the patient's extremities when at rest

Ans: A Feedback: Oral contraceptive use increases blood coagulability; with bed rest, the patient may be at increased risk of developing deep vein thrombosis. Leg exercises and early ambulation are among the interventions that address this risk. Assessment of peripheral circulation is important, but Doppler ultrasound may not be necessary to obtain these data. Dependent positioning increases the risk of venous thromboembolism (VTE). Contraceptives are not normally discontinued to address the risk of VTE in the short term.

A patient with primary hypertension complains of dizziness with ambulation. The patient is currently on an alpha-adrenergic blocker and the nurse assesses characteristic signs and symptoms of postural hypotension. When teaching this patient about risks associated with postural hypotension, what should the nurse emphasize? A) Rising slowly from a lying or sitting position B) Increasing fluids to maintain BP C) Stopping medication if dizziness persists D) Taking medication first thing in the morning

Ans: A Feedback: Patients who experience postural hypotension should be taught to rise slowly from a lying or sitting position and use a cane or walker if necessary for safety. It is not necessary to teach these patients about increasing fluids or taking medication in the morning (this would increase the effects of dizziness). Patient should not be taught to stop the medication if dizziness persists because this is unsafe and beyond the nurse's scope of practice.

A patient has come to the clinic for a follow-up assessment that will include a BP reading. To ensure an accurate reading, the nurse should confirm that the patient has done which of the following? A) Tried to rest quietly for 5 minutes before the reading is taken B) Refrained from smoking for at least 8 hours C) Drunk adequate fluids during the day prior D) Avoided drinking coffee for 12 hours before the visit

Ans: A Feedback: Prior to the nurse assessing the patient's BP, the patient should try to rest quietly for 5 minutes. The forearm should be positioned at heart level. Caffeine products and cigarette smoking should be avoided for at least 30 minutes prior to the visit. Recent fluid intake is not normally relevant.

A patient with secondary hypertension has come into the clinic for a routine check-up. The nurse is aware that the difference between primary hypertension and secondary hypertension is which of the following? A) Secondary hypertension has a specific cause. B) Secondary hypertension has a more gradual onset than primary hypertension. C) Secondary hypertension does not cause target organ damage. D) Secondary hypertension does not normally respond to antihypertensive drug therapy.

Ans: A Feedback: Secondary hypertension has a specific identified cause. A cause could include narrowing of the renal arteries, renal parenchymal disease, hyperaldosteronism, certain medications, pregnancy, and coarctation of the aorta. Secondary hypertension does respond to antihypertensive drug therapy and can cause target organ damage if left untreated.

A patient has been prescribed antihypertensives. After assessment and analysis, the nurse has identified a nursing diagnosis of risk for ineffective health maintenance related to nonadherence to therapeutic regimen. When planning this patient's care, what desired outcome should the nurse identify? A) Patient takes medication as prescribed and reports any adverse effects. B) Patient's BP remains consistently below 140/90 mm Hg. C) Patient denies signs and symptoms of hypertensive urgency. D) Patient is able to describe modifiable risk factors for hypertension.

Ans: A Feedback: The most appropriate expected outcome for a patient who is given the nursing diagnosis of risk for ineffective health maintenance is that he or she takes the medication as prescribed. The other listed goals are valid aspects of care, but none directly relates to the patient's role in his or her treatment regimen.

A nurse is caring for a patient with hepatic encephalopathy. While making the initial shift assessment, the nurse notes that the patient has a flapping tremor of the hands. The nurse should document the presence of what sign of liver disease? A) Asterixis B) Constructional apraxia C) Fetor hepaticus D) Palmar erythema

Ans: A Feedback: The nurse will document that a patient exhibiting a flapping tremor of the hands is demonstrating asterixis. While constructional apraxia is a motor disturbance, it is the inability to reproduce a simple figure. Fetor hepaticus is a sweet, slightly fecal odor to the breath and not associated with a motor disturbance. Skin changes associated with liver dysfunction may include palmar erythema, which is a reddening of the palms, but is not a flapping tremor

The clinic nurse is caring for a 57-year-old client who reports experiencing leg pain whenever she walks several blocks. The patient has type 1 diabetes and has smoked a pack of cigarettes every day for the past 40 years. The physician diagnoses intermittent claudication. The nurse should provide what instruction about long-term care to the client? A) "Be sure to practice meticulous foot care." B) "Consider cutting down on your smoking." C) "Reduce your activity level to accommodate your limitations." D) "Try to make sure you eat enough protein."

Ans: A Feedback: The patient with peripheral vascular disease or diabetes should receive education or reinforcement about skin and foot care. Intermittent claudication and other chronic peripheral vascular diseases reduce oxygenation to the feet, making them susceptible to injury and poor healing; therefore, meticulous foot care is essential. The patient should stop smoking—not just cut down—because nicotine is a vasoconstrictor. Daily walking benefits the patient with intermittent claudication. Increased protein intake will not alleviate the patient's symptoms.

A patient who has undergone a femoral to popliteal bypass graft surgery returns to the surgical unit. Which assessments should the nurse perform during the first postoperative day? A) Assess pulse of affected extremity every 15 minutes at first. B) Palpate the affected leg for pain during every assessment. C) Assess the patient for signs and symptoms of compartment syndrome every 2 hours. D) Perform Doppler evaluation once daily.

Ans: A Feedback: The primary objective in the postoperative period is to maintain adequate circulation through the arterial repair. Pulses, Doppler assessment, color and temperature, capillary refill, and sensory and motor function of the affected extremity are checked and compared with those of the other extremity; these values are recorded initially every 15 minutes and then at progressively longer intervals if the patient's status remains stable. Doppler evaluations should be performed every 2 hours. Pain is regularly assessed, but palpation is not the preferred method of performing this assessment. Compartment syndrome results from the placement of a cast, not from vascular surgery.

A patient newly diagnosed with hypertension asks the nurse what happens when uncontrolled hypertension is prolonged. The nurse explains that a patient with prolonged, uncontrolled hypertension is at risk for developing what health problem? A) Renal failure B) Right ventricular hypertrophy C) Glaucoma D) Anemia

Ans: A Feedback: When uncontrolled hypertension is prolonged, it can result in renal failure, myocardial infarction, stroke, impaired vision, left ventricular hypertrophy, and cardiac failure. Glaucoma and anemia are not directly associated with hypertension.

The nurse is caring for a patient with a large venous leg ulcer. What intervention should the nurse implement to promote healing and prevent infection? A) Provide a high-calorie, high-protein diet. B) Apply a clean occlusive dressing once daily and whenever soiled. C) Irrigate the wound with hydrogen peroxide once daily. D) Apply an antibiotic ointment on the surrounding skin with each dressing

Ans: A Feedback: Wound healing is highly dependent on adequate nutrition. The diet should be sufficiently high in calories and protein. Antibiotic ointments are not normally used on the skin surrounding a leg ulcer and occlusive dressings can exacerbate impaired blood flow. Hydrogen peroxide is not normally used because it can damage granulation tissue.

A nurse educator is teaching a group of recent nursing graduates about their occupational risks for contracting hepatitis B. What preventative measures should the educator promote? Select all that apply. A) Immunization B) Use of standard precautions C) Consumption of a vitamin-rich diet D) Annual vitamin K injections E) Annual vitamin B12 injections

Ans: A, B Feedback: People who are at high risk, including nurses and other health care personnel exposed to blood or blood products, should receive active immunization. The consistent use of standard precautions is also highly beneficial. Vitamin supplementation is unrelated to an individual's risk of HBV.

The nurse is teaching a patient about some of the health consequences of uncontrolled hypertension. What health problems should the nurse describe? Select all that apply. A) Transient ischemic attacks B) Cerebrovascular accident C) Retinal hemorrhage D) Venous insufficiency E) Right ventricular hypertrophy

Ans: A, B, C Feedback: Potential complications of hypertension include the following: left ventricular hypertrophy; MI; heart failure; transient ischemic attacks (TIAs); cerebrovascular accident; renal insufficiency and failure; and retinal hemorrhage. Venous insufficiency and right ventricular hypertrophy are not potential complications of uncontrolled hypertension.

A patient's physician has ordered a "liver panel" in response to the patient's development of jaundice. When reviewing the results of this laboratory testing, the nurse should expect to review what blood tests? Select all that apply. A) Alanine aminotransferase (ALT) B) C-reactive protein (CRP) C) Gamma-glutamyl transferase (GGT) D) Aspartate aminotransferase (AST) E) B-type natriuretic peptide (BNP)

Ans: A, C, D Feedback: Liver function testing includes GGT, ALT, and AST. CRP addresses the presence of generalized inflammation and BNP is relevant to heart failure; neither is included in a liver panel.

The nurse has performed a thorough nursing assessment of the care of a patient with chronic leg ulcers. The nurse's assessment should include which of the following components? Select all that apply. A) Location and type of pain B) Apical heart rate C) Bilateral comparison of peripheral pulses D) Comparison of temperature in the patient's legs E) Identification of mobility limitations

Ans: A, C, D, E Feedback: A careful nursing history and assessment are important. The extent and type of pain are carefully assessed, as are the appearance and temperature of the skin of both legs. The quality of all peripheral pulses is assessed, and the pulses in both legs are compared. Any limitation of mobility and activity that results from vascular insufficiency is identified. Not likely is there any direct indication for assessment of apical heart rate, although peripheral pulses must be assessed.

The nurse is taking a health history of a new patient. The patient reports experiencing pain in his left lower leg and foot when walking. This pain is relieved with rest. The nurse notes that the left lower leg is slightly edematous and is hairless. When planning this patient's subsequent care, the nurse should most likely address what health problem? A) Coronary artery disease (CAD) B) Intermittent claudication C) Arterial embolus D) Raynaud's disease

Ans: B Feedback: A muscular, cramp-type pain in the extremities consistently reproduced with the same degree of exercise or activity and relieved by rest is experienced by patients with peripheral arterial insufficiency. Referred to as intermittent claudication, this pain is caused by the inability of the arterial system to provide adequate blood flow to the tissues in the face of increased demands for nutrients and oxygen during exercise. The nurse would not suspect the patient has CAD, arterial embolus, or Raynaud's disease; none of these health problems produce this cluster of signs and symptoms.

An occupational health nurse is providing an educational event and has been asked by an administrative worker about the risk of varicose veins. What should the nurse suggest as a proactive preventative measure for varicose veins? A) Sit with crossed legs for a few minutes each hour to promote relaxation. B) Walk for several minutes every hour to promote circulation. C) Elevate the legs when tired. D) Wear snug-fitting ankle socks to decrease edema.

Ans: B Feedback: A proactive approach to preventing varicose veins would be to walk for several minutes every hour to promote circulation. Sitting with crossed legs may promote relaxation, but it is contraindicated for patients with, or at risk for, varicose veins. Elevating the legs only helps blood passively return to the heart and does not help maintain the competency of the valves in the veins. Wearing tight ankle socks is contraindicated for patients with, or at risk for, varicose veins; socks that are below the muscles of the calf do not promote venous return, the socks simply capture the blood and promote venous stasis.

A patient with a diagnosis of esophageal varices has undergone endoscopy to gauge the progression of this complication of liver disease. Following the completion of this diagnostic test, what nursing intervention should the nurse perform? A) Keep patient NPO until the results of test are known. B) Keep patient NPO until the patient's gag reflex returns. C) Administer analgesia until post-procedure tenderness is relieved. D) Give the patient a cold beverage to promote swallowing ability.

Ans: B Feedback: After the examination, fluids are not given until the patient's gag reflex returns. Lozenges and gargles may be used to relieve throat discomfort if the patient's physical condition and mental status permit. The result of the test is known immediately. Food and fluids are contraindicated until the gag reflex returns.

A nurse in a long-term care facility is caring for an 83-year-old woman who has a history of HF and peripheral arterial disease (PAD). At present the patient is unable to stand or ambulate. The nurse should implement measures to prevent what complication? A) Aoritis B) Deep vein thrombosis C) Thoracic aortic aneurysm D) Raynaud's disease

Ans: B Feedback: Although the exact cause of venous thrombosis remains unclear, three factors, known as Virchow's triad, are believed to play a significant role in its development: stasis of blood (venous stasis), vessel wall injury, and altered blood coagulation. In this woman's case, she has venous stasis from immobility, vessel wall injury from PAD, and altered blood coagulation from HF. The cause of aoritis is unknown, but it has no direct connection to HF, PAD, or mobility issues. The greatest risk factors for thoracic aortic aneurysm are atherosclerosis and hypertension; there is no direct connection to HF, PAD, or mobility issues. Raynaud's disease is a disorder that involves spasms of blood vessels and, again, no direct connection to HF, PAD, or mobility issues.

The nurse is doing discharge teaching with a patient who has coronary artery disease. The patient asks why he has to take an aspirin every day if he doesn't have any pain. What would be the nurse's best response? A) "Taking an aspirin every day is an easy way to help restore the normal function of your heart." B) "An aspirin a day can help prevent some of the blockages that can cause chest pain or heart attacks." C) "Taking an aspirin every day is a simple way to make your blood penetrate your heart more freely." D) "An aspirin a day eventually helps your blood carry more oxygen that it would otherwise."

Ans: B Feedback: An aspirin a day is a common nonprescription medication that improves outcomes in patients with CAD due to its antiplatelet action. It does not affect oxygen carrying capacity or perfusion. Aspirin does not restore cardiac function.

The nurse is caring for a patient who has a history of heart disease. What factor should the nurse identify as possibly contributing to a decrease in cardiac output? A) A change in position from standing to sitting B) A heart rate of 54 bpm C) A pulse oximetry reading of 94% D) An increase in preload related to ambulation

Ans: B Feedback: Cardiac output is computed by multiplying the stroke volume by the heart rate. Cardiac output can be affected by changes in either stroke volume or heart rate, such as a rate of 54 bpm. An increase in preload will lead to an increase in stroke volume. A pulse oximetry reading of 94% does not indicate hypoxemia, as hypoxia can decrease contractility. Transitioning from standing to sitting would more likely increase rather than decrease cardiac output.

A resident of a long-term care facility has complained to the nurse of chest pain. What aspect of the resident's pain would be most suggestive of angina as the cause? A) The pain is worse when the resident inhales deeply. B) The pain occurs immediately following physical exertion. C) The pain is worse when the resident coughs. D) The pain is most severe when the resident moves his upper body.

Ans: B Feedback: Chest pain associated with angina is often precipitated by physical exertion. The other listed aspects of chest pain are more closely associated with noncardiac etiologies

The hospital nurse cares for many patients who have hypertension. What nursing diagnosis is most common among patients who are being treated for this health problem? A) Deficient knowledge regarding the lifestyle modifications for management of hypertension B) Noncompliance with therapeutic regimen related to adverse effects of prescribed therapy C) Deficient knowledge regarding BP monitoring D) Noncompliance with treatment regimen related to medication costs

Ans: B Feedback: Deviation from the therapeutic program is a significant problem for people with hypertension and other chronic conditions requiring lifetime management. For many patients, this is related to adverse effects of medications. Medication cost is relevant for many patients, but adverse effects are thought to be a more significant barrier. Many patients are aware of necessary lifestyle modification, but do not adhere to them. Most patients are aware of the need to monitor their BP.

The nurse is caring for an older adult with a diagnosis of hypertension who is being treated with a diuretic and beta-blocker. Which of the following should the nurse integrate into the management of this client's hypertension? A) Ensure that the patient receives a larger initial dose of antihypertensive medication due to impaired absorption. B) Pay close attention to hydration status because of increased sensitivity to extracellular volume depletion. C) Recognize that an older adult is less likely to adhere to his or her medication regimen than a younger patient. D) Carefully assess for weight loss because of impaired kidney function resulting from normal aging.

Ans: B Feedback: Elderly people have impaired cardiovascular reflexes and thus are more sensitive to extracellular volume depletion caused by diuretics. The nurse needs to assess hydration status, low BP, and postural hypotension carefully. Older adults may have impaired absorption, but they do not need a higher initial dose of an antihypertensive than a younger person. Adherence to treatment is not necessarily linked to age. Kidney function and absorption decline with age; less, rather than more antihypertensive medication is prescribed. Weight gain is not necessarily indicative of kidney function decline.

A nurse is caring for a patient who has been admitted for the treatment of advanced cirrhosis. What assessment should the nurse prioritize in this patient's plan of care? A) Measurement of abdominal girth and body weight B) Assessment for variceal bleeding C) Assessment for signs and symptoms of jaundice D) Monitoring of results of liver function testing

Ans: B Feedback: Esophageal varices are a major cause of mortality in patients with uncompensated cirrhosis. Consequently, this should be a focus of the nurse's assessments and should be prioritized over the other listed assessments, even though each should be performed.

The physician has ordered a high-sensitivity C-reactive protein (hs-CRP) drawn on a patient. The results of this test will allow the nurse to evaluate the role of what process that is implicated in the development of atherosclerosis? A) Immunosuppression B) Inflammation C) Infection D) Hemostasis

Ans: B Feedback: High-sensitivity CRP is a protein produced by the liver in response to systemic inflammation. Inflammation is thought to play a role in the development and progression of atherosclerosis.

The nurse is caring for a 72-year-old patient who is in cardiac rehabilitation following heart surgery. The patient has been walking on a regular basis for about a week and walks for 15 minutes 3 times a day. The patient states that he is having a cramp-like pain in the legs every time he walks and that the pain gets "better when I rest." The patient's care plan should address what problem? A) Decreased mobility related to VTE B) Acute pain related to intermittent claudication C) Decreased mobility related to venous insufficiency D) Acute pain related to vasculitis

Ans: B Feedback: Intermittent claudication presents as a muscular, cramp-type pain in the extremities consistently reproduced with the same degree of exercise or activity and relieved by rest. Patients with peripheral arterial insufficiency often complain of intermittent claudication due to a lack of oxygen to muscle tissue. Venous insufficiency presents as a disorder of venous blood reflux and does not present with cramp-type pain with exercise. Vasculitis is an inflammation of the blood vessels and presents with weakness, fever, and fatigue, but does not present with cramp-type pain with exercise. The pain associated with VTE does not have this clinical presentation.

A nurse is caring for a patient with cirrhosis secondary to heavy alcohol use. The nurse's most recent assessment reveals subtle changes in the patient's cognition and behavior. What is the nurse's most appropriate response? A) Ensure that the patient's sodium intake does not exceed recommended levels. B) Report this finding to the primary care provider due to the possibility of hepatic encephalopathy. C) Inform the primary care provider that the patient should be assessed for alcoholic hepatitis. D) Implement interventions aimed at ensuring a calm and therapeutic care environment.

Ans: B Feedback: Monitoring is an essential nursing function to identify early deterioration in mental status. The nurse monitors the patient's mental status closely and reports changes so that treatment of encephalopathy can be initiated promptly. This change in status is likely unrelated to sodium intake and would not signal the onset of hepatitis. A supportive care environment is beneficial, but does not address the patient's physiologic deterioration.

During an adult patient's last two office visits, the nurse obtained BP readings of 122/84 mm Hg and 130/88 mm Hg, respectively. How would this patient's BP be categorized? A) Normal B) Prehypertensive C) Stage 1 hypertensive D) Stage 2 hypertensive

Ans: B Feedback: Prehypertension is defined systolic BP of 120 to 139 mm Hg or diastolic BP of 80 to 89 mm Hg.

You are caring for a patient who is diagnosed with Raynaud's phenomenon. The nurse should plan interventions to address what nursing diagnosis? A) Chronic pain B) Ineffective tissue perfusion C) Impaired skin integrity D) Risk for injury

Ans: B Feedback: Raynaud's phenomenon is a form of intermittent arteriolar vasoconstriction resulting in inadequate tissue perfusion. This results in coldness, pain, and pallor of the fingertips or toes. Pain is typically intermittent and acute, not chronic, and skin integrity is rarely at risk. In most cases, the patient is not at a high risk for injury.

While auscultating a patient's heart sounds, the nurse hears an extra heart sound immediately after the second heart sound (S2). An audible S3 would be considered an expected finding in what patient? A) An older adult B) A 20-year-old patient C) A patient who has undergone valve replacement D) A patient who takes a beta-adrenergic blocker

Ans: B Feedback: S3 represents a normal finding in children and adults up to 35 or 40 years of age. In these cases, it is called a physiologic S3. It is an abnormal finding in a patient with an artificial valve, an older adult, or a patient who takes a beta blocker.

A patient has been diagnosed with advanced stage breast cancer and will soon begin aggressive treatment. What assessment findings would most strongly suggest that the patient may have developed liver metastases? A) Persistent fever and cognitive changes B) Abdominal pain and hepatomegaly C) Peripheral edema unresponsive to diuresis D) Spontaneous bleeding and jaundice

Ans: B Feedback: The early manifestations of malignancy of the liver include pain—a continuous dull ache in the right upper quadrant, epigastrium, or back. Weight loss, loss of strength, anorexia, and anemia may also occur. The liver may be enlarged and irregular on palpation. Jaundice is present only if the larger bile ducts are occluded by the pressure of malignant nodules in the hilum of the liver. Fever, cognitive changes, peripheral edema, and bleeding are atypical signs.

The nurse is planning the care of a patient who has been diagnosed with hypertension, but who otherwise enjoys good health. When assessing the response to an antihypertensive drug regimen, what blood pressure would be the goal of treatment? A) 156/96 mm Hg or lower B) 140/90 mm Hg or lower C) Average of 2 BP readings of 150/80 mm Hg D) 120/80 mm Hg or lower

Ans: B Feedback: The goal of antihypertensive drug therapy is a BP of 140/90 mm Hg or lower. A pressure of 130/80 mm Hg is the goal for patients with diabetes or chronic kidney disease.

A nurse has written a plan of care for a man diagnosed with peripheral arterial insufficiency. One of the nursing diagnoses in the care plan is altered peripheral tissue perfusion related to compromised circulation. What is the most appropriate intervention for this diagnosis? A) Elevate his legs and arms above his heart when resting. B) Encourage the patient to engage in a moderate amount of exercise. C) Encourage extended periods of sitting or standing. D) Discourage walking in order to limit pain.

Ans: B Feedback: The nursing diagnosis of altered peripheral tissue perfusion related to compromised circulation requires interventions that focus on improving circulation. Encouraging the patient to engage in a moderate amount of exercise serves to improve circulation. Elevating his legs and arms above his heart when resting would be passive and fails to promote circulation. Encouraging long periods of sitting or standing would further compromise circulation. The nurse should encourage, not discourage, walking to increase circulation and decrease pain.

A patient's medication regimen for the treatment of hypertension includes hydrochlorothiazide. Following administration of this medication, the nurse should anticipate what effect? A) Drowsiness or lethargy B) Increased urine output C) Decreased heart rate D) Mild agitation

Ans: B Feedback: Thiazide diuretics lower BP by reducing circulating blood volume; this results in a short-term increase in urine output. These drugs do not cause bradycardia, agitation, or drowsiness.

The physical therapist notifies the nurse that a patient with coronary artery disease (CAD) experiences a much greater-than-average increase in heart rate during physical therapy. The nurse recognizes that an increase in heart rate in a patient with CAD may result in what? A) Development of an atrial-septal defect B) Myocardial ischemia C) Formation of a pulmonary embolism D) Release of potassium ions from cardiac cells

Ans: B Feedback: Unlike other arteries, the coronary arteries are perfused during diastole. An increase in heart rate shortens diastole and can decrease myocardial perfusion. Patients, particularly those with CAD, can develop myocardial ischemia. An increase in heart rate will not usually result in a pulmonary embolism or create electrolyte imbalances. Atrial-septal defects are congenital.

A postsurgical patient has illuminated her call light to inform the nurse of a sudden onset of lower leg pain. On inspection, the nurse observes that the patient's left leg is visibly swollen and reddened. What is the nurse's most appropriate action? A) Administer a PRN dose of subcutaneous heparin. B) Inform the physician that the patient has signs and symptoms of VTE. C) Mobilize the patient promptly to dislodge any thrombi in the patient's lower leg. D) Massage the patient's lower leg to temporarily restore venous return.

Ans: B Feedback: VTE requires prompt medical follow-up. Heparin will not dissolve an established clot. Massaging the patient's leg and mobilizing the patient would be contraindicated because they would dislodge the clot, possibly resulting in a pulmonary embolism.

The nurse is caring for a patient who is admitted to the medical unit for the treatment of a venous ulcer in the area of her lateral malleolus that has been unresponsive to treatment. What is the nurse most likely to find during an assessment of this patient's wound? A) Hemorrhage B) Heavy exudate C) Deep wound bed D) Pale-colored wound bed

Ans: B Feedback: Venous ulcerations in the area of the medial or lateral malleolus (gaiter area) are typically large, superficial, and highly exudative. Venous hypertension causes extravasation of blood, which discolors the area of the wound bed. Bleeding is not normally present.

A 40-year-old male newly diagnosed with hypertension is discussing risk factors with the nurse. The nurse talks about lifestyle changes with the patient and advises that the patient should avoid tobacco use. What is the primary rationale behind that advice to the patient? A) Quitting smoking will cause the patient's hypertension to resolve. B) Tobacco use increases the patient's concurrent risk of heart disease. C) Tobacco use is associated with a sedentary lifestyle. D) Tobacco use causes ventricular hypertrophy.

Ans: B Feedback:Smoking increases the risk for heart disease, for which a patient with hypertension is already at an increased risk. Quitting will not necessarily cause hypertension to resolve and smoking does not directly cause ventricular hypertrophy. The association with a sedentary lifestyle is true, but this is not the main rationale for the nurse's advice; the association with heart disease is more salient.

The nurse is reviewing the medication administration record of a patient who takes a variety of medications for the treatment of hypertension. What potential therapeutic benefits of antihypertensives should the nurse identify? Select all that apply. A) Increased venous return B) Decreased peripheral resistance C) Decreased blood volume D) Decreased strength and rate of myocardial contractions E) Decreased blood viscosity

Ans: B, C, D Feedback: The medications used for treating hypertension decrease peripheral resistance, blood volume, or the strength and rate of myocardial contraction. Antihypertensive medications do not increase venous return or decrease blood viscosity.

A nurse has entered the room of a patient with cirrhosis and found the patient on the floor. The patient states that she fell when transferring to the commode. The patient's vital signs are within reference ranges and the nurse observes no apparent injuries. What is the nurse's most appropriate action? A) Remove the patient's commode and supply a bedpan. B) Complete an incident report and submit it to the unit supervisor. C) Have the patient assessed by the physician due to the risk of internal bleeding. D) Perform a focused abdominal assessment in order to rule out injury.

Ans: C Feedback: A fall would necessitate thorough medical assessment due to the patient's risk of bleeding. The nurse's abdominal assessment is an appropriate action, but is not wholly sufficient to rule out internal injury. Medical assessment is a priority over removing the commode or filling out an incident report, even though these actions are appropriate.

A 79-year-old man is admitted to the medical unit with digital gangrene. The man states that his problems first began when he stubbed his toe going to the bathroom in the dark. In addition to this trauma, the nurse should suspect that the patient has a history of what health problem? A) Raynaud's phenomenon B) CAD C) Arterial insufficiency D) Varicose veins

Ans: C Feedback: Arterial insufficiency may result in gangrene of the toe (digital gangrene), which usually is caused by trauma. The toe is stubbed and then turns black. Raynaud's, CAD and varicose veins are not the usual causes of digital gangrene in the elderly.

The triage nurse in the ED is assessing a patient who has presented with complaint of pain and swelling in her right lower leg. The patient's pain became much worse last night and appeared along with fever, chills, and sweating. The patient states, "I hit my leg on the car door 4 or 5 days ago and it has been sore ever since." The patient has a history of chronic venous insufficiency. What intervention should the nurse anticipate for this patient? A) Platelet transfusion to treat thrombocytopenia B) Warfarin to treat arterial insufficiency C) Antibiotics to treat cellulitis D) Heparin IV to treat VTE

Ans: C Feedback: Cellulitis is the most common infectious cause of limb swelling. The signs and symptoms include acute onset of swelling, localized redness, and pain; it is frequently associated with systemic signs of fever, chills, and sweating. The patient may be able to identify a trauma that accounts for the source of infection. Thrombocytopenia is a loss or decrease in platelets and increases a patient's risk of bleeding; this problem would not cause these symptoms. Arterial insufficiency would present with ongoing pain related to activity. This patient does not have signs and symptoms of VTE.

The nurse is assessing a patient new to the clinic. Records brought to the clinic with the patient show the patient has hypertension and that her current BP readings approximate the readings from when she was first diagnosed. What contributing factor should the nurse first explore in an effort to identify the cause of the client's inadequate BP control? A) Progressive target organ damage B) Possibility of medication interactions C) Lack of adherence to prescribed drug therapy D) Possible heavy alcohol use or use of recreational drugs

Ans: C Feedback: Deviation from the therapeutic program is a significant problem for people with hypertension and other chronic conditions requiring lifetime management. An estimated 50% of patients discontinue their medications within 1 year of beginning to take them. Consequently, this is a more likely problem than substance use, organ damage, or adverse drug interactions.

An older adult is newly diagnosed with primary hypertension and has just been started on a beta-blocker. The nurse's health education should include which of the following? A) Increasing fluids to avoid extracellular volume depletion from the diuretic effect of the beta-blocker B) Maintaining a diet high in dairy to increase protein necessary to prevent organ damage C) Use of strategies to prevent falls stemming from postural hypotension D) Limiting exercise to avoid injury that can be caused by increased intracranial pressure

Ans: C Feedback: Elderly people have impaired cardiovascular reflexes and are more sensitive to postural hypotension. The nurse teaches patients to change positions slowly when moving from lying or sitting positions to a standing position, and counsels elderly patients to use supportive devices as necessary to prevent falls that could result from dizziness. Lifestyle changes, such as regular physical activity/exercise, and a diet rich in fruits, vegetables, and low-fat dairy products, is strongly recommended. Increasing fluids in elderly patients may be contraindicated due to cardiovascular disease. Increased intracranial pressure is not a risk and activity should not normally be limited.

The nurse is providing care for a patient with a diagnosis of hypertension. The nurse should consequently assess the patient for signs and symptoms of which other health problem? A) Migraines B) Atrial-septal defect C) Atherosclerosis D) Thrombocytopenia

Ans: C Feedback: Hypertension is both a sign and a risk factor for atherosclerotic heart disease. It is not associated with structural cardiac defects, low platelet levels, or migraines.

A group of student nurses are practicing taking blood pressure. A 56-year-old male student has a blood pressure reading of 146/96 mm Hg. Upon hearing the reading, he exclaims, "My pressure has never been this high. Do you think my doctor will prescribe medication to reduce it?" Which of the following responses by the nursing instructor would be best? A) "Yes. Hypertension is prevalent among men; it is fortunate we caught this during your routine examination." B) "We will need to reevaluate your blood pressure because your age places you at high risk for hypertension." C) "A single elevated blood pressure does not confirm hypertension. You will need to have your blood pressure reassessed several times before a diagnosis can be made." D) "You have no need to worry. Your pressure is probably elevated because you are being tested."

Ans: C Feedback: Hypertension is confirmed by two or more readings with systolic pressure of at least 140 mm Hg and diastolic pressure of at least 90 mm Hg. An age of 56 does not constitute a risk factor in and of itself. The nurse should not tell the student that there is no need to worry.

The nurse is caring for an acutely ill patient who is on anticoagulant therapy. The patient has a comorbidity of renal insufficiency. How will this patient's renal status affect heparin therapy? A) Heparin is contraindicated in the treatment of this patient. B) Heparin may be administered subcutaneously, but not IV. C) Lower doses of heparin are required for this patient. D) Coumadin will be substituted for heparin.

Ans: C Feedback: If renal insufficiency exists, lower doses of heparin are required. Coumadin cannot be safely and effectively used as a substitute and there is no contraindication for IV administration.

The nurse is collaborating with the dietitian and a patient with hypertension to plan dietary modifications. These modifications should include which of the following? A) Reduced intake of protein and carbohydrates B) Increased intake of calcium and vitamin D C) Reduced intake of fat and sodium D) Increased intake of potassium, vitamin B12 and vitamin D

Ans: C Feedback: Lifestyle modifications usually include restricting sodium and fat intake, increasing intake of fruits and vegetables, and implementing regular physical activity. There is no need to increase calcium, potassium, and vitamin intake. Calorie restriction may be required for some patients, but a specific reduction in protein and carbohydrates is not normally indicated.

A medical nurse has admitted four patients over the course of a 12-hour shift. For which patient would assessment of ankle-brachial index (ABI) be most clearly warranted? A) A patient who has peripheral edema secondary to chronic heart failure B) An older adult patient who has a diagnosis of unstable angina C) A patient with poorly controlled type 1 diabetes who is a smoker D) A patient who has community-acquired pneumonia and a history of COPD

Ans: C Feedback: Nurses should perform a baseline ABI on any patient with decreased pulses or any patient 50 years of age or older with a history of diabetes or smoking. The other answers do not apply.

A nurse is caring for a patient with hepatic encephalopathy. The nurse's assessment reveals that the patient exhibits episodes of confusion, is difficult to arouse from sleep and has rigid extremities. Based on these clinical findings, the nurse should document what stage of hepatic encephalopathy? A) Stage 1 B) Stage 2 C) Stage 3 D) Stage 4

Ans: C Feedback: Patients in the third stage of hepatic encephalopathy exhibit the following symptoms: stuporous, difficult to arouse, sleeps most of the time, exhibits marked confusion, incoherent in speech, asterixis, increased deep tendon reflexes, rigidity of extremities, marked EEG abnormalities. Patients in stages 1 and 2 exhibit clinical symptoms that are not as advanced as found in stage 3, and patients in stage 4 are comatose. In stage 4, there is an absence of asterixis, absence of deep tendon reflexes, flaccidity of extremities, and EEG abnormalities.

How should the nurse best position a patient who has leg ulcers that are venous in origin? A) Keep the patient's legs flat and straight. B) Keep the patient's knees bent to 45-degree angle and supported with pillows. C) Elevate the patient's lower extremities. D) Dangle the patient's legs over the side of the bed.

Ans: C Feedback: Positioning of the legs depends on whether the ulcer is of arterial or venous origin. With venous insufficiency, dependent edema can be avoided by elevating the lower extremities. Dangling the patient's legs and applying pillows may further compromise venous return.

A nurse is caring for a patient with severe hemolytic jaundice. Laboratory tests show free bilirubin to be 24 mg/dL. For what complication is this patient at risk? A) Chronic jaundice B) Pigment stones in portal circulation C) Central nervous system damage D) Hepatomegaly

Ans: C Feedback: Prolonged jaundice, even if mild, predisposes to the formation of pigment stones in the gallbladder, and extremely severe jaundice (levels of free bilirubin exceeding 20 to 25 mg/dL) poses a risk for CNS damage. There are not specific risks of hepatomegaly or chronic jaundice resulting from high bilirubin.

A nurse is closely monitoring a patient who has recently been diagnosed with an abdominal aortic aneurysm. What assessment finding would signal an impending rupture of the patient's aneurysm? A) Sudden increase in blood pressure and a decrease in heart rate B) Cessation of pulsating in an aneurysm that has previously been pulsating visibly C) Sudden onset of severe back or abdominal pain D) New onset of hemoptysis

Ans: C Feedback: Signs of impending rupture include severe back or abdominal pain, which may be persistent or intermittent. Impending rupture is not typically signaled by increased blood pressure, bradycardia, cessation of pulsing, or hemoptysis.

The nurse is preparing to administer warfarin (Coumadin) to a client with deep vein thrombophlebitis (DVT). Which laboratory value would most clearly indicate that the patient's warfarin is at therapeutic levels? A) Partial thromboplastin time (PTT) within normal reference range B) Prothrombin time (PT) eight to ten times the control C) International normalized ratio (INR) between 2 and 3 D) Hematocrit of 32%

Ans: C Feedback: The INR is most often used to determine if warfarin is at a therapeutic level; an INR of 2 to 3 is considered therapeutic. Warfarin is also considered to be at therapeutic levels when the client's PT is 1.5 to 2 times the control. Higher values indicate increased risk of bleeding and hemorrhage, whereas lower values indicate increased risk of blood clot formation. Heparin, not warfarin, prolongs PTT. Hematocrit does not provide information on the effectiveness of warfarin; however, a falling hematocrit in a client taking warfarin may be a sign of hemorrhage.

The critical care nurse is caring for a patient just admitted in a hypertensive emergency. The nurse should anticipate the administration of what medication? A) Warfarin (Coumadin) B) Furosemide (Lasix) C) Sodium nitroprusside (Nitropress) D) Ramipril (Altace)

Ans: C Feedback: The medications of choice in hypertensive emergencies are those that have an immediate effect. IV vasodilators, including sodium nitroprusside (Nitropress), nicardipine hydrochloride (Cardene), clevidipine (Cleviprex), fenoldopam mesylate (Corlopam), enalaprilat, and nitroglycerin, have immediate actions that are short lived (minutes to 4 hours), and they are therefore used for initial treatment. Ramipril is administered orally and would not meet the patient's immediate need for BP management. Diuretics, such as Lasix, are not used as initial treatments and there is no indication for anticoagulants such as Coumadin.

A patient presents to the clinic complaining of the inability to grasp objects with her right hand. The patient's right arm is cool and has a difference in blood pressure of more than 20 mm Hg compared with her left arm. The nurse should expect that the primary care provider may diagnose the woman with what health problem? A) Lymphedema B) Raynaud's phenomenon C) Upper extremity arterial occlusive disease D) Upper extremity VTE

Ans: C Feedback: The patient with upper extremity arterial occlusive disease typically complains of arm fatigue and pain with exercise (forearm claudication) and inability to hold or grasp objects (e.g., combing hair, placing objects on shelves above the head) and, occasionally, difficulty driving. Assessment findings include coolness and pallor of the affected extremity, decreased capillary refill, and a difference in arm blood pressures of more than 20 mm Hg. These symptoms are not closely associated with Raynaud's or lymphedema. The upper extremities are rare sites for VTE.

A newly diagnosed patient with hypertension is prescribed Diuril, a thiazide diuretic. What patient education should the nurse provide to this patient? A) "Eat a banana every day because Diuril causes moderate hyperkalemia." B) "Take over-the-counter potassium pills because Diuril causes your kidneys to lose potassium." C) "Diuril can cause low blood pressure and dizziness, especially when you get up suddenly." D) "Diuril increases sodium levels in your blood, so cut down on your salt."

Ans: C Feedback: Thiazide diuretics can cause postural hypotension, which may be potentiated by alcohol, barbiturates, opioids, or hot weather. Diuril does not cause either moderate hyperkalemia or severe hypokalemia and it does not result in hypernatremia.

A nurse is admitting a 45-year-old man to the medical unit who has a history of PAD. While providing his health history, the patient reveals that he smokes about two packs of cigarettes a day, has a history of alcohol abuse, and does not exercise. What would be the priority health education for this patient? A) The lack of exercise, which is the main cause of PAD. B) The likelihood that heavy alcohol intake is a significant risk factor for PAD. C) Cigarettes contain nicotine, which is a powerful vasoconstrictor and may cause or aggravate PAD. D) Alcohol suppresses the immune system, creates high glucose levels, and may cause PAD.

Ans: C Feedback: Tobacco is powerful vasoconstrictor; its use with PAD is highly detrimental, and patients are strongly advised to stop using tobacco. Sedentary lifestyle is also a risk factor, but smoking is likely a more significant risk factor that the nurse should address. Alcohol use is less likely to cause PAD, although it carries numerous health risks.

Graduated compression stockings have been prescribed to treat a patient's venous insufficiency. What education should the nurse prioritize when introducing this intervention to the patient? A) The need to take anticoagulants concurrent with using compression stockings B) The need to wear the stockings on a "one day on, one day off" schedule C) The importance of wearing the stockings around the clock to ensure maximum benefit D) The importance of ensuring the stockings are applied evenly with no pressure points

Ans: D Feedback: Any type of stocking can inadvertently become a tourniquet if applied incorrectly (i.e., rolled tightly at the top). In such instances, the stockings produce rather than prevent stasis. For ambulatory patients, graduated compression stockings are removed at night and reapplied before the legs are lowered from the bed to the floor in the morning. They are used daily, not on alternating days. Anticoagulants are not always indicated in patients who are using compression stockings.

A patient with esophageal varices is being cared for in the ICU. The varices have begun to bleed and the patient is at risk for hypovolemia. The patient has Ringer's lactate at 150 cc/hr infusing. What else might the nurse expect to have ordered to maintain volume for this patient? A) Arterial line B) Diuretics C) Foley catheter D) Volume expanders

Ans: D Feedback: Because patients with bleeding esophageal varices have intravascular volume depletion and are subject to electrolyte imbalance, IV fluids with electrolytes and volume expanders are provided to restore fluid volume and replace electrolytes. Diuretics would reduce vascular volume. An arterial line and Foley catheter are likely to be ordered, but neither actively maintains the patient's volume.

A nurse is caring for a patient with liver failure and is performing an assessment in the knowledge of the patient's increased risk of bleeding. The nurse recognizes that this risk is related to the patient's inability to synthesize prothrombin in the liver. What factor most likely contributes to this loss of function? A) Alterations in glucose metabolism B) Retention of bile salts C) Inadequate production of albumin by hepatocytes D) Inability of the liver to use vitamin K

Ans: D Feedback: Decreased production of several clotting factors may be partially due to deficient absorption of vitamin K from the GI tract. This probably is caused by the inability of liver cells to use vitamin K to make prothrombin. This bleeding risk is unrelated to the roles of glucose, bile salts, or albumin.

A local public health nurse is informed that a cook in a local restaurant has been diagnosed with hepatitis A. What should the nurse advise individuals to obtain who ate at this restaurant and have never received the hepatitis A vaccine? A) The hepatitis A vaccine B) Albumin infusion C) The hepatitis A and B vaccines D) An immune globulin injection

Ans: D Feedback: For people who have not been previously vaccinated, hepatitis A can be prevented by the intramuscular administration of immune globulin during the incubation period, if given within 2 weeks of exposure. Administration of the hepatitis A vaccine will not protect the patient exposed to hepatitis A, as protection will take a few weeks to develop after the first dose of the vaccine. The hepatitis B vaccine provides protection again the hepatitis B virus, but plays no role in protection for the patient exposed to hepatitis A. Albumin confers no therapeutic benefit.

A community health nurse teaching a group of adults about preventing and treating hypertension. The nurse should encourage these participants to collaborate with their primary care providers and regularly monitor which of the following? A) Heart rate B) Sodium levels C) Potassium levels D) Blood lipid levels

Ans: D Feedback: Hypertension often accompanies other risk factors for atherosclerotic heart disease, such as dyslipidemia (abnormal blood fat levels), obesity, diabetes, metabolic syndrome, and a sedentary lifestyle. Individuals with hypertension need to monitor their sodium intake, but hypernatremia is not a risk factor for hypertension. In many patients, heart rate does not correlate closely with BP. Potassium levels do not normally relate to BP.

A 55-year-old patient comes to the clinic for a routine check-up. The patient's BP is 159/100 mm Hg and the physician diagnoses hypertension after referring to previous readings. The patient asks why it is important to treat hypertension. What would be the nurse's best response? A) "Hypertension can cause you to develop dangerous blood clots in your legs that can migrate to your lungs." B) "Hypertension puts you at increased risk of type 1 diabetes and cancer in your age group." C) "Hypertension is the leading cause of death in people your age." D) "Hypertension greatly increases your risk of stroke and heart disease."

Ans: D Feedback: Hypertension, particularly elevated systolic BP, increases the risk of death, stroke, and heart failure in people older than 50 years. Hypertension is not a direct precursor to pulmonary emboli, and it does not put older adults at increased risk of type 1 diabetes or cancer. It is not the leading cause of death in people 55 years of age.

A patient in hypertensive urgency is admitted to the hospital. The nurse should be aware of what goal of treatment for a patient in hypertensive urgency? A) Normalizing BP within 2 hours B) Obtaining a BP of less than 110/70 mm Hg within 36 hours C) Obtaining a BP of less than 120/80 mm Hg within 36 hours D) Normalizing BP within 24 to 48 hours

Ans: D Feedback: In cases of hypertensive urgency, oral agents can be administered with the goal of normalizing BP within 24 to 48 hours. For patients with this health problem, a BP of 120/80 mm Hg may be unrealistic.

A nurse working in a long-term care facility is performing the admission assessment of a newly admitted, 85-year-old resident. During inspection of the resident's feet, the nurse notes that she appears to have early evidence of gangrene on one of her great toes. The nurse knows that gangrene in the elderly is often the first sign of what? A) Chronic venous insufficiency B) Raynaud's phenomenon C) VTE D) PAD

Ans: D Feedback: In elderly people, symptoms of PAD may be more pronounced than in younger people. In elderly patients who are inactive, gangrene may be the first sign of disease. Venous insufficiency does not normally manifest with gangrene. Similarly, VTE and Raynaud's phenomenon do not cause the ischemia that underlies gangrene.

The nurse is screening a number of adults for hypertension. What range of blood pressure is considered normal? A) Less than 140/90 mm Hg B) Less than 130/90 mm Hg C) Less than 129/89 mm Hg D) Less than 120/80 mm Hg

Ans: D Feedback: JNC 7 defines a blood pressure of less than 120/80 mm Hg as normal, 120 to 129/80 to 89 mm Hg as prehypertension, and 140/90 mm Hg or higher as hypertension.

A patient comes to the walk-in clinic complaining of frequent headaches. While assessing the patient's vital signs, the nurse notes the BP is 161/101 mm Hg. According to JNC 7, how would this patient's BP be defined if a similar reading were obtained at a subsequent office visit? A) High normal B) Normal C) Stage 1 hypertensive D) Stage 2 hypertensive

Ans: D Feedback: JNC 7 defines stage 2 hypertension as a reading 160/100 mm Hg.

A patient's recently elevated BP has prompted the primary care provider to prescribe furosemide (Lasix). The nurse should closely monitor which of the following? A) The client's oxygen saturation level B) The patient's red blood cells, hematocrit, and hemoglobin C) The patient's level of consciousness D) The patient's potassium level

Ans: D Feedback: Loop diuretics can cause potassium depletion. They do not normally affect level of consciousness, erythrocytes, or oxygen saturation.

A patient comes to the walk-in clinic with complaints of pain in his foot following stepping on a roofing nail 4 days ago. The patient has a visible red streak running up his foot and ankle. What health problem should the nurse suspect? A) Cellulitis B) Local inflammation C) Elephantiasis D) Lymphangitis

Ans: D Feedback: Lymphangitis is an acute inflammation of the lymphatic channels. It arises most commonly from a focus of infection in an extremity. Usually, the infectious organism is hemolytic streptococcus. The characteristic red streaks that extend up the arm or the leg from an infected wound outline the course of the lymphatic vessels as they drain. Cellulitis is caused by bacteria, which cause a generalized edema in the subcutaneous tissues surrounding the affected area. Local inflammation would not present with red streaks in the lymphatic channels. Elephantiasis is transmitted by mosquitoes that carry parasitic worm larvae; the parasites obstruct the lymphatic channels and results in gross enlargement of the limbs.

The nurse caring for a patient with a leg ulcer has finished assessing the patient and is developing a problem list prior to writing a plan of care. What major nursing diagnosis might the care plan include? A) Risk for disuse syndrome B) Ineffective health maintenance C) Sedentary lifestyle D) Imbalanced nutrition: less than body requirements

Ans: D Feedback: Major nursing diagnoses for the patient with leg ulcers may include imbalanced nutrition: less than body requirements, related to increased need for nutrients that promote wound healing. Risk for disuse syndrome is a state in which an individual is at risk for deterioration of body systems owing to prescribed or unavoidable musculoskeletal inactivity. A leg ulcer will affect activity, but rarely to this degree. Leg ulcers are not necessarily a consequence of ineffective health maintenance or sedentary lifestyle.

A nurse is participating in the emergency care of a patient who has just developed variceal bleeding. What intervention should the nurse anticipate? A) Infusion of intravenous heparin B) IV administration of albumin C) STAT administration of vitamin K by the intramuscular route D) IV administration of octreotide (Sandostatin)

Ans: D Feedback: Octreotide (Sandostatin)—a synthetic analog of the hormone somatostatin—is effective in decreasing bleeding from esophageal varices, and lacks the vasoconstrictive effects of vasopressin. Because of this safety and efficacy profile, octreotide is considered the preferred treatment regimen for immediate control of variceal bleeding. Vitamin K and albumin are not administered and heparin would exacerbate, not alleviate, bleeding.

The nurse is assessing a woman who is pregnant at 27 weeks' gestation. The patient is concerned about the recent emergence of varicose veins on the backs of her calves. What is the nurse's best response? A) Facilitate a referral to a vascular surgeon. B) Assess the patient's ankle-brachial index (ABI) and perform Doppler ultrasound testing. C) Encourage the patient to increase her activity level. D) Teach the patient that circulatory changes during pregnancy frequently cause varicose veins.

Ans: D Feedback: Pregnancy may cause varicosities because of hormonal effects related to decreased venous outflow, increased pressure by the gravid uterus, and increased blood volume. In most cases, no intervention or referral is necessary. This finding is not an indication for ABI assessment and increased activity will not likely resolve the problem.

The nurse is caring for a patient who is undergoing an exercise stress test. Prior to reaching the target heart rate, the patient develops chest pain. What is the nurse's most appropriate response? A) Administer sublingual nitroglycerin to allow the patient to finish the test. B) Initiate cardiopulmonary resuscitation. C) Administer analgesia and slow the test. D) Stop the test and monitor the patient closely.

Ans: D Feedback: Signs of myocardial ischemia would necessitate stopping the test. CPR would only be necessary if signs of cardiac or respiratory arrest were evident.

The home health nurse is caring for a patient who has a comorbidity of hypertension. What assessment question most directly addresses the possibility of worsening hypertension? A) "Are you eating less salt in your diet?" B) "How is your energy level these days?" C) "Do you ever get chest pain when you exercise?" D) "Do you ever see spots in front of your eyes?"

Ans: D Feedback: To identify complications or worsening hypertension, the patient is questioned about blurred vision, spots in front of the eyes, and diminished visual acuity. The heart, nervous system, and kidneys are also carefully assessed, but angina pain and decreased energy are not normally suggestive of worsening hypertension. Sodium limitation is a beneficial lifestyle modification, but nonadherence to this is not necessarily a sign of worsening symptoms.

A nurse is providing discharge teaching instructions for a patient taking cholestyramine (Questran). Which statement made by the patient demonstrates a need for further teaching? a. "I will take warfarin (Coumadin) 1 hour before my medicine." b. "I will increase the fluids and fiber in my diet." c. "I can take cholestyramine with my hydrochlorothiazide." d. "I will take digoxin 4 hours after taking the cholestyramine."

C Drugs known to form complexes with the sequestrants include thiazide diuretics, such as hydrochlorothiazide, digoxin, warfarin, and some antibiotics. To reduce the formation of sequestrant-drug complexes, oral medication should be administered either 1 hour before the sequestrant or 4 hours after. Further teaching is needed. Warfarin should be taken 1 hour before or 4 hours after cholestyramine. Adverse effects of Questran are limited to the gastrointestinal (GI) tract. Constipation, the principal complaint, can be minimized by increasing dietary fiber and fluids. Digoxin should be taken 1 hour before or 4 hours after cholestyramine.

A nurse in the rehabilitation unit is caring for an older adult patient who is in cardiac rehabilitation following an MI. The nurse's plan of care calls for the patient to walk for 10 minutes 3 times a day. The patient questions the relationship between walking and heart function. How should the nurse best reply? A) "The arteries in your legs constrict when you walk and allow the blood to move faster and with more pressure on the tissue." B) Walking increases your heart rate and blood pressure. Therefore your heart is under less stress." C) "Walking helps your heart adjust to your new arteries and helps build your self-esteem." D) "When you walk, the muscles in your legs contract and pump the blood in your veins back toward your heart, which allows more blood to return to your heart."

Ans: D Feedback: Veins, unlike arteries, are equipped with valves that allow blood to move against the force of gravity. The legs have one-way bicuspid valves that prevent blood from seeping backward as it moves forward by the muscles in our legs pressing on the veins as we walk and increasing venous return. Leg arteries do constrict when walking, which allows the blood to move faster and with more pressure on the tissue, but the greater concern is increasing the flow of venous blood to the heart. Walking increases, not decreases, the heart' pumping ability, which increases heart rate and blood pressure and the hearts ability to manage stress. Walking does help the heart adjust to new arteries and may enhance self-esteem, but the patient had an MI—there are no "new arteries."

A nurse is assessing a new patient who is diagnosed with PAD. The nurse cannot feel the pulse in the patient's left foot. How should the nurse proceed with assessment? A) Have the primary care provider order a CT. B) Apply a tourniquet for 3 to 5 minutes and then reassess. C) Elevate the extremity and attempt to palpate the pulses. D) Use Doppler ultrasound to identify the pulses.

Ans: D Feedback: When pulses cannot be reliably palpated, a hand-held continuous wave (CW) Doppler ultrasound device may be used to hear (insonate) the blood flow in vessels. CT is not normally warranted and the application of a tourniquet poses health risks and will not aid assessment. Elevating the extremity would make palpation more difficult.

In hepatitis A, the presence of Anti-HAV (antibody to HAV) immunoglobulin M (IgM)

Anti-HAV (antibody to HAV) immunoglobulin M (IgM) appears during the acute phase. Detection of hepatitis A IgM indicates acute hepatitis

Hepatitis C test

Anti-HCV (antibody to HCV) Marker for acute or chronic infection with HCV HCV RNA quantitation Indicates active ongoing viral replication HCV genotyping Indicates the genotype of HCV

D

Anti-HDV Present in past or current infection with HDV HDV Ag (hepatitis D antigen) Present within a few days after infection

HEV

Anti-HEV IgM and IgG Present 1 wk-2 mo after illness onset HEV RNA quantitation Indicates active ongoing viral replication

E

Anti-HEV IgM and IgG Present 1 wk-2 mo after illness onset HEV RNA quantitation Indicates active ongoing viral replication

What drug may be given post surgery for PVD? Why?

Anticoagulant to prevent thrombosis antiplateles

Drug Therapy VTE

Anticoagulants are used routinely for VTE prevention and treatment. The regimen depends on the patient's VTE risk. The goal of anticoagulant therapy for VTE prevention is to prevent clot formation. The goals for treatment of a confirmed VTE are to prevent new clot development, spread of the clot, and embolization. Anticoagulant therapy does not dissolve the clot. Clot lysis begins naturally through the body's intrinsic fibrinolytic system

Atorvastatin (Lipitor)

Antihyperlipidemic HMG CoA Reductase Inhibitor ("Statin") MOA: Block synthesis of cholesterol and increase LDL receptors in liver ↓ LDL ↓ Triglycerides ↑ HDL (small amount) S/E: Rash, GI problems, high liver enzymes, myopathy, rhabdomyolysis Well tolerated with few side effects. Monitor liver enzymes and creatine kinase (if muscle weakness or pain occurs).

Clopidogrel (Plavix)

Antiplatelet Agent Drug Alert Clopidogrel (Plavix) and Omeprazole (Prilosec) • Antiplatelet effect of clopidogrel is reduced by about half when given with omeprazole. • This reduced effect increases the risk for myocardial infarction (MI) and stroke.

Antiplatelet agents PAD

Aspirin Clopidogrel (Plavix)

Acute Intervention Hep

Assess for the presence and degree of jaundice.oIn light-skinned persons, usually observed first in the sclera of the eyes and later in the skin. In dark-skinned persons, observed in the hard palate of the mouth and inner canthus of the eyes. Urine may have a dark brown or brownish red color bc of bilirubin excretion from the kidneys Comfort measures to relieve pruritus (if present), headache, and arthralgias are helpfu

Thrombolytic Agents Implementation

Avoid invasive procedures, such as IM injections or arterial punctures, with this therapy Acetaminophen may be ordered to control fever. IV Administration: review process.

A nurse is reviewing a patient's medications and realizes that gemfibrozil (Lopid) and warfarin (Coumadin) are to be administered concomitantly. This finding concerns the nurse, who is aware that the _____ will be _____. a. level of gemfibrozil; increased b. level of gemfibrozil; reduced c. anticoagulant effects; increased d. anticoagulant effects; reduced

C Gemfibrozil displaces warfarin from the plasma albumin, thereby increasing anticoagulant effects. The level of gemfibrozil will not be increased or decreased. The anticoagulation effects will not be reduced, because free-floating drug is present in the system; the dosage of warfarin may need to be reduced.

After receiving report, which patient admitted to the emergency department should the nurse assess first? a. 67-year-old who has a gangrenous left foot ulcer with a weak pedal pulse b. 58-year-old who is taking anticoagulants for atrial fibrillation and has black stools c. 50-year-old who is complaining of sudden "sharp" and "worst ever" upper back pain d. 39-year-old who has right calf tenderness, redness, and swelling after a long plane ride

C The patient's presentation is consistent with dissecting thoracic aneurysm, which will require rapid intervention. The other patients do not need urgent interventions.

A patient has begun taking an HMG-COA reductase inhibitor. Which statement about this class of drugs made by the nurse during patient education would be inappropriate? a. "Statins reduce the risk of stroke." b. "You should come into the clinic for liver enzymes in 1 month." c. "Statins reduce the risk of coronary events in people with normal LDL levels." d. "You should maintain a healthy lifestyle and avoid high-fat foods."

B Baseline liver enzyme tests should be done before a patient starts taking an HMG-COA reductase inhibitor. They should be measured again in 6 to 12 months unless the patient has poor liver function, in which case the tests are indicated every 3 months. Statins do reduce the risk of stroke and coronary events in people with normal LDL levels. Maintaining a healthy lifestyle is important, as is avoiding high-fat foods.

A patient who recently started therapy with an HMG-COA reductase inhibitor asks the nurse, "How long will it take until I see an effect on my LDL cholesterol?" The nurse gives which correct answer? a. "At least 6 months is required to see a change." b. "A reduction usually is seen within 2 weeks." c. "Blood levels normalize immediately after the drug is started." d. "Cholesterol will not be affected, but triglycerides will fall within the first week."

B Reductions in LDL cholesterol are significant within 2 weeks and maximal within 4 to 6 weeks. It does not take 6 months too see a change. The blood level of LDL cholesterol is not reduced immediately upon starting the drug; a reduction is seen within 2 weeks. Blood cholesterol is affected, specifically LDL cholesterol, not triglycerides.

Elevated serum lipids Hypertension Tobacco use Physical inactivity

CAD risks An elevated serum lipid level is one of the four most firmly established risk factors for CAD. The risk of CAD is associated with a serum cholesterol level greater than 200 mg/dL (5.2mmol/L) or a fasting triglyceride level greater than 150 mg/dL (3.7 mmol/L). {Discuss the differences between HDL, LDL, and VLDL} The second major risk factor in CAD is hypertension, which is defined as a BP > 140/90 mm Hg or >130/80 mm Hg if the patient has diabetes or chronic kidney disease. In postmenopausal women, hypertension is associated with a higher incidence of CAD than in men and premenopausal women. Hypertension increases the risk of death from CAD 10-fold in all persons. The third major risk factor is tobacco use. The risk of developing CAD is 2 to 6 times higher in those who smoke tobacco or use smokeless tobacco than in those who do not. Further, tobacco smoking decreases estrogen levels, placing premenopausal women at greater risk for CAD. Risk is proportionate to the number of cigarettes smoked. The fourth major risk factor is physical inactivity. Physical inactivity implies lack of adequate physical exercise on a regular basis.

Telangiectasias:

Capillary Spider/Spider Angioma Telangiectasias are permanently dilated, small blood vessels consisting of venules, capillaries, or arterioles. How do you tell the difference between a capillary spider and a spider angioma? Capillary spiders are little masses of venules. When you blanch them, they will refill in an erratic, not-at-all-organized way. Spider angiomas are arterial. Blanch the center and they will refill in a very organized way, from the center out and evenly in all directions.

aortic dissection

Cause: HTN and atherosclerosis of vaso vasorum Presents with tearing chest pain to back Type A: ascending and urgent B: descending and B-blockers Complications: pericaridal tamponade or renal failure diagnosis in which the arterial wall splits apart

A 46-year-old is diagnosed with thromboangiitis obliterans (Buergers disease). When the nurse is developing a discharge teaching plan for the patient, which outcome has the highest priority for this patient? ​

Cessation of all tobacco use

What can we teach the patient with PVD to prevent further injury?

Change position frequently; wear nonrestrictive clothing; avoid crossing legs; avoid keeping legs dependent; wear good shoes; proper foot and nail care

Chronic Venous Insufficiency and Venous Leg Ulcers

Chronic venous insufficiency (CVI) describes abnormalities of the venous system that result in advanced signs and symptoms, such as edema, skin changes, and/or venous leg ulcers.22 CVI can lead to venous leg ulcers (formerly called venous stasis ulcers or varicose ulcers). Although CVI and venous leg ulcers are not life-threatening diseases, they are painful, slow to heal, debilitating, and costly conditions that adversely affect patients' quality of life. They are a common problem in older adults.

Drugs prescribed for treatment of intermittent claudication

Cilostazol (Pletal) Inhibits platelet aggregation ↑ vasodilation Pentoxifylline (Trental) Improves flexibilty of RBCs and WBCs ↓ fibrinogen concentration, platelet adhesiveness, and blood viscosity

The nurse reviews the admission orders shown in the accompanying figure for a patient newly diagnosed with peripheral artery disease. Which admission order should the nurse question?

Combined clopidogrel and omeprazole therapy reduce the antiplatelet effect of clopidogrel, and concomitant use should be avoided.

Vitamin K Antagonists (VKA) warfarin (Coumadin) PO

Considerations INR used to monitor therapeutic levels. Give at the same time each day. Variations of certain genes (e.g., CYP2CP, VKORC1) may influence response to drug. Antidote: Vitamin K. For VKA-related bleeding, treatment with prothrombin complex concentrate (human) (Kcentra), IV vitamin K and/or fresh frozen plasma is recommended.

Unfractionated Heparin (UH) heparin sodium

Continuous IV Intermittent IV Subcutaneous Therapeutic effects measured at regular intervals by the aPTT or ACT. Monitor CBC counts at regular intervals and titrate according to parameters. Follow administration guidelines for LMWH if giving subcutaneously. Antidote: Protamine neutralizes the effect of UH.

In early onset of hepatitis what systemic side effects can occur

Cpmpliment activation rash, angioedema, arthritis, fever, and malaise. Cryoglobulinemia (abnormal proteins found in the blood), glomerulonephritis, vasculitis, and involvement of other organs can occur from immune complex activation

Critical limb ischemia (CLI)

Critical limb ischemia (CLI) is a condition characterized by chronic ischemic rest pain lasting more than 2 weeks, nonhealing arterial leg ulcers, or gangrene of the leg from PAD. Patients with PAD who have diabetes, heart failure (HF), and a history of a stroke are at increased risk for CLI.

When planning care for a pt with cirrhosis, the nurse will give highest priority to which nursing diagnosis? a. impaired skin integrity related to edema, ascites, and pruritis b. imbalanced nutrition: less than body requirements related to anorexia c. excess fluid volume related to portal hypertension and hyperaldosteronism d. ineffective breathing pattern related to pressure on diaphragm and reduced lung volume

D airway and breathing are always highest priority.

The health care provider orders lactulose for a patient with hepatic encephalopathy. The nurse will monitor for effectiveness of this medication for this pt by assessing what? a. relief of constipation b. relief of ab pain c. decreased liver enzymes d. decreased ammonia levels

D hepatic encephalopathy is associated with elevated ammonia levels. Lactulose traps ammonia in the intestinal tract. It's laxative effect then expels ammonia from the colon, resulting in decreased ammonia levels, correcting hepatic encephalopathy.

A nurse is providing patient education about colesevelam (Welchol), a bile acid sequestrant. Which statement made by the patient demonstrates a need for further teaching? a. "Colesevelam will reduce my levels of low-density lipoprotein." b. "Colesevelam will augment my statin drug therapy." c. "I will not have to worry about having as many drug interactions as I did when I took cholestyramine." d. "I cannot take digoxin while on this medication."

D Digoxin can be taken 1 hour before or 4 hours after taking a bile acid sequestrant. Colesevelam reduces the LDL cholesterol level, which is one of its therapeutic uses. Colesevelam augments statin therapy. Colesevelam does not significantly interact with or reduce the absorption of statins, digoxin, warfarin, or most other drugs.

A nurse is instructing a patient receiving a cholesterol-lowering agent. Which information should the nurse include in the patient education? a. "This medication will replace other interventions you have been trying." b. "It is important for you to double your dose if you miss one to maintain therapeutic blood levels." c. "Stop taking the medication if you experience constipation." d. "You should continue your exercise program to increase your HDL serum levels."

D Regular exercise can reduce LDL cholesterol and elevate high-density lipoprotein (HDL) cholesterol, thereby reducing the risk of coronary heart disease (CHD). The patient should consider the cholesterol-lowering drug an adjunct to a proper diet and exercise. Drug therapy cannot replace other important interventions, such as diet and exercise. The patient should never be instructed to double the dose. Constipation is a side effect of most cholesterol-lowering agents. The patient should be encouraged to eat a high-fiber diet and increase fluids if not contraindicated.

A patient taking gemfibrozil (Lopid) and rosuvastatin (Crestor) concurrently begins to complain of muscle aches, fatigue, and weakness. What should the nurse monitor? a. For tendon tenderness b. For a lupuslike syndrome c. The patient's LFT results d. The patient's creatinine kinase levels

D Creatinine kinase levels are the best laboratory indicator of myopathy and/or rhabdomyolysis, which may lead to renal failure. As can the statins, gemfibrozil and other fibrates can cause myopathy. Fibrates must be used with caution in patients taking statins. Concurrent use of gemfibrozil and rosuvastatin does not cause tendon tenderness or a lupuslike syndrome. Liver function levels should be determined at the start of statin therapy and every 6 months thereafter in patients who do not have liver disease.

Endothelial Damage

Damage to the endothelium of the vein may be caused by direct (e.g., surgery, intravascular catheterization, trauma, burns, prior VTE) or indirect (chemotherapy, diabetes, sepsis) injury. Damaged endothelium stimulates platelet activation and starts the coagulation cascade. This predisposes the patient to thrombus development.

Which nursing diagnosis is not appropriate for a client with cirrhosis? Decreased fluid volume Diminished protection Impaired nutrition Impaired skin integrity

Decreased fluid volume Appropriate nursing diagnoses for a client with cirrhosis include impaired skin integrity, diminished protection and impaired nutrition. Increased, not decreased, fluid volume is appropriate for a client with cirrhosis.

Thrombolytic Agents Assessment Acute Ischemic Stroke:Assess neurologic status.

Determine time of onset of stroke symptoms. Alteplase must be administered within 3 hr of onset.

Renin Inhibitors Aliskiren hemifumarate (Tekturna)

Directly inhibits renin, thus reducing the conversion of angiotensinogen to angiotensin I May cause angioedema of the face, extremities, lips, tongue, glottis, and/or larynx Not to be used in pregnancy

Clinical Manifestations and Complications VV

Discomfort from varicose veins varies among people and tends to be worse after episodes of superficial vein thrombosis. Symptoms affect women more often than men. The most common symptoms include a heavy, achy feeling or pain after prolonged standing or sitting, which is relieved by walking or limb elevation. Some patients feel pressure or an itchy, burning, tingling, throbbing, or cramp-like leg sensation. Swelling, restless or tired legs, fatigue, and nocturnal leg cramps may occur. Superficial venous thrombosis is the most frequent complication of varicose veins. It may occur spontaneously or after trauma, surgical procedures, or pregnancy. Rare complications include rupture of the varicose veins resulting in external bleeding and skin ulcerations.

bifurcated

Divided into two

Anticoagulant Agents Direct Thrombin Inhibitors bivalirudin (Angiomax) argatroban

Drug TherapyChronic Stable Angina and Acute Coronary Syndrome • Direct inhibition of the clotting factor thrombin • Used during PCI procedure

Omega-3 fatty acids (fish oil)

Effective for reducing cholesterol and triglyceride levels

A 46-year-old service-counter worker undergoes sclerotherapy for treatment of superficial varicose veins at an outpatient center. Which instructions should the nurse provide to the patient before discharge?

Elastic compression stockings should be applied before getting out of bed.

Possible Diagnostic Findings

Elevated liver enzyme levels. ↑ Serum total bilirubin, hypoalbuminemia, anemia, bilirubin in urine and increased urobilinogen, prolonged PT time, positive tests for hepatitis, including anti-HAV IgM, HBsAg, anti-HBs, HBeAg, anti-HBe, anti-HBc IgM and IgG, HBV DNA quantitation, anti-HCV, HCV RNA quantitation, anti-HDV, HDV Ag. Abnormal liver scan, abnormal results on liver biopsy

Modifiable risk factors CAD

Elevated serum lipids Hypertension Tobacco use Physical inactivity Obesity Diabetes Metabolic syndrome Psychologic states Homocysteine level Substance abuse

What color changes will be observed with elevation and dependent position when PVD is arterial in origin?

Elevation - pallor; Dependent - Rubor

Name 5 surgical interventions for arterial PVD.

Embolectomy, Endarterectomy, Aterial bypass, Percutaneous transluminal angioplasty, Amputation

Explain nursing care of the patient with PVD in regards to activity level, positioning, clothing, and promoting warmth of the extremity.

Encourage rest at first sign of pain, schedule activities in patient's tolerance level, encourage elevation of extremity if venous and frequent position change, avoid restrictive clothing and crossing legs, keep warm with extra clothing not heating pads

Risk Factors for VTE

Endothelial Damage • Abdominal and pelvic surgery (e.g., gynecologic, urologic surgery) • Caustic or hypertonic IV drugs • Pelvis, hip, or leg fractures • History of VTE • Indwelling, peripherally inserted central vein catheter • IV drug abuse • Trauma

Intermittent pneumatic compression devices (IPCs) or SCDs

Ensure correct fit of IPCs by accurately measuring the extremities. IPCs will not be effective if they are not applied correctly, if the fit is incorrect, or if the patient does not wear the device continuously while at rest. The IPCS can be removed for bathing, skin assessment, and ambulation. IPCs are not worn when a patient has an active VTE because of the risk for PE.

are a complex of tortuous veins at the lower end of the esophagus, which are enlarged and swollen as a result of portal HTN

Esophageal varices

Esophageal varices

Esophageal varices are dilated, tortuous veins occurring in the lower part of the esophagus because of portal hypertension. Esophageal varices are a common complication of liver cirrhosis. Ruptured esophageal varices are the most life-threatening complication of cirrhosis and considered a medical emergency. Look for signs of melena and hematemesis.

Esophageal varices

Esophageal varices are responsible for approximately 80% of variceal hemorrhages Bleeding esophageal varices are the most life-threatening complication of cirrhosis Pt may have melena or hematemesis. There may be slow oozing or massive hemorrhage Massive hemorrhage is a medical emergency. Ruptured esophageal varices are the most life-threatening complication

Varicose veins etiology

Etiology and Pathophysiology Superficial veins in the lower extremities become dilated and tortuous in response to backward (retrograde) blood flow and increased venous pressure. Risk factors include family history of chronic venous disease, weak vein structure, female gender, tobacco use, increasing age, obesity, multiparity, history of VTE, venous obstruction resulting from extrinsic pressure by tumors, thrombophilia, phlebitis, previous leg injury, and occupations that require prolonged standing or sitting. In primary varicose veins, weak vein walls allow the vein valve ring to enlarge, so the leaflets no longer fit together properly (incompetent). Incompetent vein valves allow backward blood flow, particularly when the patient is standing. This results in increased venous pressure and further venous distention. High pressure in the superficial veins also can be caused by vein valve dysfunction in the deep veins or perforator veins (veins that perforate the deep fascia of muscles to connect the superficial veins to the deep veins).

Open aneurysm repair (OAR)

Excision of the aneurysm and the placement of a synthetic graft. *Most common surgical procedure for AAA* 1. OAR involves a large abdominal incision through which the surgeon (1) cuts into the diseased aortic segment, (2) removes any thrombus or plaque, (3) sutures a synthetic graft to the aorta proximal and distal to the aneurysm, and (4) sutures the native aortic wall around the graft to act as a protective cover 2. All OARs require aortic cross-clamping proximal and distal to the aneurysm. Most resections are done within 30-45 min, after which the clamps are removed and blood flow is restored 3. *If the cross clamp must be applied above the renal arteries, adequate renal blood flow after clamp removal should be determined before closure* 4. *The risk of post-op renal complications such as AKI increases in patients who have OAR of AAAs above the level of the renal arteries*

Nursing ManagementCare of the Patient Undergoing Paracentesis Preprocedure

Have the patient void or insert an indwelling catheter. • Obtain baseline vital signs and pulse oximetry. Weigh patient, inspect and palpate abdomen, and assess abdominal girth. Assess bladder for distention and determine last voiding. • Assess baseline laboratory values (e.g., CBC, electrolytes, coagulation studies). • Give any sedation or analgesia, if ordered. • Teach patient to remain immobile during the procedure. • Help the patient to a high-Fowler (sitting) position with feet on the floor. Postprocedure • Perform assessment and compare to baseline: vital signs, pulse oximetry, abdominal girth, abdominal pain. Note any signs of hypovolemia. • Have the patient sit on the side of the bed or place in high-Fowler's position. • Label and send the fluid for laboratory analysis. • Check the dressing for bleeding and/or leakage of ascitic fluid. • Give IV fluid and/or albumin as ordered. • Measure any drainage and describe the collected fluid. • Reweigh the patient and monitor intake and output. • Maintain bedrest per agency protocol.

chronic endothelial injury

Hypertension Tobacco use Hyperlipidemia Hyperhomocysteinemia Diabetes Infections viruses, immune reactions

A 67-year-old patient is admitted to the hospital with a diagnosis of venous insufficiency. Which patient statement is most supportive of the diagnosis? ​

I cant get my shoes on at the end of the day.

Cirrhosis S/S

If a person does have symptoms, these may include fatigue or an enlarged liver. The diagnosis of cirrhosis is often made later when a patient presents with symptoms of more advanced liver disease. Late manifestations result from liver failure and portal hypertension Jaundice, peripheral edema, and ascites develop gradually. Other late manifestations include skin lesions, hematologic problems, endocrine problems, and peripheral neuropathies In the advanced stages, the liver becomes small and nodular. Liver function is dramatically impaired.

Drug therapy fro CVI

If the ulcer does not heal with conservative therapy, drug therapy should be considered. Pentoxifylline or micronized purified flavonoid fraction (Daflon) is recommended with compression therapy to improve healing. Pentoxifylline minimizes WBC activation and adhesion to capillary endothelium and decreases oxidative stress. Micronized purified flavonoid fraction(Daflon) acts on WBCs to decrease inflammation and edema. Patients with CVI have dry, flaky, itchy skin. Daily moisturizing decreases itching and prevents skin cracking. Contact dermatitis may result from contact with sensitizing products, such as topical antimicrobial agents (e.g., gentamicin); additives in bandages or dressings (e.g., adhesives); ointments containing lanolin, alcohols, or benzocaine; and over-the-counter creams or lotions with fragrance or preservatives. Assess wounds for signs of infection with each dressing change.

Clinical Manifestations and Complications CVI

In patients with CVI, the skin of the lower leg is leathery, with a characteristic brownish or "brawny" appearance from the hemosiderin deposition. Edema usually has been persistent for a prolonged period. Eczema with itching and scratching is often present Venous ulcers classically occur above the medial malleolus The ulcer is often quite painful, particularly when edema or infection is present. Pain may be worse when the leg is in a dependent position. If the venous ulcer is untreated, the wound becomes wider and deeper, increasing the risk for infection.

Proprotein Convertase Subtilisin/Kexin 9 (PCSK9) Inhibitors alirocumab (Praluent) evolocumab (Repatha)

Inactivate PCSK9 protein ↓ LDL Injection-site reactions, muscle pain, limb pain, and fatigue Used with diet and maximum statin therapy to treat familial hypercholesterolemia and for those who need further LDL lowering or for those who are statin intolerant Drugs are given by injection every 2-4 wks (evolocumab may be given every 4 wks at a higher dose)

Which nursing diagnosis supports a medical diagnosis of cirrhosis? Increased risk for acute confusion Anxiety Activity intolerance Fatigue

Increased risk for acute confusion Clients with cirrhosis deal with a variety of problems, but fatigue, activity intolerance, and anxiety are not among them. A few nursing diagnoses that are appropriate include impaired skin integrity, increased risk for acute confusion, diminished protection, increased fluid volume, and reduced nutrition, less than body requirements.

Drug Alert Simvastatin (Zocor)

Increased risk for rhabdomyolysis when used with fibric acid derivatives (e.g., gemfibrozil [Lopid]), niacin (Niaspan), or erythromycin. • Manifestations of rhabdomyolysis include high creatine kinase levels and muscle pain. • Prothrombin times may increase in patients taking warfarin (Coumadin).

HCV RNA Quantitation - DIAGNOSTIC TEST FOR HEP C

Indicates active ongoing viral replication

HBV DNA quantitation

Indicates active ongoing viral replication Best indicator of viral replication and effectiveness of therapy in patient with chronic hepatitis B

Anti-HBc (antibody to hepatitis B core antigen) IgM diagnostic test for Hepatitis B

Indicates acute infection Does not appear after vaccination

HBeAg (hepatitis B e antigen) Hepatitis B virus test

Indicates high infectivity Used to determine the clinical management of patients with chronic hepatitis B

Anti-HBe (hepatitis B e antibody) - DIAGNOSTIC TEST FOR HEP B

Indicates previous infection In chronic hepatitis B, indicates a low viral load and low degree of infectivity

anti-HBc (IgG) diagnostic test for Hepatitis B

Indicates previous infection or ongoing infection with hepatitis B Does not appear after vaccination (hepatitis serology panel): anti-HBcAg antibody, indicates prior exposure/chronic infection

Anti-HBs (hepatitis B surface antibody) Hepatitis B virus test

Indicates previous infection with HBV or immunization

Niacin Niacin (Niaspan)

Inhibits synthesis and secretion of VLDL and LDL ↓ LDL ↓ Triglycerides ↑ HDL S/E:Flushing and pruritus in upper torso and face, GI problems (e.g., nausea and vomiting, dyspepsia, diarrhea), orthostatic hypotension Most side effects subside with time. Taking aspirin or NSAID 30 min before drug may prevent flushing. Take drug with food. Decreased liver function may occur with high doses.

Omega-3 Fatty Acid icosapent ethyl (Vascepa) (contains eicosapentaenoic acid [EPA])

Inhibits synthesis and/or secretion of triglycerides Arthralgia Used for patients with severe hypertriglyceridemia (levels ≥ 500 mg/dL)

Decrease Cholesterol Absorption Cholesterol Absorption Inhibitor ezetimibe (Zetia)

Inhibits the intestinal absorption of cholesterol ↓ LDL ↑ HDL Infrequent, but may include headache and mild GI distress When used with a statin, further reduces LDL. Should not be used by patients with liver impairment

A client calls the clinic to ask the nurse if it would be okay to take the herbal medication kava-kava to help reduce anxiety. What is the nurse's best response?

Kava-kava can cause liver damage. It is recommended that if if taking kava-kava the client should be under the direct supervision of a primary healthcare provider.

What the nurse must monitor in a patient after a thoracic aneurysm

LOC, neuro check blood pressure signs of aneurism peripheral pulses lower legs for eschemia palpate pulses below the graft stent repair Use dopler ultrasound when pulses not palpable Report sudden sign of pain and discoloration to MD as it might indicate graft oclusion

Red yeast rice

Likely effective for reducing total cholesterol, LDL, and triglycerides. Red yeast rice contains a chemical similar to statins.

Pathophysiology VTE

Localized platelet aggregation and fibrin entrap RBCs, WBCs, and more platelets to form a thrombus. A frequent site of thrombus formation is the valve cusps of veins, where venous stasis occurs. As a thrombus enlarges, increased numbers of blood cells and fibrin collect behind it. This makes a larger clot with a "tail" that eventually blocks the lumen of the vein. If a thrombus only partially blocks the vein, endothelial cells cover the thrombus and stop the thrombotic process. If the thrombus does not become detached, it undergoes lysis or becomes firmly organized and adherent within 5 to 7 days. The organized thrombus may detach and result in an embolus. Turbulence of blood flow is a major factor in embolization. The thrombus can become an embolus that flows through the venous circulation to the heart and lodges in the pulmonary circulation, becoming a PE.

A patient with a venous thromboembolism (VTE) is started on enoxaparin (Lovenox) and warfarin (Coumadin). The patient asks the nurse why two medications are necessary. Which response by the nurse ismostappropriate? ​

Lovenox will work right away, but Coumadin takes several days to have an effect on preventing clots.

Antiplatelet Therapy

Low-dose aspirin (81 mg) is recommended for people who have CAD. For people at risk but without known CAD, low-dose aspirin is recommended for adults 50 to 59 years old who have a calculated 10-year CVD risk of 10% or more, are not at increased risk for bleeding (e.g., history of GI bleeding), have a life expectancy of at least 10 years, and are willing to take low-dose aspirin for at least 10 years. For adults 60 to 69 years old who have a calculated 10-year CVD risk of 10% or more, the decision to take low-dose aspirin is an individual one made in conjunction with the HCP. Adults who have no contraindications (e.g., history of bleeding), have a life expectancy of at least 10 years, and are willing to take low-dose aspirin daily for at least 10 years are more likely to benefit. Currently there is not enough evidence to recommend low-dose aspirin for primary prevention in those younger than 50 or older than 70. Clopidogrel (Plavix) is an option for people who are aspirin intolerant.

The nurse is reviewing the lab results for a pt with cirrhosis and notes that the ammonia level is elevated. Which diet does the nurse anticipate to be presribed for this pt? Low-protein High-protein Moderate-fat High-carb

Low-protein diet Protein provided by the diet is transported to the liver via the portal vein. The liver breaks down protein, which results in the formation of ammonia.

Peripheral Artery Disease of The Lower Extremities

Lower extremity PAD may affect the iliac, femoral, popliteal, tibial, or peroneal arteries, or any combination of these arteries (Fig. 37.1). The femoral popliteal area is the most common site in nondiabetic patients. Patients with diabetes tend to develop PAD in the arteries below the knee. Those with advanced PAD often have multiple arterial occlusions.

α1-Adrenergic Blockers doxazosin (Cardura) prazosin (Minipress) terazosin phentolamine

MOA: Block α1-adrenergic effects, producing peripheral vasodilation (decreases SVR and BP) Beneficial effects on lipid profile consideration: Reduced resistance to the outflow of urine in benign prostatic hyperplasia Take at bedtime to reduce risk associated with orthostatic hypotension phentolamine: Blocks α1-adrenergic receptors, resulting in peripheral vasodilation (decreases SVR and BP) consideration: Used in the short-term management of pheochromocytoma Used locally to prevent necrosis of skin and subcutaneous tissue after extravasation of adrenergic drug No oral formulation

Calcium Channel Blockers amlodipine (Norvasc) clevidipine (Cleviprex) felodipine isradipine nicardipine sustained release nifedipine long acting (Procardia XL) nisoldipine (Sular)

MOA: Cause vascular smooth muscle relaxation resulting in decreased SVR and arterial BP Nursing considerations: More potent peripheral vasodilators Clevidipine is for IV use only; solution must be changed every 12 hrs Use of sublingual short-acting nifedipine in hypertensive emergencies is unsafe and not effective Serious adverse events (e.g., stroke, acute MI) have occurred IV nicardipine is available for hypertensive crisis in hospitalized patients; change peripheral IV infusion sites every 12 hrs

Thiazide and Related Diuretics chlorothiazide chlorthalidone hydrochlorothiazide indapamide metolazone (Zaroxolyn)

MOA: Inhibit NaCl reabsorption in the distal convoluted tubule Increase excretion of Na+ and Cl− Initial decrease in ECF Sustained decrease in SVR Lower BP moderately in 2-4 wk NC: Monitor for orthostatic hypotension, hypokalemia, and alkalosis May potentiate cardiotoxicity of digoxin by producing hypokalemia Dietary sodium restriction reduces the risk for hypokalemia NSAIDs can decrease diuretic and antihypertensive effect and potentially cause renal impairment Teach patient to supplement with potassium-rich foods

Calcium Channel Blockers Non-Dihydropyridines diltiazem extended release (Cardizem LA) verapamil intermediate release (Calan) verapamil timed-release (Verelan PM)

MOA: Inhibit movement of Ca++ across cell membrane, resulting in vasodilation Cardioselective resulting in a decrease in heart rate and slowing of AV conduction Nurse considerations: Use with caution in patients with HF Grapefruit juice may increase serum concentrations and toxicity of certain calcium channel blockers; avoid concurrent use Used for supraventricular tachydysrhythmias Avoid in patients with second- or third-degree AV block or left ventricular systolic dysfunction

Aldosterone Receptor Blockers spironolactone (Aldactone) eplerenone (Inspra)

MOA: Inhibit the Na+-retaining and K+-excreting effects of aldosterone in the distal and collecting tubules Monitor for orthostatic hypotension and hyperkalemia NC: Do not combine with potassium-sparing diuretics or potassium supplements Use with caution in patients on ACE inhibitors or angiotensin II blockers Classified as potassium-sparing diuretics

β-Adrenergic Blockers Cardioselective Blockers acebutolol (Sectral) atenolol (Tenormin) betaxolol bisoprolol esmolol (Brevibloc) metoprolol (Lopressor)

MOA: Reduce BP by blocking β-adrenergic effects Decrease CO and reduce sympathetic vasoconstrictor tone Decrease renin secretion by kidneys Consideration: Monitor pulse and BP regularly Use with caution in patients with diabetes because may depress the tachycardia associated with hypoglycemia and adversely affect glucose metabolism Drug of choice for patients with a history of an MI or HF Less effective BP reduction in black patients Esmolol is for IV use only Lose cardioselectivity at higher doses

Central-Acting α-Adrenergic Agonist clonidine (Catapres) clonidine patch (Catapres-TTS) guanabenz guanfacine (Tenex)not available in pathch) methyldopa

MOA: Reduce sympathetic outflow from central nervous system Reduce peripheral sympathetic tone, produces vasodilation, decreases SVR and BP Nursing consideration: Sudden discontinuation may cause withdrawal syndrome, including rebound hypertension, tachycardia, headache, tremors, apprehension, sweating Chewing gum or hard candy may relieve dry mouth Alcohol and sedatives increase sedation Transdermal patch may be related to fewer side effects and better adherence methyldopa/Teach patient about daytime sedation and avoiding hazardous activities Taking a single daily dose at bedtime minimizes the sedative effect

Direct Vasodilators nitroglycerin

MOA:Relaxes arterial and venous smooth muscle, reducing preload and SVR At a low dose, venous dilation predominates; at a higher dose, arterial dilation is present nurse considerations: IV use for hypertensive crisis in hospitalized patients with myocardial ischemia Given by continuous IV infusion with pump or control device

How is the affected extremity to be treated preoperatively for the patient with PVD?

Maintain in a level position if venous, 15 degrees dependent if arterial; room temperature; protect from trauma

Anti-HCV (antibody to HCV)

Marker for acute or chronic infection with HCV

DAAs DRUG ALERT: Ribavirin (Rebetol, Copegus)

May cause birth defects. During treatment, pregnancy must be avoided both by women taking the drug and by women whose male partners are taking this drug. Monitor Hgb and Hct as it may cause anemia Many pts w/ HIV also have HCV

The nurse observes a distinct change in the Mr. Dontay's level of consciousness during a routine assessment during a scheduled physical examination. Mr. Dontay is 55 years old, a recovering alcoholic, and has a primary diagnosis of cirrhosis. Which intervention is appropriate for Mr. Dontay while providing care? Encouraging large meals Measuring abdominal girth Providing a diet high in sodium Using hot water for bathing

Measuring abdominal girth The client with cirrhosis is at risk for ascites; therefore it is important to measure the client's abdominal girth while providing care. The nurse should encourage small meals, provide a diet low in sodium, and use warm water for bathing.

Endovascular Graft Procedure

Minimally invasive endovascular aneurysm repair (EVAR) is an alternative to OAR for select patients. Eligibility criteria include iliofemoral vessels that allow for safe graft insertion and vessels of sufficient length and width to support the graft EVAR involves the placement of a sutureless aortic graft into the abdominal aorta inside the aneurysm via the femoral artery. Grafts are made of various materials, such as a Dacron cylinder consisting of several sections, and supported with multiple rings of flexible wire.

Ms. Charlotte is 66-years-old and admits to being an alcoholic for most of her adult life. She is brought to the emergency department with bleeding esophageal varices. Which therapy should be the most effective for Ms. Charlotte at this time? Beta-blocker Minnesota tube Paracentesis Transjugular intrahepatic portosystemic shunt (TIPS)

Minnesota tube While a beta-blocker can be used for esophageal varices, the best therapy at this time is a balloon tamponade (either a Sengstaken-Blakemore or Minnesota tube). A paracentesis is done to relieve severe ascites. A transjugular intrahepatic portosystemic shunt (TIPS) relieves portal hypertension and reduces the onset of esophageal varices and ascites.

Which nursing action should be included in the plan of care after endovascular repair of an abdominal aortic aneurysm?

Monitor fluid intake and urine output.

Nurse provides the client with education about her prescription to lower cholesterol Rosuvastatin (Crestor) and what labs must be monitored:

Monitor liver enzymes and creatine kinase (if muscle weakness or pain occurs).

While working in the outpatient clinic, the nurse notes that a patient has a history of intermittent claudication. Which statement by the patient would support this information?

My legs cramp whenever I walk more than a block.

Neurologic complications can occur after aortic surgery.

Neurologic Status When the ascending aorta and aortic arch are involved, assess the patient's level of consciousness, pupil size and response to light, facial symmetry, tongue position, speech, upper extremity movement, and quality of hand grasps). When the descending aorta is involved, perform a neurovascular assessment of the lower extremities. Record all assessments and report changes from baseline to the HCP immediately.

Which assessment finding for a patient who has been admitted with a right calf venous thromboembolism (VTE) requires immediate action by the nurse? ​

New onset shortness of breath

Niacin

Niacin, a water-soluble B vitamin, is effective in lowering LDL and triglyceride levels by interfering with their synthesis Niacin, at high doses, increases HDL levels better than many other lipid-lowering drugs. Unfortunately, side effects are common. They may include severe flushing, pruritus, gastrointestinal (GI) symptoms, and orthostatic hypotension.

C-reactive protein (CRP)

Nonspecific marker of inflammation Increased in many patients with CAD Chronic elevations of CRP linked with unstable plaques and oxidation of LDL cholesterol Intact normal endothelium is more than a simple barrier between the vessel wall and the lumen of the vessel. Normally, it is nonreactive to platelets and leukocytes, as well as to coagulation, fibrinolytic, and complement factors. However, the endothelial lining can be injured as a result of tobacco use, hyperlipidemia, hypertension, diabetes, hyperhomocysteinemia, and infection (e.g., Chlamydia pneumoniae, herpes), causing a local inflammatory response

The nurse is caring for a patient immediately after repair of an abdominal aortic aneurysm. On assessment, the patient has absent popliteal, posterior tibial, and dorsalis pedis pulses. The legs are cool and mottled. Which action should the nurse take first?

Notify the surgeon and anesthesiologist.

OARs complications

OARs require aortic cross-clamping proximal and distal to the aneurysm. Most resections are done in 30 to 45 minutes, then the clamps are removed and blood flow is restored. The risk for postoperative complications, such as acute kidney injury, increases in patients who have OAR of AAAs above the level of the renal arteries.

The nurse is caring for a patient with critical limb ischemia who has just arrived on the nursing unit after having percutaneous transluminal balloon angioplasty. Which action should the nurse perform first? ​

Obtain vital signs.

The health care provider has prescribed bed rest with the feet elevated for a patient admitted to the hospital with venous thromboembolism. Which action by the nurse to elevate the patients feet is best? ​

One pillow is placed under the thighs and two pillows are placed under the lower legs.

amivudine /Epivir, adefovir (Hepsera), entecavir (Baraclude), telbivudine (Tyzeka), and tenofovir (Viread).

Oral meds are used in the treatment of chronic HBV when there is evidence of significant active viral replication and liver inflammationoBeneficial effects in terms of reducing viral load, ↓ liver damage, and ↓ liver enzymes. oMost pts w/ HBV require long-term treatment w/ these meds When these drugs are stopped, the majority of pts (except those who have seroconverted) have HBV DNA and liver enzyme levels that return to pretreatment levels. oSevere exacerbations of Hep B have developed after discontinuation of treatment. oIf these drugs are discontinued, monitor liver function closely for several months

Thrombolytic Agents Implementation High (2)Alert:

Overdosage and underdosage of thrombolytic medications have resulted in patient harm or death.*Do not confuse the abbreviation t-PA for alteplase (Activase) with the abbreviation TNK t-PA for tenecteplase (TNKase) and r-PA for reteplase (Retavase).

What must the nurse teach the patient with diabetes regarding Tricor

Patients with diabetes mellitus are at increased risk for serious muscle toxicity, including myopathy and rhabdomyolysis, when treated with fibrate therapy. Additionally, because fenofibrate therapy has been associated with elevations in serum creatinine, renal monitoring should be considered in patients at increased risk for renal impairment including those with diabetes. Of note, fenofibrate was not shown to reduce coronary heart disease morbidity and mortality in a large, randomized controlled trial of patients with type 2 diabetes mellitus.

PegIntron, Pegasys) Pegylated interferon

Pegasys is used alone or with other medicines to treat chronic hepatitis C in adults and children at least 5 years old. Pegasys is also used to treat chronic hepatitis B when the virus begins to damage the liver in adults and children at least 3 years old. is given subcut The numerous side effects w/ interferon therapy, including flu-like symptoms (e.g., fever, malaise, fatigue), make adherence to therapy challenging for some pts Pts receiving interferon should have blood counts and liver function testes performed q4-6wks oDepression is a side effect; screening for this and other mood disorders before starting interferon treatment and monitor frequentl

Describe the pain associated with PVD of venous origin. What relieves this pain?

Persistent, aching, full feeling, dull sensation; relieved when horizontal ( elevate and use elastic stockings)

Damaged endothelium

Possible causative agents that damage endothelium include smoking, hypertension, diabetes mellitus, turbulent blood flow, bacteria, viruses and/or homocysteine Make hemostatic "plug" to fill hole; vessel constriction, platelet plug, reinforced by fibrin

Green tea

Possibly effective for reducing total cholesterol

Black psyllium

Possibly effective for reducing total cholesterol and LDL

Berberine

Possibly effective for reducing total cholesterol, low-density lipoproteins (LDLs), triglyceride This product can cause kernicterus in newborns, particularly preterm neonates with hyperbilirubinemia.

Soy

Possibly effective for slight reduction in total cholesterol and LDL

Anti-HDV

Present in past or current infection with HDV

HDV Ag (hepatitis D antigen)

Present within a few days after infection

Anti-HAV immunoglobulin G (IgG) - DIAGNOSTIC TEST FOR HEP A

Previous infection or immunization Not routinely done in clinical practice

Graduated compression stockings

Proper stocking use means any toe hole is under the toes, the heel patch is over the heel, a thigh gusset is on the inner thigh (thigh length only), and there are no wrinkles. The stockings should not be rolled down, cut, or otherwise altered. If the stockings are not fitted and worn correctly, venous return is impeded. This can cause arterial ischemia, edema, skin breakdown, and VTE. Stockings are not recommended if the patient already has a VTE.

Nurse provides the client with education about her prescription to lower cholestrol medication Rosuvastatin (Crestor) and its side effects of:

Rash, GI problems, high liver enzymes, myopathy, rhabdomyolysis

Nursing Implementation: Health Promotion Aneurysm

Reduce risks for CVD including controlling BP, ceasing tobacco maintaining normal body weight and serum lipid levels. These measures also help ensure continued graft patency after surgical repair. Counsel the patient about taking part in moderate physical activity.

Direct Vasodilators hydralazine

Reduces SVR and BP by direct arterial vasodilation IV use for hypertensive crisis in hospitalized patients Twice-daily oral dosage Not used as monotherapy because of side effects: chest pain or pressure, pain spreading to your jaw or shoulder; fast or pounding heartbeats; a light-headed feeling, like you might pass out; numbness, tingling, or burning pain in your hands or feet; painful or difficult urination; little or no urination Contraindicated in patients with CAD

Direct Vasodilators minoxidil

Reduces SVR and BP by direct arterial vasodilation Reserved for treatment of severe hypertension associated with renal failure and resistant to other therapy Once- or twice-daily dosage

Which therapy for cirrhosis is considered nutritional therapy? Increasing fluid intake Restricting sodium intake Administering vitamin K Recommending antacids

Restricting sodium intake Nutritional support for cirrhosis includes restricting sodium intake to 2 g per day. Administering vitamin K and recommending antacids is pharmacologic therapy. Decreasing fluid intake, not increasing it, is considered a nutritional therapy for cirrhosis.

Why shouldn't a patient with PVD use external heat sources such as a heating pad?

Sensation in the extremity is diminished so that the patient could burn themselves without realizing it

Rosuvastatin (Crestor) is the most potent statin currently available. What side effects the patient should be aware of and report to PCM

Serious adverse effects of these drugs are rare and include liver damage and myalgia (muscle ache or weakness without breakdown of skeletal muscle) that can progress to rhabdomyolysis (breakdown of skeletal muscle). Liver enzymes are initially monitored and rechecked with any increase in dosage. The creatine kinase isoenzyme, CK-MM (found in skeletal muscle), is assessed if symptoms of myopathy (e.g., muscle aches, weakness) occur.

Nurse provides the client with education about her prescription to lower cholesterol Rosuvastatin (Crestor) and serious adverse side effects:

Serious adverse effects of these drugs are rare and include liver damage and myalgia (muscle ache or weakness without breakdown of skeletal muscle) that can progress to rhabdomyolysis (breakdown of skeletal muscle). Liver enzymes are initially monitored and rechecked with any increase in dosage. The creatine kinase isoenzyme, CK-MM (found in skeletal muscle), is assessed if symptoms of myopathy (e.g., muscle aches, weakness) occur.

What type of pain is associated with PVD of arterial origin?

Sharp pain that increases with walking and elevation; intermittent claudication; rest pain when extremities are horizontal - may be relieved by dependent position

hepatitis A test

Significance: Anti-HAV immunoglobulin M (IgM): Acute infection Anti-HAV immunoglobulin G (IgG): Previous infection or immunization Not routinely done in clinical practice

Why shouldn't a patient with PVD smoke?

Smoking causes vasoconstriction and spasm of arteries

Patient teaching for diuretics

Some unpleasant side effects result from a drug's therapeutic effect, but these can be decreased. For example, diuretics cause dry mouth and frequent voiding. Sugarless gum or hard candy may help ease the dry mouth. Taking diuretics earlier in the day may limit frequent voiding during the night and preserve sleep.

A nurse is caring for a client with ascites secondary to cirrhosis. Which medication is the treatment of choice? Spironolactone (Aldactone) Neomycin sulfate Oxazepam (Serax) Furosemide (Lasix)

Spironolactone (Aldactone) Diuretics are used to reduce fluid retention and ascites. While furosemide (Lasix) may be used, the drug of choice is spironolactone (Aldactone) because it is also an Aldosterone Antagonist. Neomycin sulfate reduces the number of ammonia-forming bacteria in the bowel, and oxazepam (Serax) is used for acute agitation.

A patient in the outpatient clinic has a new diagnosis of peripheral artery disease (PAD). Which group of medications will the nurse plan to include when providing patient teaching about PAD management?

Statins

Drugs That Decrease Cholesterol Absorption

Statins. PCSK9 inhibitors. Fibric acid derivatives (also called fibrates). Bile acid sequestrants (also called bile acid resins). Nicotinic acid (also called niacin). Selective cholesterol absorption inhibitors. Omega 3 fatty acids and fatty acid esters. Adenosine triphosphate-citrate lyase (ACL) inhibitors. Ezetimibe (Zetia) selectively inhibits the absorption of dietary and biliary cholesterol across the intestinal wall. It serves as an adjunct to dietary changes, especially for patients with primary hypercholesterolemia. When it is combined with a statin, either as a combination drug (e.g., ezetimibe and simvastatin [Vytorin]) or as 2 separate drugs (ezetimibe and a statin), even greater reductions in LDLs occur.

Thrombin Inhibitors: Indirect Low-Molecular-Weight Heparin (LMWH) dalteparin (Fragmin) enoxaparin (Lovenox)

Subcutaneous Considerations: Routine coagulation tests typically not needed. Monitor CBC count at regular intervals. Do not expel air bubble from prefilled syringe. If giving subcutaneously, inject deep into subcutaneous tissue (preferably into the abdominal fatty tissue or above the iliac crest), inserting the entire length of the needle. Hold skinfold during injection but release before removing needle. Do not aspirate. Do not inject IM. Do not rub site after injection. Rotate sites. Reduced dosage needed in patients with renal impairment. Use extreme caution in patients with a history of HIT. Antidote: Protamine neutralizes the effects of LMWH.

signs and symptoms of intracraneal bleeding

Sudden tingling, weakness, numbness, or paralysis of the face, arm or leg, particularly on one side of the body. Headache. ... Nausea and vomiting. Confusion. Dizziness. Seizures. Difficulty swallowing. Loss of vision or difficulty seeing.

aneurysm rupture

Surgery is the only intervention for clients with rupture. Pt will have significant hemodynamic instability due to preoperative blood loss and ishemic organ disease. Abrupt excruciating pain is the most common presenting manifestation in clients with aortic dissection. -ripping or knifelike, tearing sensations that radiate to the back, abdomen, extremities or anterior part of the chest. -looks "shocky" and is sweating profusely, is severely apprehensive and has diminished peripheral pulses. -unequal pulses. -Different blood pressures in the arms, -paraplegia or hemiplegia, -decreased uring output or hematuria, -mental status changes -chest pain.

Surgical repair aneurysms

Surgical repair is recommended in patients with asymptomatic aneurysms 5.5 cm in diameter or larger. Surgical intervention may occur sooner if the patient has a genetic disorder (e.g., Marfan's, Ehlers-Danlos syndrome), the aneurysm expands rapidly, the patient becomes symptomatic, or the risk for rupture is high.

What are some noninvasive treatments for venous PVD?

Systemic antibiotics, compression dressing, limb elevation, fibrinolytic agents and anticoagulants if thrombosis

Ascites, a common manifestation of hepatitis

T (especially chronic hepatitis) Ascites, a common manifestation of hepatitis (especially chronic hepatitis), is the accumulation of excess fluid in the peritoneal cavity. Fluid accumulates due to reduced protein levels in the blood, which reduces the plasma oncotic pressure.

Nutritional Therapy education in PAD

Teach patients with PAD to maintain a body mass index (BMI) less than 25 kg/m2 and a waist circumference less than 40 inches for men and less than 35 inches for women. Even modest, sustained weight loss of 3% to 5% yields important reductions in triglycerides, glucose, A1C, and the risk for developing type 2 diabetes. Greater weight loss has greater benefits.5 Recommend a diet reduced in calories and salt for obese or overweight persons.5

Ambulatory Care teaching Aneurysm

Teach the patient and caregiver to gradually increase activities once home. Fatigue, poor appetite, and irregular bowel patterns are common. Have the patient avoid heavy lifting for 6 weeks after surgery. Report any redness, swelling, increased pain, drainage from incisions, or fever greater than 100° F (37.8° C) to the HCP. Teach the patient and caregiver to look for changes in color or warmth of the extremities. Patients and caregivers can learn to palpate peripheral pulses to assess changes in their quality. Sexual dysfunction in male patients is common after aortic surgery. A referral to a urologist and counseling may be useful if erectile dysfunction occurs. Evaluation Expected outcomes are that the patient who undergoes aortic surgery will have: • Patent arterial graft with adequate distal perfusion • Adequate urine output • No signs of infection

Interpretation of the hepatitis B serologic panel

Tests HBsAg anti-HBc anti-HBs IgM anti-HBc HBsAg (hepatitis B surface antigen) Marker of infectivity Present in acute or chronic infection Positive in chronic carriers Anti-HBs (hepatitis B surface antibody) Indicates previous infection with HBV or immunization HBeAg (hepatitis B e antigen) Indicates high infectivity Used to determine the clinical management of patients with chronic hepatitis B Anti-HBe (hepatitis B e antibody) Indicates previous infection In chronic hepatitis B, indicates a low viral load and low degree of infectivity Anti-HBc (antibody to hepatitis B core antigen) IgM Indicates acute infection Does not appear after vaccination Anti-HBc IgG Indicates previous infection or ongoing infection with hepatitis B Does not appear after vaccination HBV DNA quantitation Indicates active ongoing viral replication Best indicator of viral replication and effectiveness of therapy in patient with chronic hepatitis B HBV genotyping Indicates the genotype of HBV

Your patient with cirrhosis has severe splenomegaly. As the nurse you will make it priority to monitor the patient for signs and symptoms of? Select all that apply: A. Thrombocytopenia B. Vision changes C. Increased PT/INR D. Leukopenia

The answers are A, C, and D. A patient with an enlarged spleen (splenomegaly) due to cirrhosis can experience thrombocytopenia (low platelet count), increased PT/INR (means it takes the patient a long time to stop bleeding), and leukopenia (low white blood cells). The spleen stores platelets and WBCs. An enlarged spleen can develop due to portal hypertension, which causes the platelets and WBCs to become stuck inside the spleen due to the increased pressure in the hepatic vein (hence lowering the count and the body's access to these important cells for survival).

clinical manifestation PAD

The classic symptom of lower extremity PAD is intermittent claudication. This ischemic muscle pain is caused by exercise, resolves within 10 minutes or less with rest, and is reproducible. The ischemic pain is due to the buildup of lactic acid from anaerobic metabolism. Once the patient stops exercising, the lactic acid clears, and the pain subsides. PAD of the iliac arteries causes claudication in the buttocks and thighs. Calf pain indicates femoral or popliteal artery involvement. Others have no symptoms or present with atypical leg symptoms (e.g., burning, heaviness, pressure, soreness, tightness, weakness) in atypical locations (e.g., ankle, foot, hamstring, hip, knee, shin). PAD involving the internal iliac arteries may result in erectile dysfunction. Paresthesia (numbness or tingling) in the toes or feet may result from nerve tissue ischemia. True peripheral neuropathy occurs more often in patients with diabetes and in those with long-standing ischemia. Neuropathy causes severe shooting or burning pain in the extremity. It does not follow particular nerve roots and may be present near ulcerated areas. Gradual, reduced blood flow to neurons causes loss of pressure and deep pain sensations. So, patients may not notice lower extremity injuries. The skin becomes thin, shiny, and taut. The lower legs lose their hair. Pedal, popliteal, or femoral pulses are decreased or absent. Pallor (blanching of the foot) develops when the leg is elevated (elevation pallor). Conversely, reactive hyperemia (redness of the foot) develops when the limb is in a dependent position (dependent rubor)

Peripheral Perfusion Status post aneurysm repair

The location of the aneurysm determines what type of peripheral perfusion assessment to do. Check and record all peripheral pulses hourly for several hours and then routinely (based on agency policy). When the ascending aorta and aortic arch are involved, assess the carotid, radial, and temporal artery pulses. For surgery of the descending aorta, assess the femoral, popliteal, posterior tibial, and dorsalis pedis pulses a Doppler maybe used to assess peripheral pulses. Check skin temperature and color, capillary refill time, and sensation and movement of the extremities. Sometimes, lower extremity pulses may be absent for a short time after surgery because of vasospasm and hypothermia. A decreased or absent pulse together with a cool, pale, mottled, or painful extremity may indicate embolization or graft occlusion. Report these findings to the HCP at once. Graft occlusion requires reoperation if identified early. It is essential to compare your findings with the preoperative status to determine the cause of a decreased or absent pulse and the proper treatment. In some patients, pulses may have been absent before surgery because of coexistent PAD.

Complications VTE

The most serious complications of VTE are PE, chronic thromboembolic pulmonary hypertension, post-thrombotic syndrome, and phlegmasia cerulea dolens. Post-thrombotic syndrome (PTS) occurs in 8% to 70% of patients. It results from chronic inflammation and chronic venous hypertension. Chronic venous hypertension is caused by vein wall and vein valve damage (from acute inflammation and thrombus reorganization), venous valve reflux, and persistent venous (outflow) obstruction. Symptoms include pain, aching, fatigue, heaviness, sensation of swelling, cramps, pruritus, tingling, paresthesia, bursting pain with exercise, and venous claudication.1 Manifestations include persistent edema, spider veins (telangiectasia), venous dilation (ectasia), redness, cyanosis, increased pigmentation, eczema, pain during compression, atrophie blanche (white scar tissue), and lipodermatosclerosis Venous ulceration can occur with severe PTS. Signs of PTS typically begin within a few months to a few years of a VTE. Risk factors include persistent leg symptoms 1 month after VTE, proximal VTE location (e.g., near the iliofemoral junction), extensive VTE, recurrent ipsilateral (same side) VTE, residual thrombus, obesity, older age, poor INR control, daily tobacco use before pregnancy, increased D-dimer levels, elevated inflammatory markers, varicose veins, and asymptomatic VTE. Phlegmasia cerulea dolens (swollen, blue, painful leg) is a rare complication of a severe lower extremity VTE(s). It involve(s) the major leg veins, causing near-total occlusion of venous outflow. Patients typically have sudden, massive swelling; deep pain; and intense cyanosis of the extremity. If untreated, the venous obstruction causes arterial occlusion and gangrene, requiring amputation.

Which action by a nurse who is giving fondaparinux (Arixtra) to a patient with a lower leg venous thromboembolism (VTE) indicates that more education about the drug is needed?

The nurse ejects the air bubble in the syringe before giving the drug.

Interpretation of Hepatitis testing

The only definitive way to distinguish among the types of viral hepatitis is by testing the patient's blood for the specific antigen or antibody. In some types of viral hepatitis, the blood can be tested for the viral load (viral level). Antibodies can be detected within 4 weeks of infection. If the antibody test is positive, HCV RNA testing is done to assess for chronic infection. A positive result confirms chronic infection. A few patients may have a false-positive HCV antibody result with a negative HCV RNA test HBV genotype may be useful in predicting disease course and treatment outcomes. HBV core Ab (HBVcAb) IgG indicates that the patient has a history of HBV infection. HBV may reactivate in these patients if they become immunosuppressed.

Renal Perfusion Status after anuerysm repair

The patient will have an indwelling urinary catheter after surgery. In the immediate postoperative period, record hourly urine output. Further evaluate renal function by monitoring daily BUN and serum creatinine levels. CVP pressures give vital information about hydration status. Maintain accurate fluid intake and output and record daily weights until the patient resumes a regular diet. Decreased renal perfusion can occur from embolization of an aortic thrombus or plaque to 1 or both renal arteries. This causes ischemia of 1 or both kidneys. Hypotension, dehydration, prolonged aortic clamping during surgery, or blood loss can lead to decreased renal perfusion. Irreversible renal failure may occur after surgery, particularly in high-risk people (e.g., patients with diabetes).

Venous Thromboembolism Clinical Manifestations

The patient with lower extremity VTE may or may not have unilateral leg edema, pain, tenderness with palpation, dilated superficial veins, a sense of fullness in the thigh or the calf, paresthesias, warm skin, erythema, or a systemic temperature greater than 100.4° F If the inferior vena cava is involved, both legs may be edematous and cyanotic. About 10% of VTEs involve the upper extremity veins and may extend into the internal jugular vein or superior vena cava.17 If the superior vena cava is involved, similar symptoms may occur in the arms, neck, back, and face. Diagnosis of an initial VTE is based on the assessment combined with D-dimer testing and/or ultrasound.

Superficial Vein Thrombosis Clinical Manifestations

The patient with superficial vein thrombosis may have a palpable, firm, subcutaneous cordlike vein The area surrounding the vein may be itchy, painful to the touch, reddened, and warm. A mild temperature elevation and leukocytosis may be present. Extremity edema may occur. Lower extremity superficial vein thrombosis often involves 1 or more varicose veins. Risk factors include increased age, pregnancy, obesity, cancer, recent fracture(s), estrogen therapy, recent sclerotherapy (e.g., treatment for varicose veins), recent surgery or long-distance travel, hypercoagulability, and a history of chronic venous insufficiency (CVI), superficial vein thrombosis, or VTE.18 It can occur in people with endothelial problems (e.g., Buerger's disease). If the superficial vein thrombosis affects a very short vein segment (less than 5 cm) and is not near the saphenofemoral junction, anticoagulants may not be needed and oral NSAIDs can ease symptoms. Other interventions include telling the patient to wear graduated compression stockings or bandages, apply warm compresses, elevate the affected limb above the level of the heart, apply topical NSAIDS, and perform mild exercise, such as walking.

What nursing education should you provide to a patient prescribed Tricor who is also taking Statins

The risk of a serious muscle problem may be higher when TriCor is given with statins. If you take a statin tell your healthcare provider. Some people require blood tests to check for kidney problems while taking TriCor. TriCor may cause inflammation (swelling) of the gallbladder or pancreas. eg)Using simvastatin together with fenofibrate may increase the risk of a rare but serious condition called rhabdomyolysis

Splenomegaly in liver disease

Thrombocytopenia, leukopenia, and anemia are probably caused by the splenomegaly that results from backup of blood from the portal vein into the spleen (portal HTN)

QUESTION: A patient will be receiving a thrombolytic drug as part of the treatment for acute myocardial infarction. The nurse explains to him that this drug is used to:

Thrombolytic drugs lyse, or dissolve, thrombi.

What is the purpose of liver functions tests in diagnosing cirrhosis? To determine the presence of anemia To determine the prothrombin time To determine glucose and lipid metabolism To determine the degree of elevation of liver enzymes

To determine the degree of elevation of liver enzymes The purpose of liver functions tests in diagnosing cirrhosis is to determine the degree of elevation of liver enzymes. A CBC is used to determine the presence of anemia. Coagulation studies are used to determine the prothrombin time. Serum glucose and cholesterol levels are used to determine the effect cirrhosis is having on glucose and lipid metabolism.

TIPS

Transjugular intrahepatic portosystemic shunt (TIPS) is a nonsurgical procedure in which a tract (shunt) between the systemic and portal venous systems is created to redirect portal blood flow is used to treat ascites that does not respond to diuretics. Limitations of TIPS include the increased risk for hepatic encephalopathy (toxin-containing blood bypasses the liver) and stenosis of the stent. TIPS is contraindicated in patients with severe hepatic encephalopathy, liver cancer, severe hepatorenal syndrome, and portal vein thrombosis.

PAD treatement

Treatment of hyperlipidemia and hypertriglyceridemia Antiplatelet agent (aspirin, clopidogrel [Plavix]) ACE inhibitors

carotid bruit

Turbulent blood flow heard with auscultation over the carotid artery. (Due to carotid narrowing or plaque)

Arterial bruit

Turbulent flow sound in peripheral artery Arterial obstruction or aneurysm

Drug Alert: Doxazosin (Cardura)

Use caution when giving the first dose. It is best to give the first dose at bedtime to reduce the first dose BP drop. • Syncope occasionally occurs 30 to 90 min after the first dose, a too-rapid increase in dose, or addition of another antihypertensive agent to therapy. • Drug interactions (severe low BP) can occur with patients taking phosphodiesterase inhibitors, such as sildenafil (Viagra) or tadalafil (Cialis).

Venous Thromboembolism (VTE)*

Usual location:Deep veins of arms (e.g., axillary, subclavian), legs (e.g., femoral), pelvis (e.g., iliac), vena cava, and pulmonary system. Clinical findings:Tenderness to pressure over involved vein, induration of overlying muscle, venous distention. Unilateral Edema. May have mild to moderate pain, deep reddish color to area caused by venous congestion. Some have no obvious physical changes in the affected extremity. Sequelae If untreated, clot may extend to deeper veins and VTE may occur.

Varicose Veins

Varicose veins, or varicosities, are dilated (3 mm or larger in diameter), tortuous superficial veins often found in the saphenous vein system. Varicosities may be small and harmless or large and bulging. Primary varicose veins (idiopathic) are due to a weakness of the vein walls. They are more common in women. Secondary varicose veins result from direct injury, a previous VTE, or excessive vein distention. Secondary varicose veins may occur in the esophagus (esophageal varices), vulva, spermatic cords (varicoceles), and anorectal area (hemorrhoids), and as abnormal arteriovenous (AV) connections. Congenital varicose veins result from chromosomal defects that cause abnormal development of the venous system.22 Reticular veins are smaller varicose veins that appear flat, less tortuous, and blue-green in color. Telangiectasias (often called spider veins), are small visible vessels (generally less than 1 mm in diameter) that appear bluish black, purple, or red.

Venous stasis

Venous stasis occurs when the valves are dysfunctional or the muscles of the extremities are inactive. Venous stasis occurs most often in people who are obese or pregnant, have chronic HF or atrial fibrillation, have been traveling on long trips without regular exercise, have a prolonged surgical procedure, or are immobile for long periods (e.g., spinal cord injury, fractured hip, limb paralysis).

Venous Thromboembolism (VTE)

Venous thrombosis involves the formation of a thrombus (blood clot) with vein inflammation. It is the most common disorder of the veins. We classify it as either superficial vein thrombosis or deep vein thrombosis. Superficial vein thrombosis is the formation of a thrombus in a superficial vein, usually the greater or lesser saphenous vein. Deep vein thrombosis (DVT) involves a thrombus in a deep vein, most often the iliac and/or femoral veins. Venous thromboembolism (VTE) is the preferred terminology. It represents the spectrum from DVT to pulmonary embolism (PE) Superficial vein thrombosis is serious. Nearly 25% of patients with superficial vein thrombosis have a DVT or PE at the time of diagnosis. Further, these patients are at risk for developing recurrent VTE.

The nurse assesses for which item during the health history for a client with cirrhosis? Skin color Mental status Weight loss Vital signs

Weight loss For a client with cirrhosis, the nurse assesses recent weight loss during the heath history portion of the nursing assessment. Vital signs, mental status, and skin color are assessed during the physical examination portion of the nursing assessment.

presence of hepatitis B surface antigen (HBsAg), hepatitis B antibody (anti-HBs), and hepatitis B core antibody (anti-HBc).

When screening for HVB what are the clinical findings: -The presence of anti-HBs in the blood indicates immunity from the HBV vaccine or from past HBV infection -HBsAg in the serum for 6 months or longer after infection indicates chronic HBV infection.

The nurse is caring for an 82-year-old patient. The nurse knows that changes in cardiac structure and function occur in older adults. What is a normal change expected in the aging heart of an older adult? A) Decreased left ventricular ejection time B) Decreased connective tissue in the SA and AV nodes and bundle branches C) Thinning and flaccidity of the cardiac values D) Widening of the aorta

Widening of the aorta

Venous Stasis

a condition of slow blood flow in the veins, usually of the legs • Advanced age • Atrial fibrillation • Bed rest • Chronic heart failure • Fractured leg or hip • Long trips without adequate exercise • Obesity • Orthopedic surgery (especially hip or lower extremity) • Pregnancy and postpartum period • Prolonged immobility • Spinal cord injury or limb paralysis • Stroke • Varicose veins

Lipid lab values

a high-density lipoprotein (HDL) level less than 40 mg/dL (1.0 mmol/L) in men and less than 50 mg/dL (1.3 mmol/L) in women, and/or a fasting triglyceride level greater than 150 mg/dL (1.7 mmol/L).

Hepatitis C diagnosis

a positive viral load is needed to confirm active infection because anti-HCV can also indicate a past infection. Rapid diagnostic tests (RDTs), including oral HCV antibody (Ab) RDTs

Complications that can result from acute hepatitis are

acute liver failure, chronic hepatitis, cirrhosis of the liver, portal hypertension, and liver cancer. chronic liver disease, cirrhosis, portal hypertension, and liver cancer. Risk factors for progression to cirrhosis include male gender, alcohol use, concomitant fatty liver disease, and excess iron deposition in the liver.

Supportive measures during an acute variceal bleed include

administration of fresh frozen plasma and packed RBCs, vitamin K, and PPI (e.g., pantoprazole [Protonix]) Lactulose (Cephulac) and rifaximin (Xifaxan) may be administered to prevent hepatic encephalopathy from breakdown of blood and the release of ammonia in the intestine. Antibiotics are given to prevent bacterial infection Longterm management of pts who have had an episode of bleeding includes nonselective β-blockers.

Nonmodifiable risk factors are/CAD

age, gender, ethnicity, family history, and genetics.

Asterixis

aka Liver Flap, a flapping tremor of the hands. When the client extends the arms & hands in front of the body, the hands rapidly flex & extend.

ALT, AST

alanine transaminase, aspartate transaminase

Manifestations of chronic hepatitis include

anemia and coagulation problems (easy bruising, bleeding). Since the liver produces clotting factors, clotting and bleeding times can be impaired or prolonged. Skin manifestations may include spider angiomas, palmar erythema, and gynecomastia. Some patients have spleen, liver, or cervical lymph node enlargement.

The nurse is reviewing the record of a client with a dx of cirrhosis and notes that there is documentation of the presence of asterixis. How should the nurse assess for its presence? a)Dorsiflex the foot b)Measure abdominal girth c)Ask pt to extend the arms d)Instruct pt to lean forward

ans:C Ask the pt to extend the armsAsterixis is irregular flapping movements of the fingers and wrists when the hands and arms are outstretched, with the palms down, wrists bent up, and fingers spread. It is the most common and reliable sign that hepati encephalopathy is developing.

Factor Xa Inhibitors

apixaban (Eliquis) PO betrixaban (Bevyxxa) PO edoxaban (Savaysa) PO fondaparinux (Arixtra) Subcutaneous rivaroxaban (Xarelto) PO Considerations: All are approved for VTE prevention and treatment. Routine coagulation tests not needed. Monitor CBC and creatinine at regular intervals. May cause thrombocytopenia. Do not expel air bubble before giving fondaparinux. Follow administration guidelines as described for subcutaneous LMWHs. Antidote: Andexanet alfa (Andexxa) reverses the effects of rivaroxaban and apixaban.

Thoracic aortic aneurysms (TAAs)

are often asymptomatic. When present, symptoms include deep, diffuse chest pain that may extend to the interscapular area. Ascending aorta and aortic arch aneurysms can cause: (1) angina from decreased blood flow to the coronary arteries; (2) transient ischemic attacks from decreased blood flow to the carotid arteries; (3) coughing, shortness of breath, hoarseness, and/or difficulty swallowing from pressure on the laryngeal nerve. If the aneurysm presses on the superior vena cava, decreased venous return can result in jugular venous distention and edema of the face and arms.

Synthetic Thrombin Inhibitors

argatroban (Acova) IV dabigatran (Pradaxa) PO Argatroban therapeutic effect measured by aPTT. No routine coagulation tests needed for dabigatran. Used in patients at risk for or with HIT, for VTE prevention in joint replacement surgery, stroke prevention in nonvalvular atrial fibrillation Antidote: Idarucizumab (Praxbind) neutralizes the effect of dabigatran only.

A patient at the clinic says, I have always taken a walk after dinner, but lately my leg cramps and hurts after just a few minutes of starting. The pain goes away after I stop walking, though. The nurse should

assess for the presence of the dorsalis pedis and posterior tibial pulses.

Fatty streaks in an organ are associated with

atherosclerosis Earliest lesions Characterized by lipid-filled smooth muscle cells Potentially reversible

Thrombin Inhibitors: Direct Hirudin Derivatives

bivalirudin (Angiomax) IV desirudin (Iprivask) Subcutaneous Considerations: Therapeutic effect measured by ACT or aPTT. Used in patients with HIT when anticoagulation is needed. Antidote: None.

Several hours after an open surgical repair of an abdominal aortic aneurysm, the UAP reports to the nurse that urinary output for the past 2 hours has been 40 mL. The nurse notifies the health care provider and anticipates an order for a(n)

blood urea nitrogen (BUN) level.

bruit

blowing, swooshing sound heard through a stethoscope when an artery is partially occluded, notify MD and Document location.

LDL greater than 130 mg/dL (3.4 mmol/L)

boarderline

a soft swishing sound that can occur with a goiter or hyperthyroidism.

bruit

VTE 3 key factors

called Virchow's triad that cause venous thrombosis are (1) venous stasis, (2) damage of the endothelium (inner lining of the vein), and (3) hypercoagulability of the blood The patient at risk for developing VTE usually has predisposing conditions to these 3 disorders

HBV

can cause acute and chronic hepatitis The incidence of HBV infection has decreased in areas where the use of the HBV vaccine is widespread the incidence of infection has declined since we started vaccinating at-risk persons and children (beginning 12 to 24 months)

What is atherosclerosis, how do we get it?

can occur in any artery in the body. atherosclerosis begins as soft deposits of fat that harden with age "hardening of the arteries." Atherosclerosis is thickening or hardening of the arteries caused by a buildup of plaque in the inner lining of an artery. Risk factors may include high cholesterol and triglyceride levels, high blood pressure, smoking, diabetes, obesity, lack of physical activity, and eating saturated fats.

many patients with chronic HCV infection develop

chronic liver disease, cirrhosis, portal hypertension, and liver cancer.

Chronic inflammation and cell necrosis result in fibrosis can lead to

cirrohsis

Restrict Lipoprotein Production Fibric Acid Derivatives fenofibrate (Tricor) gemfibrozil (Lopid)

classification Therapeutic: lipid-lowering agents Pharmacologic: fibric acid derivatives Decrease hepatic synthesis and secretion of VLDL. Reduce triglycerides by ↓ VLDL ↓ LDL ↓ Triglycerides ↑ HDL Rashes, mild GI problems (e.g., nausea, diarrhea), high liver enzymes May ↑ effects of warfarin (Coumadin) and some antihyperglycemic drugs. When used in combination with statins, may increase adverse effects of statins, especially myopathy.

During assessment of a pt with obstructive jaundice, the nurse would expect to find: clay colored stools dark urine and stool pyrexia and pruritis elevated urinary urobilinogen

clay colored stool

Hepatic encephalopathy:

confusion, difficulty concentrating, easy agitation

Balloon tamponade

controls hemorrhage by mechanical compression of the varices Balloon tamponade is an option for patients who have bleeding that is unresponsive to band ligation or sclerotherapy. When balloon tamponade is used, explain to the patient and caregiver the use of the tube and how the balloon is inserted. Check the balloons for patency. It is usually the HCP's responsibility to insert the tube by either the nose or mouth. Then the gastric balloon is inflated with 250 mL of air, and the tube is retracted until resistance (lower esophageal sphincter) is felt. The tube is secured by placing a piece of sponge or foam rubber at the nostrils (nasal cuff). For continued bleeding, the esophageal balloon is then inflated. A sphygmomanometer is used to measure and maintain the desired pressure at 20 to 40 mm Hg. An x-ray verifies the balloon's position. If the gastric balloon breaks or is deflated, the esophageal balloon will slip upward, obstructing the airway and causing asphyxiation. If this happens, cut the tube or deflate the esophageal balloon. Keep scissors at the bedside. Minimize regurgitation by oral and pharyngeal suctioning and by keeping the patient in a semi-Fowler's position. Encourage the patient to expectorate and provide an emesis basin and tissues. Frequent oral and nasal care offers relief from the taste of blood and irritation from mouth breathing.

HMG-CoA Reductase Inhibitors (Statins)

decrease LDL, increase HDL, hepatotoxic, myopathy, monitor CK, no grapefruit juice, erythromycin, ketoconazole, ezetimibe, gemfibrozil, fenofibrate inhibit the synthesis of cholesterol in the liver. is an increase in hepatic LDL receptors. This allows the liver to remove more LDL from the blood. In addition, statins cause a minor increase in HDL and lower CRP levels. atorvastatin (Lipitor) fluvastatin (Lescol XL) lovastatin pitavastatin (Livalo) pravastatin (Pravachol) rosuvastatin (Crestor) simvastatin (Zocor) Mechanism of Action: Block synthesis of cholesterol and increase LDL receptors in liver ↓ LDL ↓ Triglycerides ↑ HDL (small amount) Side Effects: Rash, GI problems, high liver enzymes, myopathy, rhabdomyolysis Nursing Considerations: Well tolerated with few side effects. Monitor liver enzymes and creatine kinase (if muscle weakness or pain occurs).

Fenofibrate (Tricor)

decreases LDL from liver,reduces triglycerides increases HDL s/e rash, mild GI , nausea, diarrhea, high liver enzymes May increase effects of warfarin/Coumadin and other antihyperlipid meds When combined with statins may increase side effects of statins, specially myopathy

dyspepsia

difficult digestion; indigestion

rhabdomyolysis

dissolution of striated muscle (caused by trauma, extreme exertion, or drug toxicity; in severe cases renal failure can result) destruction of muscle to produce myoglobin

EVAR

endovascular aneurysm repair

Coxsackievirus (hand, foot, and mouth disease)

epidemic in children younger than 5 painful vesicles/ulcers in mouth multiple papules or macules on skin, feet, toes, hands, fingers *supportive (hydration, acetaminophen or NSAIDs) *infected children should be kept home until lesions resolve MCC of myocarditis (lymphocyte infiltrate in myocardium) and pericarditis cause: (organism) coxachie a virus symptom: lesions on body treatment/prevention: common in children, treat symptoms Can cause liver disease

CAD s/s that might go unrecognized due to age

fatigue, shortness of breath, upper back pain, indigestion, weakness, sleep problems, palpitations, and anxiety. These atypical symptoms can relate to many different diseases and syndromes.

transjugular biopsy

for patient who have bleeding disorders biopsy consists of obtaining liver tissue through a rigid cannula introduced into a hepatic vein, typically using jugular venous access.

Superficial Vein Thrombosis

formation of a thrombus in a superficial vein, usually the greater or lesser saphenous vein Usual location Typically, superficial leg veins (e.g., varicosities). Sometimes superficial arm veins Clinical findings Tenderness, itchiness, redness, warmth, pain, inflammation, and induration along the course of superficial vein. Vein appears as a palpable cord. Edema rarely occurs. Sequelae If untreated, clot may extend to deeper veins and VTE may occur.

abdominal bruit

heard over the renal arteries may indicate renal disease. It is often present in patients with obstructive sleep apnea.

Ascites, a common manifestation of

hepatitis (especially chronic hepatitis)

cholesterol level greater than 200 mg/dL (5.2 mmol/L)

high

Modifiable risk factors include/CAD

high serum lipids, high BP, tobacco use, physical inactivity, obesity, diabetes, metabolic syndrome, psychologic states, and high homocysteine level.

iliac artery aneurysm

iliac artery: artery that carries oxygenated blood to the hip and groin area For iliac artery aneurysms, a bifurcated graft replaces the entire diseased segment. With saccular aneurysms, it may be possible to excise only the bulbous lesion and repair the artery by primary closure (suturing the artery together) or applying an autogenous or synthetic patch graft.

Nurse provides the client with education about her prescription Rosuvastatin (Crestor) and its benefits of blocking synthesis of cholesterol and

increase LDL receptors in liver,↓ LDL,↓ Triglycerides, ↑ HDL (small amount)

Bile Acid Sequestrants

increase conversion of cholesterol to bile acids in the liver and decrease hepatic cholesterol primary effect is a decrease in total cholesterol and LDL levels. These drugs have been associated with side effects related to taste and a variety of upper and lower GI symptoms. These include belching, heartburn, nausea, abdominal pain, and constipation.

HCV Genotyping - DIAGNOSTIC TEST FOR HEP C

indicates the genotype of HCV directs course of treatment for patients infected with hep C

vasculitis

inflammation of blood vessels

Define thrombophlebitis

inflammation of the vein associated with thrombus formation

major risk factor for HCV is

injection drug use

Etiology and Pathophysiology Atherosclerosis

is the major cause of CAD. It is characterized by lipid deposits within the intima of the artery. Endothelial injury and inflammation play a key role in the development of atherosclerosis. Damage to the endothelial lining can result from tobacco use, hyperlipidemia, hypertension, toxins, diabetes, high homocysteine levels, and infection causing a local inflammatory response

If the blood supply to the bowel is disrupted during abdominal aneurysm repair surgery, may result. and manifestations of?

ischemia or infarction (death) of intestinal tissue complication include absent bowel sounds, fever, abdominal distention and pain, diarrhea, and bloody stools. If bowel infarction occurs, immediate reoperation is needed to restore blood flow, with resection of the infarcted bowel.

Skin manifestations cirrhosis include

jaundice, vascular changes, pruritus, and nail changes.spider angiomata, skin telangiectasias ("paper money skin"), palmar erythema, white nails, disappearance of lunulae, and finger clubbing, especially in the setting of hepatopulmonary syndrome.

Chronic viral hepatitis can

leads to fibrosis and compromised liver function. Fibrosis can lead to cirrhosis and liver failure. Cirrhosis is a generally irreversible condition that can increase one's risk for liver dysfunction, portal hypertension, and primary liver cancer.

Side effects of HMG-CoA Reductase Inhibitors (Statins)

liver damage and myalgia (muscle ache or weakness without breakdown of skeletal muscle) that can progress to rhabdomyolysis (breakdown of skeletal muscle). Liver enzymes are initially monitored and rechecked with any increase in dosage. The creatine kinase isoenzyme, CK-MM (found in skeletal muscle), is assessed if symptoms of myopathy (e.g., muscle aches, weakness) occur.

rubella virus can cause

liver disease

Risk factors for progression to cirrhosis include

male gender, alcohol use, concomitant fatty liver disease, and excess iron deposition in the liver. In some patients, co-infection with HIV may cause complications or require treatment modification.

Bile-acid sequestrants decrease absorption of

many other drugs (e.g., warfarin, thiazides, thyroid hormones, β-adrenergic blockers [β-blockers]). Separating the time of giving these drugs from that of other drugs by at least 2 hours decreases this adverse effect.

HCV infection is more likely than HBV to become chronic

many patients with chronic HCV infection develop chronic liver disease, cirrhosis, portal hypertension, and liver cancer.

cardioprotective effects of estrogen diminsh with

menopause hormone replacement therapy does not protect women after menopause and may be harmful

VV treatement

micronized purified flavonoid fraction, rutosides (e.g., horse chestnut seed extract [Aesculus hippocastanum]), proanthocyanidins (from grapes and apples), and Ruscus (butcher's broom). Therapeutic benefits of venoactive drugs include pain relief, edema reduction, and decreased leg cramping and restless legs. These drugs are widely used in Europe. They are not approved by the FDA. However, many are available over the counter as dietary or herbal supplements.

Thrombolytic Agents Assessment Pulmonary Embolism:

monitor pulse, pressure, hemodynamics, and respiratory status (rate, degree of dyspnea, ABGs).

chronic Hep C infection and alcohol-induced liver disease

most common causes of cirrohsis

A 73-year-old patient with chronic atrial fibrillation develops sudden severe pain, pulselessness, pallor, and coolness in the right leg. The nurse should

notify the health care provider and immediately keep the patient in bed in the supine position.

extreme dieting, malabsorption, and obesity can cause

nutrition-related cirrhosi

IAH Interventions

open (surgical) decompression, percutaneous drainage, and percutaneous drainage combined with a thrombolytic infusion. Conservative measures, such as intubation, ventilation, patient positioning, gastric decompression, cautious fluid resuscitation, pain management, and temporary hemofiltration, are used.

OOB

out of bed (up and about)

lateral malleolus

process forming the outer ankle

The nurse performing an assessment with a patient who has chronic peripheral artery disease (PAD) of the legs and an ulcer on the right second toe would expect to find

prolonged capillary refill in all the toes.

PE

pulmonary embolism

Thrombolytic Agents Patient Teaching:

purpose of medication to report hypersensitivity reactions bedrest and minimal handling during therapy to avoid injury. Avoid all unnecessary procedures such as shaving and vigorous tooth brushing.

Reactive hyperemia

redness of the skin resulting from dilation of the superficial capillaries if blood supply cut off then restored, flow increases above normal a bright red flush on the skin occurring after pressure is relieved

dissolution of striated muscle (caused by trauma, extreme exertion, or drug toxicity; in severe cases renal failure can result)

rhabdomyolysis

Acute Liver Failure(fulminant hepatic failure)

serious condition with a poor prognosis Manifestations include: -encephalopathy, -GI bleeding, -disseminated intravascular coagulation (DIC), - fever with leukocytosis, - renal manifestations (oliguria, azotemia), -ascites, edema, - hypotension, - respiratory failure, -hypoglycemia, -bacterial infections, -thrombocytopenia, - coagulopathies. Liver transplantation is usually the cure for these patients.

Hypercoagulability of Blood*

severe anemias, polycythemia, cancers (e.g., breast, brain, pancreas, GI tract); nephrotic syndrome; high homocysteine levels; and protein C, protein S, and antithrombin deficiency A patient with sepsis is predisposed to hypercoagulability because of endotoxins released from bacteria. Some drugs (e.g., corticosteroids, estrogens) predispose a patient to thrombus formation. Women who use tobacco, are of childbearing age and take estrogen-based oral contraceptives, are postmenopausal on oral hormone therapy, are over 35 years old, and have a family history of VTE are at a very high risk for VTE.17 Women who use oral contraceptives with tobacco double their risk. Smoking causes hypercoagulability by increasing plasma fibrinogen and homocysteine levels and activating the intrinsic coagulation pathway.

TIPS is contraindicated in pts with

severe hepatic encephalopathy,hepatocellular carcinoma, severe hepatorenal syndrome, andportal vein thrombosis

Spider angiomas

small dilated blood vessels on the face and chest occur with portal hypertension or liver disease

What alterations might a nurse find during assessment of the patient's skin and appendages if PVD is due to arterial insufficiency?

smooth, shiney skin; loss of hair; thickened nails

What are some noninvasive treatments for arterial PVD?

stop smoking, topical antibiotic, saline dressing, bed rest/immobilization, finbrinolytic agents if clots - not for Buerger or Raynaud

A bruit, best heard with

the bell of the stethoscope, is a swishing or buzzing sound and indicates turbulent blood flow. Normally you should not hear aortic bruits.

Fatty streaks

the earliest lesion of athersclerosis and can be seen as 15 or early as the second decade of life. They appear as a collection of lipid-laden macrophages (foam cells) in the intima that can eventually progress to athersclerotic plaques

Drug therapy for bleeding varices may include

the somatostatin analog octreotide (Sandostatin) or vasopressin. somatostatin analog octreotide (Sandostatin) or vasopressin (VP). Goal of drug therapy: stop bleeding so that treatment measures can be initiated. IV administration of octreotide and VP produces vasoconstriction of the splanchnic arterial bed,↓ portal blood flow, and ↓ portal HTN. Octreotide is more widely used bc of limited side effects

A patient has a 6-cm thoracic aortic aneurysm that was discovered during a routine chest x-ray. When obtaining an admission history from the patient, it will be most important for the nurse to ask about

trouble swallowing.

venous thromboembolism clinical manifestations

unilateral leg edema, pain , erythma chronic venous insuficency embolism and thrombi fragments

azotemia

urea in the blood

When evaluating the discharge teaching for a patient with chronic peripheral artery disease (PAD), the nurse determines a need for further instruction when the patient says, I will

use a heating pad on my feet at night to increase the circulation and warmth in my feet.

vaccination against HBV reduces the risk for what other hepatitis virus

vaccination against HBV reduces the risk for HDV co-infection. HDV is a defective single-stranded RNA virus that cannot survive on its own. It requires HBV surface Ag to serve as its outer shell. So, only those who are infected with HBV can be infected with HDV. It can be acquired at the same time as HBV or a person with HBV can be infected with HDV later.

Name 3 surgical interventions for venous PVD

vein ligation, thrombectomy, debridement

Describe ulcers associated with PVD of arterial origin.

very painful on lateral lower legs, toes, and heels; demarcated edges; necrotic; not edematous most often occur over bony prominences on the toes, feet, and lower legs Arterial ulcer: Pale ischemic base, well-defined edges usually found on toes, heels, lateral malleoli

Avoid severe hypertension, anuerysm repair

which may stress the arterial anastomoses, resulting in leakage of blood or rupture at the suture lines. Drug therapy with IV diuretics (e.g., furosemide) or IV antihypertensive agents (e.g., labetalol, metoprolol, hydralazine, sodium nitroprusside [Nipride]) may be indicated.

Fibric Acid Derivatives (Fibrates)

work by aiding the removal of VLDLs. They are very effective for lowering triglycerides and increasing HDL levels. They have no effect on LDLs. Although most patients tolerate the drugs well, GI irritability is common. These drugs should be used with caution when combined with statin medications due to increased risk for myopathy.

provide a soft bristtle toothbrush to a patient receiving blood thinner

yes

Mixed α- and β-Blockers carvedilol (Coreg) labetalol

α1-, β1-, and β2-adrenergic blocking properties producing peripheral vasodilation and decreased heart rate Reduce CO, SVR, and BP Monitor pulse and BP regularly Use with caution in patients with diabetes because may depress the tachycardia associated with hypoglycemia and adversely affect glucose metabolism Drug of choice for patients with a history of an MI or HF Less effective BP reduction in black patients Esmolol is for IV use only Lose cardioselectivity at higher doses IV form is available for hypertensive crisis in hospitalized patients Keep patient supine during IV administration Assess patient tolerance of upright position (severe orthostatic hypotension) before allowing upright activities (e.g., commode)

Chronic Hepatitis

• ALT, AST elevations (may be normal in some people) • Ascites and lower extremity edema • Asterixis ("liver flap") • Bleeding abnormalities (thrombocytopenia, easy bruising, prolonged clotting time) • Fatigue, malaise • Hepatic encephalopathy: confusion, difficulty concentrating, easy agitation • Hepatomegaly • Increased bilirubin • Jaundice • Myalgias and/or arthralgias • Palmar erythema • Spider angiomas

Manifestations of Hepatitis Acute Hepatitis

• Anorexia • Clay-colored stools • Dark urine • Decreased sense of taste and smell • Diarrhea or constipation • Fatigue, lethargy, malaise • Flu-like symptoms (e.g., headache) • Hepatomegaly • Jaundice • Low-grade fever • Lymphadenopathy • Myalgias and/or arthralgias • Nausea, vomiting • Pruritus • Right upper quadrant tenderness • Splenomegaly • Weight loss

Hypercoagulability of Blood risk factors

• Antiphospholipid antibody syndrome • Antithrombin III deficiency • Cancer (especially breast, brain, hepatic, pancreatic, GI) • Dehydration or malnutrition • Elevated (clotting) factor VIII or lipoprotein (a) • Erythropoiesis-stimulating drugs (e.g., epoetin alfa [Procrit]) • High altitudes • Hormone therapy • High homocysteine levels • Nephrotic syndrome • Oral contraceptives, especially in women older than 35 years who use tobacco • Polycythemia vera • Pregnancy and postpartum period • Protein C deficiency • Protein S deficiency • Sepsis • Severe anemias • Tobacco use

Drug Alert ClopidogrelPlavix) and Omeprazole (Prilosec)

• Antiplatelet effect of clopidogrel is reduced by about half when given with omeprazole. • This reduced effect increases the risk for myocardial infarction (MI) and stroke

Drug Alert Anticoagulant Therapy

• Avoid IM injections. • Observe closely for any signs of bleeding: hypotension, tachycardia, hematuria, melena, hematemesis, petechiae, bruising, oozing or visible bleeding from trauma site or surgical incision. • Tell patients to report bleeding: black or bloody stools, bleeding gums, bloody urine or sputum, coffee-ground or bloody vomit, excessive bruising, nosebleeds, excessive menstrual bleeding. • Assess for mental status changes, especially in the older patient, since this may indicate cerebral bleeding. • Tell patients to avoid taking aspirin, NSAIDs, fish oil supplements, garlic supplements, ginkgo biloba, and certain antibiotics (e.g., sulfamethoxazole and trimethoprim [Bactrim]).

Low-Molecular-Weight Heparin† dalteparin (Fragmin) enoxaparin (Lovenox)

• Binds to antithrombin III, enhancing its effect • Heparin-antithrombin III complex inactivates activated factor X and thrombin. • Prevents conversion of fibrinogen to fibrin The low molecular weight heparins enoxaparin, dalteparin and tinzaparin are purified fragments of natural heparins that have anticoagulant activity and are used to treat patients at high risk of venous thrombosis. Use extreme caution in patients with a history of HIT. Antidote: Protamine neutralizes the effects of LMWH. The antidote for warfarin-related bleeding is vitamin K, Patients who have had HIT should never receive heparin or low-molecular-weight heparin (LMWH) again. This should be clearly marked in the patient's medical record.

HAV sources of infection

• Contaminated food, milk, water, shellfish • Crowded conditions (e.g., day care, nursing home) • Persons with subclinical infections, infected food handlers, sexual contact, IV drug users • Poor personal hygiene • Poor sanitation

HBV sources of infection

• Contaminated needles, syringes, and blood products • HBV-infected mother (perinatal transmission) • Sexual activity with infected partners. Asymptomatic carriers • Tattoos or body piercing with contaminated needles

Drug Alert Cilostazol

• Contraindicated in patients with HF.

Hepatitis testing

• Hepatitis A: Anti-HAV IgM, anti-HAV IgG, or HAV total antibody • Hepatitis B: HBsAg, anti-HBs, HBeAg, anti-HBe, anti-HBc IgM and IgG, HBV DNA quantitation, HBV genotyping • Hepatitis C: Anti-HCV, HCV RNA quantitation, HCV genotyping • Hepatitis D: Anti-HDV, HDV Ag

Drug TherapyChronic Stable Angina and Acute Coronary Syndrome Angiotensin II Receptor Blockers candesartan (Atacand) irbesartan (Avapro) losartan (Cozaar) olmesartan (Benicar) valsartan (Diovan)

• Inhibits binding of angiotensin II to angiotensin I receptors, resulting in vasodilation • Used for patients intolerant of ACE inhibitors

Antiplatelet Agents aspirin

• Inhibits cyclooxygenase, which in turn produces thromboxane A2, a potent platelet activator • Should be given as soon as ACS is suspected, unless truly allergic

Antiplatelet Agents ticagrelor (Brilinta)

• Inhibits platelet aggregation • Alternative for patient who cannot take aspirin • Used in combination with low-dose aspirin to treat ACS with or without PCI • Effectiveness may be decreased by aspirin dosages >100 mg/day

Antiplatelet Agents clopidogrel (Plavix)

• Inhibits platelet aggregation • Alternative for patients who cannot take aspirin • Used in combination with low-dose aspirin to treat ACS with or without PCI and after elective PCI

Antiplatelet Agents cangrelor (Kengreal)

• Inhibits platelet aggregation • Given IV • Approved for use in patients having PCI during procedure

Antiplatelet Agents vorapaxar (Zontivity)

• Inhibits platelet aggregation • Increases risk for bleeding, including life-threatening and fatal bleeding (boxed warning) • Must not be used in people who have had a stroke, transient ischemic attack (TIA), or bleeding in the head, because the risk for bleeding in the head is too great

Antiplatelet Agents prasugrel (Effient)

• Inhibits platelet aggregation • Used as an alternative to clopidogrel in combination with aspirin for patient with ACS who had PCI • Not indicated to treat ACS without PCI or for use after elective PCI

β-Adrenergic Blockers atenolol (Tenormin) carvedilol (Coreg) metoprolol (Lopressor) nadolol (Corgard)

• Inhibits sympathetic nervous stimulation of the heart • Reduces heart rate, contractility, and BP • Reduces ischemia • Decreases afterload • Cardioselective β-blockers include atenolol and metoprolol The most common side effects of beta-blockers are: Fatigue and dizziness. Beta-blockers slow down your heart rate. ... Poor circulation. Your heart beats more slowly when you take beta-blockers. ... Gastrointestinal symptoms. These include upset stomach, nausea, and diarrhea or constipation. ... Sexual dysfunction. ... Weight gain.

Vitamin K Antagonist† warfarin (Coumadin)

• Interferes with hepatic synthesis of vitamin K-dependent clotting factors • Used for patients with atrial fibrillation or those at risk for thromboembolism (large anterior wall infarctions or ventricular aneurysms) • Closely monitor for bleeding when taking with antiplatelet drugs

Drug Alert Ribavirin (Rebetol, Ribasphere)

• May cause severe birth defects. During treatment, women taking the drug and women whose male partners are taking the drug must avoid pregnancy. • Monitor hemoglobin and hematocrit as it may cause anemia. • Used only for specific populations.

Drug Alert: Gemfibrozil (Lopid)

• May increase the risk for bleeding in patients taking warfarin (Coumadin) • Increases the risk of hypoglycemia in patients taking repaglinide (Prandin)

Drug Alert Horse Chestnut Seed Extract (Aesculus hippocastanum)

• May interact with lithium and antidiabetes, antiplatelet, and anticoagulant drugs. • Should not be taken by persons with liver or kidney disease or with a latex allergy.

Calcium Channel Blockers amlodipine (Norvasc) diltiazem (Cardizem) felodipine nicardipine (Cardene) verapamil (Calan) nifedipine (Procardia)

• Prevents calcium entry into vascular smooth muscle cells and myocytes (cardiac cells) • May prevent or control coronary vasospasm • Promotes coronary and peripheral vasodilation • Reduces HR, contractility, and BP Use with caution in patients with HF Grapefruit juice may increase serum concentrations and toxicity of certain calcium channel blockers; avoid concurrent use Used for supraventricular tachydysrhythmias Avoid in patients with second- or third-degree AV block or left ventricular systolic dysfunction Full effect on BP may not be seen for 3-6 wk Do not affect bradykinin levels, therefore an acceptable alternative to ACE inhibitors in people who develop a dry cough In patients with kidney disease, ACE inhibitors and ARBs should not be used together due to adverse renal effects

Drug TherapyChronic Stable Angina and Acute Coronary Syndrome Angiotensin-Converting Enzyme (ACE) Inhibitors benazepril (Lotensin) captopril enalapril (Vasotec) lisinopril (Zestril) quinapril (Accupril) ramipril (Altace)

• Prevents conversion of angiotensin I to angiotensin II, resulting in vasodilation • May prevent or limit ventricular remodeling • Decreases endothelial dysfunction • Useful with HF, decreased LV function, tachycardia, MI, hypertension, diabetes, and chronic kidney disease

Unfractionated Heparin† heparin

• Prevents conversion of fibrinogen to fibrin and prothrombin to thrombin

Glycoprotein IIb/IIIa Inhibitors abciximab (ReoPro) eptifibatide (Integrilin) tirofiban (Aggrastat)

• Prevents the binding of fibrinogen to platelets, thereby blocking platelet aggregation • May be used in combination with aspirin for patients with ACS or during PCI

Nitrates Sublingual nitroglycerin (Nitrostat) Translingual spray nitroglycerin (Nitrolingual) nitroglycerin ointment Transdermal nitroglycerin isosorbide dinitrate (Isordil) isosorbide mononitrate IV nitroglycerin

• Promotes peripheral vasodilation, decreasing preload and afterload • Promotes coronary artery vasodilation • May prevent or control coronary vasospasm IV use for hypertensive crisis in hospitalized patients with myocardial ischemia Given by continuous IV infusion with pump or control device

Treatment of claudication symptoms

• Structured walking or exercise program • Cilostazol or pentoxifylline

Drug Alert: Niacin (Niaspan)

• Teach the patient that flushing (especially of face and neck) may occur within 20 minutes after taking drug and last for 30 to 60 minutes. • Premedicate with aspirin or NSAID 30 minutes before taking to reduce flushing. • Use of extended-release niacin may decrease side effects.

Drug Alert Doxazosin (Cardura)

• Use caution when giving the first dose. It is best to give the first dose at bedtime to reduce the first dose BP drop. • Syncope occasionally occurs 30 to 90 min after the first dose, a too-rapid increase in dose, or addition of another antihypertensive agent to therapy. • Drug interactions (severe low BP) can occur with patients taking phosphodiesterase inhibitors, such as sildenafil (Viagra) or tadalafil (Cialis).

Management Acute and Chronic for hepatitis

• Well-balanced diet • Vitamin supplements • Rest (degree of strictness varies) • Avoiding alcohol and drugs detoxified by liver

ACE inhibitors Ramipril (Altace) PAD

↓ Cardiovascular morbidity ↓ Mortality ↑ Peripheral blood flow ↑ ABI ↑ Walking distance

Omega-3 Fatty Acid omega-3 acid ethyl esters (Lovaza) (contains eicosapentaenoic acid [EPA] and docosahexaenoic acid [DHA])

↓ Triglycerides ↑ HDL Anaphylaxis, rash, taste changes, GI problems (e.g., constipation, vomiting) Used for patients with high triglycerides. Give with meals and do not open or dissolve capsules


Set pelajaran terkait

Chapter 4: Financial Services- Savings Plans and Payment Accounts

View Set

MedSurg - 1 Health Care Delivery & Evidence-Based Practice

View Set

Aortic Regurgitation Case Study |, Respiratory Assessment, Coronary Artery Disease

View Set

History 11-5 Guided questions STARRED

View Set

6.G.1 Area of Regular and Irregular Polygons

View Set